You are on page 1of 76

CÂU HỎI GỢI Ý MÔN TẬP LÝ THUYẾT

MÔN: LUẬT SỞ HỮU TRÍ TUỆ

1. Phân tích khái niệm quyền sở hữu trí tuệ


Theo khoản 1 Điều 4 Luật sở hữu trí tuệ sửa đổi, bổ sung năm 2009, Quyền sở hữu trí tuệ là
quyền của tổ chức, cá nhân đối với tài sản trí tuệ, bao gồm quyền tác giả và quyền liên quan đến
quyền tác giả, quyền sở hữu công nghiệp và quyền đối với giống cây trồng.
Có thể thấy việc một tổ chức hay cá nhân có công sức nghiên cứu, sáng tạo để hoàn thành một
công trình, một sản phẩm mới sẽ được công nhận quyền sở hữu trí tuệ. Các tác phẩm sẽ được
pháp luật bảo hộ trên các phương diện như quyền sở hữu trí tuệ của tác giả, quyền liên quan đến
quyền tác giả, quyền sở hữu công nghiệp, quyền sở hữu trí tuệ đối với giống cây trồng.
Quyền SHTT có thể được hiểu theo hai phương diện:
– Phương diện khách quan: Quyền SHTT là tổng hợp các quy phạm pháp luật điều chỉnh các
quan hệ xã hội phát sinh trong quá trình xác lập và khai thác, sử dụng và định đoạt các đối tượng
SHTT.
– Phương diện chủ quan: Quyền SHTT là tập hợp các quyền và nghĩa vụ cụ thể của các cá nhân,
tổ chức là chủ thể của quyền SHTT.
2. Phân tích các đặc trưng của quyền sở hữu trí tuệ?
1. Quyền sở hữu trí tuệ là một quan hệ pháp luật đặc biệt vì khách thể của quyền này không phải
vật cụ thể mà là sản phẩm của lao động sáng tạo được thể hiện dưới dạng phi vật chất. Nó được
vật chất hoá khi được mang ra áp dụng vào sản xuất, kinh doanh. Khách thể của quyền sở hữu trí
tuệ được phân thành hai nhóm chính, đó là nhóm vận dụng trong đời sống tinh thần của con
người và làm phong phú hơn nhu cầu tinh thần của con người và nhóm được áp dụng vào sản
xuất, kinh doanh để tạo ra các sản phẩm vật chất mang tính công nghệ.

2. Các quyền chiếm hữu, sử dụng và định đoạt trong quyền sở hữu trí tuệ được chia thành hai
nhóm quyền, đó là quyền tài sản và quyền nhân thân. Giữa quyền nhân thân và quyền tài sản
luôn tồn tại mối quan hệ hữu cơ. Quyền này là tiền đề của quyền kia. Quyền tài sản chỉ có thể
xác định cho một chủ thể nhất định dựa trên căn cứ quyền nhân thân có mối liên hệ trực tiếp với
tài sản. Những quyền liên quan đến nhân thân người sáng tạo không thể tách rời và thuộc về
người sáng tạo vĩnh viễn như quyền đứng tên tác giả, quyền đặt tên tác phẩm, những quyền tài
sản có thể chuyển giao cho người khác thông qua các hợp đồng chuyển giao quyền.

3. Quyền sở hữu trí tuệ không mang tính tuyệt đối và vô thời hạn như quyền sở hữu các tài sản
hữu hình, vì thế, quyền sở hữu trí tuệ luôn luôn bị giới hạn lợi ích cộng đồng. Do đó, tác giả, chủ
sở hữu một tác phẩm không thể có độc quyền chiếm hữu, sử dụng và định đoạt vô thời hạn đối
với tác phẩm. Độc quyền này chỉ tồn tại trong thời hạn nhất định, trong khoảng thời gian nhất
định và giới hạn về điều kiện sử dụng.

4. Tài sản trí tuệ cũng mang tính thương mại khi nền kinh tế phát triển mạnh mẽ. Vì vậy, pháp
luật phải có cách tiếp cận mới đối với quyền sở hữu trí tuệ, tính thương mại của quyền sở hữu trí
tuệ.

1
5. Quyền sở hữu trí tuệ dễ bị xâm phạm, những sản phẩm sáng tạo của trí tuệ một khi đã được
công khai sẽ dễ dàng được phổ biến và bị khai thác giá trị kinh tế thông qua hệ thống thông tin
của một quốc gia, một khu vực, của các tổ chức quốc tế. Quyền sở hữu trí tuệ được ghi nhận và
bảo hộ ở một quốc gia không có nghĩa là sẽ được bảo hộ ở các quốc gia khác, việc xâm phạm có
thể diễn ra ngay trước mắt chủ sở hữu tại quốc gia khác mà không hề bị coi là phạm pháp. Vì
vậy, trong điều kiện hiện nay, việc bảo hộ quyền sở hữu trí tuệ trên quy mô quốc tế thông qua
các điều ước quốc tế là cần thiết.

3. Phân loại các đối tượng của quyền sở hữu trí tuệ?
Theo Điều 3 Luật sở hữu trí tuệ, đối tượng quyền sở hữu trí tuệ bao gồm các đối tượng sau:
1. Đối tượng quyền tác giả
– Quyền tác giả là quyền của tổ chức, cá nhân đối với tác phẩm do mình sáng tạo ra hoặc
sở hữu.
– Quyền liên quan đến quyền tác giả (sau đây gọi là quyền liên quan) là quyền của tổ
chức, cá nhân đối với cuộc biểu diễn, bản ghi âm, ghi hình, chương trình phát sóng, tín
hiệu vệ tinh mang chương trình được mã hóa.
Đối tượng quyền tác giả bao gồm tác phẩm văn học, nghệ thuật, khoa học; đối tượng
quyền liên quan đến quyền tác giả bao gồm cuộc biểu diễn, bản ghi âm, ghi hình, chương
trình phát sóng, tín hiệu vệ tinh mang chương trình được mã hóa.
2. Đối tượng quyền sở hữu công nghiệp
– Quyền sở hữu công nghiệp là quyền của tổ chức, cá nhân đối với sáng chế, kiểu dáng
công nghiệp, thiết kế bố trí mạch tích hợp bán dẫn, nhãn hiệu, tên thương mại, chỉ dẫn địa
lý, bí mật kinh doanh do mình sáng tạo ra hoặc sở hữu và quyền chống cạnh tranh không
lành mạnh.
Đối tượng quyền sở hữu công nghiệp bao gồm sáng chế, kiểu dáng công nghiệp, thiết kế
bố trí mạch tích hợp bán dẫn, bí mật kinh doanh, nhãn hiệu, tên thương mại và chỉ dẫn
địa lý.
3. Đối tượng quyền đối với giống cây trồng
Quyền đối với giống cây trồng là quyền của tổ chức, cá nhân đối với giống cây trồng mới
do mình chọn tạo hoặc phát hiện và phát triển hoặc được hưởng quyền sở hữu.
Đối tượng quyền đối với giống cây trồng là vật liệu nhân giống và vật liệu thu hoạch.
Căn cứ phát sinh, xác lập quyền sở hữu trí tuệ đối với quyền tác giả và quyền liên quan, quyền sở
hữu công nghiệp, quyền đối với giống cây trồng được xác lập như sau:
1. Quyền tác giả và quyền liên quan
– Quyền tác giả phát sinh kể từ khi tác phẩm được sáng tạo và được thể hiện dưới một
hình thức vật chất nhất định, không phân biệt nội dung, chất lượng, hình thức, phương
tiện, ngôn ngữ, đã công bố hay chưa công bố, đã đăng ký hay chưa đăng ký.
– Quyền liên quan phát sinh kể từ khi cuộc biểu diễn, bản ghi âm, ghi hình, chương trình
phát sóng, tín hiệu vệ tinh mang chương trình được mã hóa được định hình hoặc thực
hiện mà không gây phương hại đến quyền tác giả.

2
2. Quyền sở hữu công nghiệp
Quyền sở hữu công nghiệp được xác lập như sau:
– Quyền sở hữu công nghiệp đối với sáng chế, kiểu dáng công nghiệp, thiết kế bố trí,
nhãn hiệu được xác lập trên cơ sở quyết định cấp văn bằng bảo hộ của cơ quan nhà nước
có thẩm quyền theo thủ tục đăng ký quy định tại Luật sở hữu trí tuệ hoặc công nhận đăng
ký quốc tế theo điều ước quốc tế mà Cộng hòa xã hội chủ nghĩa Việt Nam là thành viên.
Quyền sở hữu công nghiệp đối với nhãn hiệu nổi tiếng được xác lập trên cơ sở sử dụng,
không phụ thuộc vào thủ tục đăng ký.
Quyền sở hữu công nghiệp đối với chỉ dẫn địa lý được xác lập trên cơ sở quyết định cấp
văn bằng bảo hộ của cơ quan nhà nước có thẩm quyền theo thủ tục đăng ký quy định tại
Luật sở hữu trí tuệ hoặc theo điều ước quốc tế mà Cộng hòa xã hội chủ nghĩa Việt Nam
là thành viên;
– Quyền sở hữu công nghiệp đối với tên thương mại được xác lập trên cơ sở sử dụng hợp
pháp tên thương mại đó;
– Quyền sở hữu công nghiệp đối với bí mật kinh doanh được xác lập trên cơ sở có được
một cách hợp pháp bí mật kinh doanh và thực hiện việc bảo mật bí mật kinh doanh đó;
– Quyền chống cạnh tranh không lành mạnh được xác lập trên cơ sở hoạt động cạnh tranh
trong kinh doanh.
3. Quyền đối với giống cây trồng
Quyền đối với giống cây trồng được xác lập trên cơ sở quyết định cấp Bằng bảo hộ giống
cây trồng của cơ quan nhà nước có thẩm quyền theo thủ tục đăng ký quy định tại Luật sở
hữu trí tuệ.
4. Phân tích ý nghĩa của việc bảo hộ quyền sở hữu trí tuệ?
● Đối với chủ thể quyền sở hữu trí tuệ

Bảo hộ quyền sở hữu trí tuệ khuyến khích tạo động lực cho sự sáng tạo, thúc đẩy những nỗ lực,
cống hiến của nhiều cá nhân vào hoạt động cải tiến kỹ thuật, nghiên cứu khoa học nhằm tạo ra
những sản phẩm vật chất và tinh thần cho xã hội.

Sở hữu trí tuệ là kết quả của một quá trình sáng tạo, đầu tư trí tuệ, tiền bạc, công sức của cá
nhân, tổ chức. Hoạt động sáng tạo trí tuệ mong muốn đạt được những lợi ích nhất định trong việc
nghiên cứu. Bằng việc bảo hộ tài sản trí tuệ, nhà nước khuyến khích và hỗ trợ những tổ chức, cá
nhân sáng tạo nhiều hơn. Bởi bảo hộ tài sản trí tuệ sẽ đảm bảo quyền ( quyền nhân thân và quyền
tài sản) của các chủ sở hữu đối với sản phẩm mà mình sáng tạo ra.

● Đối với chủ thể sản xuất, kinh doanh

Bảo vệ quyền sở hữu trí tuệ cũng góp phần giảm thiểu tổn thất cho các nhà sản xuất, kinh doanh
và thúc đẩy họ phát triển sản xuất và kinh doanh hợp pháp.

3
● Đối với người tiêu dùng

Bảo vệ quyền sở hữu trí tuệ hiệu quả sẽ giúp cho người tiêu dùng có cơ hội lựa chọn và được sử
dụng các dịch vụ, hàng hóa chất lượng cao, đáp ứng được các nhu cầu của người tiêu dùng. Bảo
vệ quyền sở hữu trí tuệ đã hạn chế được các hành vi vi phạm sở hữu trí tuệ, tạo ra hàng nhái hay
hàng kém chất lượng và các hành vi cạnh tranh không lành mạnh khác.

● Đối với quốc gia

Sở hữu trí tuệ được khẳng định là “một công cụ đắc lực để phát triển kinh tế”, cho nên việc bảo
vệ quyền sở hữu trí tuệ hiệu quả sẽ tạo ra môi trường cạnh tranh lành mạnh. Đây là động lực thúc
đẩy tăng trưởng kinh tế, thu hút chuyển giao công nghệ, nhận được đầu tư nước ngoài.

Trong bối cảnh hội nhập quốc tế hiện nay, cùng với sự luân chuyển mạnh mẽ và liên tục của các
tài sản hữu hình cũng như tài sản vô hình giữa các quốc gia, việc bảo vệ quyền sở hữu trí tuệ còn
góp phần bảo vệ lợi ích quốc gia.

● “Bảo hộ quyền sở hữu trí tuệ là nghĩa vụ bắt buộc và là điều kiện tiên quyết đối với các
quốc gia là thành viên của Tổ chức thương mại thế giới và với những quốc gia muốn trở
thành thành viên của Tổ chức này. Nhiều nước, đặc biệt là các nước phát triển, đã coi
việc bảo hộ quyền sở hữu trí tuệ là một điều kiện không thể thiếu để thiết lập mối quan hệ
thương mại, việc thực hiện không đầy đủ về việc bảo hộ quyền sở hữu trí tuệ có thể tạo ra
sự căng thẳng về thương mại’’.

Bên cạnh đó, bảo vệ quyền sở hữu trí tuệ còn góp phần trong việc thúc đẩy hoạt động thương
mại lành mạnh trên phạm vi toàn cầu.

5. Phân tích khái niệm quyền tác giả.


Theo khái niệm chung của các nước thì quyền tác giả là quyền sở hữu của cá nhân, pháp
nhân đối với tác phẩm văn học, nghệ thuật và khoa học do mình sáng tạo ra hoặc sở hữu,
thường gọi là bản quyền tác giả.
Quyền tác giả được xác lập đối với những người sáng tạo ra tác phẩm gốc về văn học,
kịch, âm nhạc, nghệ thuật.
Quyền này cho phép người sáng tạo kiểm soát được việc khai thác, sao chép, cải biên,
công bố tác phẩm của mình. Quyền tác giả bảo hộ sự biểu hiện ý tưởng (tác phẩm) về văn
hóa nghệ thuật (có thể hai tác phẩm có ý tưởng giống sau nhưng thể hiện khác nhau vẫn được
bảo hộ)
Quyền tác giả được phát sinh kể từ khi tác phẩm được công bố không bắt buộc phải đăng
ký, nhưng nếu được đăng ký thì thủ tục bảo hộ quyền tác giả được chặt chẽ hơn. Việc bảo hộ
quyền tác giả đã mang lại những lợi ích to lớn cho bản thân tác giả và cho cả xã hội, không
chỉ tạo ra những giá trị về tinh thần, về tài sản mà còn thúc đẩy sự sáng tạo của con người,
góp phần phát triển nền văn hóa của một quốc gia.
Luật về quyền tác giả không chỉ bảo vệ quyền lợi cá nhân của tác giả trong một giới hạn
nhất định mà còn bảo vệ lợi ích chung của cộng đồng. Theo Luật Sở hữu trí tuệ Việt Nam thì
quyền tác giả cũng bao gồm quyền nhân thân và quyền tài sản của tác giả đối với tác phẩm
do mình sáng tạo.

4
● Quyền nhân thân gồm các quyền:
- Đặt tên cho tác phẩm; đứng tên thật hoặc bút danh trên tác phẩm, nêu tên thật
hoặc bút danh khi tác phẩm được công bố, sử dụng; công bố, phổ biến hoặc cho
người khác công bố, phổ biến tác phẩm của mình; bảo vệ sự toàn vẹn của tác
phẩm, không cho phép người khác sửa chữa, cắt xén hoặc xuyên tạc tác phẩm
dưới bất kỳ hình thức nào gây phương hại đến danh dự và uy tín của tác giả
- Tác giả không đồng thời là chủ sở hữu tác phẩm có các quyền nhân thân đối với
tác phẩm mà mình sáng tạo gồm: đặt tên cho tác phẩm; đứng tên thật hoặc bút
danh trên tác phẩm; bảo vệ sự toàn vẹn tác phẩm, cho phép hoặc không cho phép
người khác sửa đổi nội dung của tác phẩm.
- Chủ sở hữu tác phẩm không đồng thời là tác giả có các quyền nhân thân đối với
tác phẩm gồm: công bố, phổ biến hoặc cho người khác công bố, phổ biến tác
phẩm thuộc sở hữu của mình, trừ trường hợp giữa tác giả và chủ sở hữu có thỏa
thuận khác; cho hoặc không cho người khác sử dụng tác phẩm thuộc quyền sở
hữu của mình, trừ trường hợp giữa tác giả và chủ sở hữu có thỏa thuận khác
● Quyền tài sản của tác giả bao gồm:
- Đối với tác giả đồng thời là chủ sở hữu tác phẩm có các quyền như được hưởng
nhuận bút; được hưởng thù lao khi tác phẩm được sử dụng; được hưởng lợi ích
vật chất từ việc cho người khác sử dụng tác phẩm dưới các hình thức như xuất
bản, tái bản, trưng bày, triển lãm, biểu diễn, phát thanh, truyền hình, ghi âm, ghi
hình, chụp ảnh; dịch, phóng tác, cải biên, chuyển thể, cho thuê; được nhận giải
thưởng đối với tác phẩm mà mình là tác giả
- Đối với tác giả không đồng thời là chủ sở hữu tác phẩm có các quyền tài sản đối
với tác phẩm mà mình là tác giả gồm: được hưởng nhuận bút; được hưởng thù lao
khi tác phẩm được sử dụng; được nhận giải thưởng đối với tác phẩm mà mình là
tác giả.
- Chủ sở hữu không đồng thời là tác giả được hưởng lợi ích vật chất từ việc sử
dụng tác phẩm dưới các hình thức xuất bản, tái bản, trưng bày, triển lãm, biểu
diễn, phát thanh, truyền hình, ghi âm, ghi hình, chụp ảnh; dịch, phóng tác, cải
biên, chuyển thể, cho thuê.
Như vậy, về cơ bản nội dung quyền tác giả quy định trong Luật sở hữu trí tuệ Việt Nam
phù hợp với thông lệ chung của luật pháp quốc tế. Pháp luật Việt Nam quy định rõ quyền
nhân thân bao gồm quyền nhân thân không gắn với tài sản và quyền nhân thân gắn với tài
sản. Các quyền nhân thân không gắn với tài sản là những quyền gắn liền với các giá trị
nhân thân của tác giả và không thể chuyển giao, bao gồm ba quyền: quyền được đặt tên
tác phẩm, đứng tên tác phẩm và bảo vệ sự toàn vẹn nội dung tác phẩm. Quyền nhân thân
gắn với tài sản là quyền cho hay không cho người khác sử dụng tác phẩm, làm phát sinh
bản chất độc quyền của quyền tác giả

6. Phân tích nguyên tắc cân bằng lợi ích trong bảo hộ quyền tác giả?
Trong pháp luật về SHTT, nguyên tắc cân bằng lợi ích được thể hiện ở các khía cạnh, bao
gồm: Quy định về giới hạn của chủ sở hữu quyền SHTT về thời hạn bảo hộ; Quy định về sử
dụng đối tượng quyền SHTT (xin phép sử dụng, trả thù lao, nhuận bút,..); Quy định về các
trường hợp bắt buộc phải chuyển giao, có thể chuyển giao,..
Theo Tổ chức sở hữu trí tuệ thế giới (WIPO) định nghĩa: “Quyền tác giả là một thuật ngữ
pháp lý chí quyền của người sáng tác đối với tác phẩm văn học và nghệ thuật của họ.” Pháp luật

5
về quyền tác giả trong SHTT trên toàn thế giới thống nhất chỉ bảo hộ hình thức thể hiện ý tưởng
(Expression) là tác phẩm chứ không bảo hộ ý tưởng (Idea) tức là chủ đề, thông điệp tác phẩm. Ở
đây, tác phẩm được hiểu là sản phẩm sáng tạo trong lĩnh vực văn học, nghệ thuật và khoa học thể
hiện bằng bất kỳ phương tiện hay hình thức nào.
Để một tác phẩm được bảo hộ cần đáp ứng hai tiêu chí cơ bản theo pháp luật quốc tế, bao
gồm: Tính nguyên gốc (Originality), Tính định hình (Fixation). Các quốc gia có thể áp dụng
thêm các điều kiện bảo hộ khác, nhưng cần đảm bảo 2 điều kiện cơ bản trên. Theo pháp luật về
quyền tác giả trong sở hữu trí tuệ Việt Nam thì có thêm 2 tiêu chí là tác phẩm thuộc các loại hình
được bảo hộ, không nằm trong danh mục các đối tượng không thuộc phạm vi bảo hộ quyền tác
giả.
Tác phẩm là sự sáng tạo của trí tuệ và là thành quả của sự lao động miệt mài, sử dụng
công, đầu tư cả về mặt chất xám và kinh tế để tạo ra. Đây là tài sản vô hình, rất dễ bị xâm phạm
nên đòi hỏi một cơ chế bảo hộ độc quyền có khả năng tạo ra môi trường pháp lý để bảo vệ quyền
nhân thân và quyền tài sản của tác giả. “Độc quyền là sự trao thưởng của toàn xã hội, mà đại
diện là Nhà nước cho công sức, vốn đầu tư mà các chủ thể quyền SHTT đã bỏ ra để nghiên cứu,
sáng tạo”. Thời hạn bảo hộ cần phải đủ để các tác giả, chủ sở hữu trí tuệ khai thác các đối tượng
được bảo hộ của quyền SHTT không chỉ nhằm bù đắp các chi phí để tạo ra chúng mà còn đảm
bảo cho họ khả năng thu lợi nhuận từ các đối tượng này. “Một tác phẩm thật giá trị, phải vượt
lên trên tất cả bờ cõi và giới hạn, phải là tác phẩm chung cho cả loài người”. Hết thời hạn bảo
hộ độc quyền, tác phẩm sẽ được công chúng tiếp tục đón nhận, khai thác và sáng tạo để phát
triển hơn nữa. Tuy nhiên, việc khai thác giá trị thương mại phải duy trì lợi ích chính đáng của tác
giả (quyền nhân thân). Đây cũng chính là tinh thần mà nguyên tắc cân bằng lợi ích thể hiện, các
chủ thể không triệt tiêu lợi ích của nhau mà cùng phát triển. Theo WIPO, sự bảo hộ cân bằng
trong quyền tác giả thường được thể hiện bởi quy định về giới hạn quyền của tác giả, chủ sở hữu
và các trường hợp ngoại lệ.
7. Vì sao nói sự phát triển của pháp luật về quyền tác giả song hành với sự phát triển của
công nghệ?
Ngày nay, công nghệ ngày một phát triển đem đến nhiều thành tựu to lớn cho xã hội
nhưng cũng vì thế mà không ít cá nhân lợi dụng điều đó để trục lợi. Nhất là trong lĩnh vực
bảo vệ bản quyền hay quyền tác giả gặp nhiều khó khăn. Sự hình thành và phát triển của
pháp luật về quyền tác giả nói riêng và quyền sở hữu trí tuệ nói chung luôn gắn chặt với sự
phát triển của khoa học và công nghệ. Môi trường kỹ thuật số tạo cơ hội cho người sử dụng
tiếp cận các tác phẩm nhanh nhất, đồng thời cũng làm gia tăng quy mô của hoạt động sao
chép, vi phạm quyền tác giả. Kỹ thuật số, công nghệ mang đến cơ hội mới trong cách thức
thể hiện tác phẩm, đồng thời đã làm thay đổi cách thức lưu giữ tác phẩm, bản ghi âm, chương
trình phát sóng, dạng vật chất chứa đựng tác phẩm thay đổi. Với những thành tựu sáng tạo
của công nghệ thông tin, con người có thể tiếp cận, khai thác, sử dụng dễ dàng các nguồn
thông tin, điều này đồng nghĩa với việc vi phạm quyền tác giả cũng có thể xảy ra một cách dễ
dàng và phổ biến.
Chính vì thế, từ phía các cơ quan chức năng, cần có những chính sách, cơ chế phù hợp,
chặt chẽ nhằm bảo vệ quyền lợi hợp pháp của người sáng tạo, có như vậy mới khuyến khích
được hoạt động sáng tạo. Vậy nên sự phát triển về quyền tác giả song hành với sự phát triển
của công nghệ.
8. Phân tích điều kiện bảo hộ quyền tác giả: tác phẩm phải có tính nguyên gốc.
Để được bảo hộ quyền tác giả, tác phẩm phải có tính nguyên gốc. Theo pháp luật về
quyền tác giả, tính nguyên gốc liên quan đến hình thức thể hiện ý tưởng chứ không liên quan

6
đến bản thân ý tưởng hay ý nghĩa bên trong nó. Tuy nhiên, ý nghĩa chính xác về tính nguyên
gốc là khác nhau trong pháp luật về quyền tác giả của mỗi nước. Tính nguyên gốc có nghĩa là
tác phẩm được sáng tạo ra một cách độc lập và không sao chép từ bất kỳ một tác phẩm nào
khác. Việc bảo hộ quyền tác giả chỉ được áp dụng đối với những đóng góp mang tính nguyên
gốc cho tác phẩm và không được áp dụng đối với bất kỳ yếu tố vay mượn từ tác phẩm khác.
Tính nguyên gốc được xác định trên cơ sở lao động sáng tạo mang dấu ấn cá nhân của
chủ thể. Việc quyền liên quan phát sinh trên cơ sở hành vi sử dụng tác phẩm đã có từ trước
không ảnh hưởng tới tính nguyên gốc của các đối tượng mà nó bảo hộ. Không khó để nhận ra
rằng, sự trình diễn của người nghệ sĩ là kết quả của những cống hiến nghệ thuật mang tính
chất sáng tạo của họ. Cùng là một tác phẩm, nhưng mỗi người biểu diễn sẽ thể hiện theo cách
riêng của mình, thậm chí cũng chính người nghệ sĩ ấy nhưng mỗi lần biểu diễn lại đem đến
những cảm nhận riêng cho khán giả. Chính sự sử dụng tác phẩm một cách sáng tạo và mang
đậm dấu ấn cá nhân của chủ thể quyền liên quan đòi hỏi việc sử dụng ấy cũng phải được
pháp luật bảo vệ.
Tính nguyên gốc của quyền liên quan còn được thể hiện ở việc quyền liên quan chỉ được
xác định theo các đối tượng của quyền liên quan được tạo ra lần đầu tiên. Việc xác định tính
nguyên gốc từ góc độ này thường được áp dụng đối với các bản ghi, chương trình phát thanh
truyền hình. Cụ thể, quyền liên quan đối với bản ghi âm, ghi hình chỉ được xác định cho
người tạo ra bản ghi âm, ghi hình lần đầu âm thanh, hình ảnh của cuộc biểu diễn hoặc các âm
thanh, hình ảnh khác; quyền liên quan đối với tổ chức phát sóng chỉ được xác định cho tổ
chức khởi xướng và thực hiện việc phát sóng. Theo đó, trong trường hợp: một công ty ghi âm
ký hợp đồng thu âm phần biểu diễn của ca sĩ dưới dạng đĩa CD (compact disc), sau đó một
công ty khác sao chép lại đĩa CD đó và cũng đưa ra thị trường, thì ta sẽ có hai dạng bản ghi
âm cho cùng một phần trình diễn gốc, tuy nhiên, chỉ dạng bản ghi thứ nhất được coi là có
tính nguyên gốc (là sự định hình lần đầu) và là đối tượng bảo hộ của quyền liên quan, dạng
bản ghi thứ hai không đảm bảo tính nguyên gốc và không những không được bảo hộ mà còn
bị coi là hành vi xâm phạm quyền liên quan và tùy theo tính chất, mức độ của sự xâm phạm
mà sẽ bị xử lý theo quy định pháp luật. Tương tự, một tổ chức phát sóng đã đầu tư tài chính
và các điều kiện cần thiết để làm một chương trình truyền hình, hoặc đã mua bản quyền phát
sóng trực tiếp một sự kiện văn hoá nào đó... thì việc phát sóng các chương trình đó của họ
được xác định là thoả mãn điều kiện về tính nguyên gốc của đối tượng quyền liên quan, và
nếu tổ chức phát sóng nào khác tiến hành tiếp sóng, phát sóng đồng thời, hoặc phát lại các
chương trình này thì việc phát sóng của tổ chức thứ hai này là không có tính nguyên gốc và
cũng như trường hợp trên,có thể bị xác định là hành vi xâm phạm quyền liên quan theo pháp
luật sở hữu trí tuệ.
Nếu như đặc trưng thứ nhất của quyền liên quan (tính chất sử dụng các tác phẩm đã có) là
điểm quan trọng phân biệt nó với các loại quyền sở hữu trí tuệ khác, thì tính nguyên gốc của
quyền liên quan cho phép xác định đâu là quyền liên quan được pháp luật bảo hộ, và khi nào
xảy ra sự xâm phạm quyền liên quan. Khi tính nguyên gốc được thoả mãn, ta có thể xác định
được đối tượng được bảo hộ bởi quyền liên quan, chủ thể của quyền liên quan, và ngược lại,
việc không đảm bảo tính nguyên gốc đó là căn cứ để nhận định đã có hành vi xâm phạm đến
quyền liên quan
9. Phân tích điều kiện bảo hộ quyền tác giả: tác phẩm phải được định hình dưới một hình
thức vật chất nhất định.
Có nhiều phương thức khác nhau để xác định căn cứ phát sinh, xác lập quyền sở hữu trí
tuệ. Khác căn cứ phát sinh đối với quyền sở hữu công nghiệp đối với sáng chế; kiểu dáng công

7
nghiệp; thiết kế bố trí; nhãn hiệu, căn cứ là được phát sinh, xác lập dựa trên cơ sở quyết định cấp
văn bằng bảo hộ của cơ quan nhà nước có thẩm quyền thì căn cứ phát sinh, xác lập quyền tác giả
lại được quy định theo Khoản 1 Điều 6 Luật Sở hữu trí tuệ 2005 sửa đổi, bổ sung năm 2009 như
sau:
“1.Quyền tác giả phát sinh kể từ khi tác phẩm được sáng tạo và được thể hiện dưới một hình thức
vật chất nhất định,không phân biệt nội dung,chất lượng,hình thức,phương tiện,ngôn ngữ,đã công
bố hay chưa công bố,đã đăng ký hay chưa đăng ký.”
Các tác phẩm thông thường được sáng tạo và thể hiện dưới một số hình thức vật chất phổ biến
như: nốt trong bản nhạc, bản họa đồ, sơ đồ, bản đồ, bản vẽ, mô hình, sa bàn về công trình kiến
trúc…
Tuy nhiên ta phải lưu ý đến trường hợp đặc biệt quy định tại Khoản 1 Điều 20 Nghị định
100/2006/NĐ-CP như sau:
“1.Tác phẩm văn học, nghệ thuật dân gian quy định tại điểm a, b, c khoản 1 Điều 23 của Luật Sở
hữu trí tuệ được bảo hộ không phụ thuộc vào việc định hình.”
Điểm a,b,c khoản 1 Điều 23 của Luật Sở hữu trí tuệ năm 2005 sửa đổi, bổ sung năm 2009 quy
định bao gồm:
a) Truyện,thơ,câu đố;
b) Điệu hát,làn điệu âm nhạc;
c) Điệu múa,vở diễn,nghi lễ và các trò chơi;
Truyện, thơ, câu đố, điệu hát, làn điệu âm nhạc, điệu múa, vở diễn, nghi lễ và các trò chơi đa
phần là những tác phẩm được lưu giữ nhờ phương thức truyền miệng; đời trước truyền cho đời
sau, không mang tính chính xác cao, không biết có từ khi nào, từ bao giờ và ai là chủ nhân đầu
tiên sáng tác ra tác phẩm đó. Những tác phẩm này chỉ có tính tương đối và ngay bản thân nó
không có một hình thức cụ thể nào.
Vì vậy, không phải tất cả tác phẩm chỉ được bảo hộ khi đã hình thành dưới một hình thức
vật chất nhất định. Vẫn phải có trường hợp tác phẩm KHÔNG được thể hiện dưới một hình thức
vật chất nhất định vẫn được phát sinh quyền tác giả.
10. Phân tích các loại hình tác phẩm được bảo hộ quyền tác giả theo Luật Sở hữu trí tuệ?
Theo điều 14 Luật SHTT quy định về các loại hình tác phẩm được bảo hộ quyền tác giả:
Tác phẩm văn học, nghệ thuật và khoa học được bảo hộ bao gồm:
− Tác phẩm văn học, khoa học, sách giáo khoa, giáo trình và tác phẩm khác được thể hiện dưới
dạng chữ viết hoặc ký tự khác;
Bài ging, bài phát biu và bài nói khác;
Tác phm báo chí;
Tác phm âm nhc;
Tác phm sân khu;
− Tác phẩm điện ảnh và tác phẩm được tạo ra theo phương pháp tương tự (sau đây gọi chung là
tác phẩm điện ảnh);
Tác phm to hình, m thut ng dng;
Tác phm nhip nh;
Tác phm kin trúc;
− Bản họa đồ, sơ đồ, bản đồ, bản vẽ liên quan đến địa hình, kiến trúc, công trình khoa học;
− Tác phẩm văn học, nghệ thuật dân gian;

8
Chng trình máy tính, su tp d liu.
2. Tác phẩm phái sinh chỉ được bảo hộ theo quy định tại khoản 1 Điều này nếu không gây
phương hại đến quyền tác giả đối với tác phẩm được dùng để làm tác phẩm phái sinh.
3. Tác phẩm được bảo hộ quy định tại khoản 1 và khoản 2 Điều này phải do tác giả trực tiếp sáng
tạo bằng lao động trí tuệ của mình mà không sao chép từ tác phẩm của người khác.
11. Phân tích các đối tượng không thuộc phạm vi bảo hộ quyền tác giả theo Luật Sở hữu
trí tuệ?
Các đối tượng không thuộc phạm vi bảo hộ quyền tác giả:
1.Tin tức thời sự thuần tuý đưa tin là các thông tin báo chí ngắn hàng ngày, chỉ mang tính chất
đưa tin không có tính sáng tạo.
2. Văn bản quy phạm pháp luật, văn bản hành chính, văn bản khác thuộc lĩnh vực tư pháp và bản
dịch chính thức của văn bản đó bao gồm văn bản của cơ quan nhà nước, tổ chức chính trị, tổ
chức chính trị - xã hội, tổ chức chính trị - xã hội - nghề nghiệp, tổ chức xã hội, tổ chức xã hội -
nghề nghiệp, tổ chức kinh tế, đơn vị vũ trang nhân dân và các tổ chức khác theo quy định của
pháp luật.
3. Quy trình, hệ thống, phương pháp hoạt động, khái niệm, nguyên lý, số liệu.
Lý do các đối tượng trên không thuộc phạm vi bảo hộ quyền tác giả:
- Đối với tin tức thời sự: Nếu là các tin tức thời sự thuần túy, chỉ tường thuật lại một hành vi , sự
việc.. thì không mang tính sáng tạo. Ví dụ như phóng viên đưa tin về một vụ cướp trên đường
phố thì phải nêu chính xác tên, tuổi người bị cướp, thiệt hại về người và của…. Tuy nhiên, bản
tin thời sự có kèm theo lời phân tích, nhận xét thể hiện sự sáng tạo trí tuệ của tác giả thì vẫn được
bảo hộ.
- Đối với văn bản quy phạm pháp luật, văn bản hành chính: mục đích của loại văn bản này nhằm
tuyên truyền, phổ biến về một vấn đề nào đó trong phạm vi nhỏ từ phường, xã đến phạm vi lớn
như một quốc gia. Mọi người đều có thể sử dụng, trích dẫn các văn bản này nhằm đưa thông tin
mà văn bản chứa đựng áp dụng vào các tình huống cụ trong phạm vi văn bản hướng tới. Nếu
đăng ký bảo hộ văn bản này thì phải xin phép chủ sở hữu, điều đó dẫn đến làm sai lệch mục đích
của văn bản này.
- Đối với quy trình, hệ thống, phương pháp hoạt động,…: các đối tượng này không thể hiện tính
sáng tạo và không thể hiện dưới dạng vật chất nhất định.
12. Phân tích khái niệm tác phẩm phái sinh?
Tác phẩm phái sinh là tác phẩm dịch từ ngôn ngữ này sang ngôn ngữ khác, tác phẩm phóng tác,
cải biên, chuyển thể, biên soạn, chú giải, tuyển chọn.

Như vậy, khi khán giả thưởng thức tác phẩm phái sinh sẽ liên tưởng đến tác phẩm gốc bởi tác
phẩm phái sinh thừa hưởng những nền tảng nhất định của tác phẩm gốc như nội dung, giai điệu,

Một số tác phẩm phái sinh nổi tiếng như: Nhạc kịch Những Người Khốn Khổ được chuyển soạn
từ tiểu thuyết cùng tên của Victor Hugo; album Classics In The Air của Paul Mauriat chơi lại các
trích đoạn cổ điển nổi tiếng theo phong cách hòa tấu hiện đại…

Tác phẩm phái sinh là một trong các dạng tác phẩm thực hiện việc kế thừa vừa nêu và là đối
tượng được bảo hộ quyền tác giả. Khoản 3 điều 2 Công ước Berne về bảo hộ các tác phẩm văn
học và nghệ thuật (sau đây viết tắt là Công ước Berne) đã quy định: “Các tác phẩm dịch, mô
phỏng, chuyển nhạc và các chuyển thể khác từ một tác phẩm văn học hoặc nghệ thuật đều được

9
bảo hộ như các tác phẩm gốc, miễn không phương hại đến quyền tác giả của tác phẩm gốc”.

Luật Sở hữu trí tuệ năm 2005 không đưa ra một định nghĩa rõ ràng về tác phẩm phái sinh mà chỉ
đề cập một cách chung nhất về các loại hình được coi là tác phẩm phái sinh

Đây là các tác phẩm bắt nguồn từ các tác phẩm khác đang tồn tại. Ví dụ về các tác phẩm phái
sinh bao gồm:

– Bản dịch các tác phẩm sang một ngôn ngữ khác;

– Bản phóng tác tác phẩm, ví dụ viết kịch bản một bộ phim dựa theo một cuốn tiểu thuyết;

– Bản chuyển nhạc, ví dụ một bản nhạc dành cho dàn nhạc mà ban đầu được viết cho piano;

– Các hình thức cải biên khác, ví dụ bản tóm tắt một cuốn tiểu thuyết;

– Các tuyển tập các tác phẩm văn học và nghệ thuật, ví dụ các bộ bách khoa toàn thư, hợp tuyển.
Trong trường hợp này, tính nguyên gốc nằm ở sự lựa chọn và cách sắp xếp các nội dung của tác
phẩm.
13. Phân tích điều kiện bảo hộ đối với tác phẩm phái sinh?
Tác phẩm phái sinh được bảo hộ quyền tác giả nếu đáp ứng đủ các điều kiện sau:

Thứ nhất, không gây phương hại đến quyền tác giả đối với tác phẩm

Khoản 2 Điều 14 Luật Sở hữu trí tuệ 2005 sửa đổi 2009 nhấn mạnh, tác phẩm phái sinh chỉ được
bảo hộ quyền tác giả nếu không gây phương hại đến quyền tác giả đối với tác phẩm được dùng
để làm tác phẩm phái sinh.

Tức là, sự ra đời của tác phẩm phái sinh không được gây tổn hại đến quyền lợi của tác giả tác
phẩm gốc, bao gồm cả quyền tài sản và quyền nhân thân.

Thứ hai, được sự cho phép của tác giả, chủ sở hữu quyền tác giả

Tại khoản 7 Điều 28 Luật Sở hữu trí tuệ 2005 sửa đổi bổ sung 2009 quy định:

Làm tác phẩm phái sinh mà không được phép của tác giả, chủ sở hữu quyền tác giả đối với tác
phẩm được dùng để làm tác phẩm phái sinh, trừ trường hợp quy định tại điểm i khoản 1 Điều 25
của Luật này.

Có nghĩa là, ngoài trường hợp chuyển tác phẩm sang chữ nổi hoặc ngôn ngữ khác cho người
khiếm thị, thì bất kể hành động làm tác phẩm phái sinh nào cũng phải được sự cho phép của tác
giả/chủ sở hữu tác phẩm gốc.

Thứ ba, phải mang dấu ấn riêng của tác giả tác phẩm phái sinh

Quyền tác giả phát sinh kể từ khi tác phẩm được sáng tạo và được thể hiện dưới một hình thức
vật chất nhất định, không phân biệt nội dung, chất lượng, hình thức, phương tiện, ngôn ngữ, đã

10
công bố hay chưa công bố, đã đăng ký hay chưa đăng ký.

Theo đó, có thể hiểu, quyền tác giả không bảo hộ nội dung, ý tưởng mà chỉ bảo hộ hình thức,
cách thức thể hiện nội dung, ý tưởng đó.

Tác phẩm phái sinh là một tác phẩm sáng tạo từ tác phẩm gốc nên để được bảo hộ một cách độc
lập, phải thể hiện được sự sáng tạo mới mẻ mang dấu ấn tác giả của nó nhưng đồng thời phải
đảm bảo không xâm phạm tới quyền tài sản của tác giả tạo ra tác phẩm gốc và không trái với
thuần phong mỹ tục.

Do vậy, tác phẩm phái sinh muốn được bảo hộ một cách độc lập thì nó phải thể hiện được sự
sáng tạo nhất định về hình thức thể hiện nội dung, ý nghĩa, giai điệu…dựa trên tác phẩm gốc và
mang dấu ấn mới mẻ của tác giả tác phẩm phái sinh.
14. Phân tích khái niệm tác phẩm thuộc về công chúng?
Theo Điều 43 Luật sở hữu trí tuệ 2019 thì những tác phẩm không còn thời hạn bảo hộ thì thuộc
về công chúng.
Điều 43. Tác phẩm thuộc về công chúng
1. Tác phẩm đã kết thúc thời hạn bảo hộ theo quy định tại Điều 27 của Luật này thì thuộc về
công chúng.
2. Mọi tổ chức, cá nhân đều có quyền sử dụng tác phẩm quy định tại khoản 1 Điều này nhưng
phải tôn trọng các quyền nhân thân của tác giả quy định tại Điều 19 của Luật này.
3. Chính phủ quy định cụ thể việc sử dụng tác phẩm thuộc về công chúng.
Nghị định 22/2018/NĐ-CP hướng dẫn cụ thể hơn cho trường hợp này như sau: Tổ chức, cá nhân
sử dụng tác phẩm thuộc về công chúng phải tôn trọng quyền nhân thân quy định tại các khoản 1,
2 và 4 Điều 19 của Luật SHTT, bao gồm các quyền:

● Đặt tên cho tác phẩm.


● Đứng tên thật hoặc bút danh trên tác phẩm; được nêu tên thật hoặc bút danh khi tác
phẩm được công bố, sử dụng.
● Bảo vệ sự toàn vẹn của tác phẩm, không cho người khác sửa chữa, cắt xén hoặc
xuyên tạc tác phẩm dưới bất kỳ hình thức nào gây phương hại đến danh dự và uy tín
của tác giả.

Như vậy, đối với tác phẩm thuộc về công chúng có thể sử dụng tuy nhiên không được
phép gây phương hại đến những quyền nhân thân gắn liền với tác giả (mà theo quy định
của pháp luật tại Điều 27 Luật SHTT được bảo hộ vô thời hạn).

11
15. Phân tích khái niệm quyền liên quan?
Quy định tại Khoản 3 Điều 4 LSHTT như sau:
Quyền liên quan đến quyền tác giả (sau đây gọi là quyền liên quan) là quyền của tổ chức, cá
nhân đối với cuộc biểu diễn, bản ghi âm, ghi hình, chương trình phát sóng, tín hiệu vệ tinh mang
chương trình được mã hóa.

16. Phân tích điều kiện bảo hộ đối với cuộc biểu diễn?
Theo quy định pháp luật hiện hành thì quyền liên quan đến quyền tác giả (gọi tắt là quyền liên
quan) là quyền của tổ chức, cá nhân đối với cuộc biểu diễn, bản ghi âm, ghi hình, chương trình
phát sóng, tín hiệu vệ tinh mang chương trình được mã hóa.

Quyền liên quan phát sinh kể từ khi cuộc biểu diễn, bản ghi âm, ghi hình, chương trình phát
sóng, tín hiệu vệ tinh mang chương trình được mã hoá được định hình hoặc thực hiện mà không
gây phương hại đến quyền tác giả.

Trong đó, theo quy định tại Khoản 1 Điều 17 Luật Sở hữu trí tuệ 2019 thì cuộc biểu diễn được
bảo hộ nếu thuộc một trong các trường hợp sau đây:

- Cuộc biểu diễn do công dân Việt Nam thực hiện tại Việt Nam hoặc nước ngoài;

- Cuộc biểu diễn do người nước ngoài thực hiện tại Việt Nam;

- Cuộc biểu diễn được định hình trên bản ghi âm, ghi hình được bảo hộ theo quy định tại Điều 30
của Luật Sở hữu trí tuệ 2019;

- Cuộc biểu diễn chưa được định hình trên bản ghi âm, ghi hình mà đã phát sóng được bảo hộ
theo quy định tại Điều 31 của Luật Sở hữu trí tuệ 2019;

- Cuộc biểu diễn được bảo hộ theo điều ước quốc tế mà Cộng hoà xã hội chủ nghĩa Việt Nam là
thành viên.

Tuy nhiên, không phải mọi trường hợp cuộc biểu diễn thuộc các trường hợp trên đây đều được
bảo hộ quyền liên quan. Mà những cuộc biểu diễn thuộc các trường hợp được trích dẫn trên đây
chỉ được bảo hộ quyền liên quan với điều kiện không gây phương hại đến quyền tác giả.

Điều đó đồng nghĩa với việc các cuộc diểu diễn (dù thuộc các trường hợp được trích dẫn trên
đây) sẽ không được bảo hộ quyền liên quan nếu cuộc biểu diễn đó gây phương hại đến quyền tác
giả hoặc việc bảo hộ quyền liên quan đối với các cuộc biểu diễn đó sẽ gây phương hại đến quyền
tác giả đối với tác phẩn đó.

17. Phân tích điều kiện bảo hộ đối với bản ghi âm, ghi hình?
Không phải tất cả các bản ghi âm, ghi hình được tạo ra đều được pháp luật Việt Nam bảo hộ.
Điều kiện để Nhà nước Việt Nam bảo hộ quyền liên quan đối với bản ghi âm, ghi hình là chủ thể
đó phải là tổ chức, cá nhân định hình lần đầu âm thanh, hình ảnh của cuộc biểu diễn hoặc các âm

12
thanh, hình ảnh khác (Điều 16); Nhà sản xuất bản ghi âm, ghi hình phải có quốc tịch Việt Nam
(Điều 17). Những nhà sản xuất bản ghi âm, ghi hình nếu thoả mãn được cả hai điều kiện trên thì
được pháp luật bảo hộ quyền sở hữu trí tuệ đối với bản ghi âm, ghi hình đó.
Tuy nhiên không phải bản ghi âm, ghi hình nào cũng được pháp luật bảo hộ.Nhà sản xuất bản
ghi âm, ghi hình có độc quyền thực hiện hoặc cho phép người khác thực hiện các quyền: Sao
chép trực tiếp hoặc gián tiếp bản ghi âm, ghi hình của mình; Nhập khẩu, phân phối đến công
chúng bản gốc và bản sao bản ghi âm, ghi hình của mình thông qua hình thức bán, cho thuê hoặc
phân phối bằng bất kỳ phương tiện kỹ thuật nào mà công chúng có thể tiếp cận được. Nhà sản
xuất bản ghi âm, ghi hình được hưởng quyền lợi vật chất khi bản ghi âm, ghi hình của mình được
phân phối đến công chúng (Điều 30).
Tổ chức, cá nhân sử dụng trực tiếp hoặc gián tiếp bản ghi âm, ghi hình đã công bố nhằm mục
đích thương mại để phát sóng có tài trợ, quảng cáo hoặc thu tiền dưới bất kỳ hình thức nào
không phải xin phép, nhưng phải trả tiền nhuận bút, thù lao theo thỏa thuận cho tác giả, chủ sở
hữu quyền tác giả, người biểu diễn, nhà sản xuất bản ghi âm, ghi hình, tổ chức phát sóng kể từ
khi sử dụng (Điều 33). Như vậy, đơn vị phát sóng có trách nhiệm thoả thuận và trả thù lao cho
nhà sản xuất bản ghi âm, ghi hình.

Tuy nhiên, bản ghi âm, ghi hình không được bảo hộ vĩnh viễn, quyền của nhà sản xuất bản ghi
âm, ghi hình được bảo hộ năm mươi năm tính từ năm tiếp theo năm công bố hoặc năm mươi năm
kể từ năm tiếp theo năm bản ghi âm, ghi hình được định hình nếu bản ghi âm, ghi hình chưa
được công bố (Điều 34). Đồng thời, hành vi công bố, sản xuất và phân phối cuộc biểu diễn đã
được định hình, bản ghi âm, ghi hình, chương trình phát sóng mà không được phép của người
biểu diễn, nhà sản xuất bản ghi âm, ghi hình, tổ chức phát sóng được coi là xâm phạm tới quyền
liên quan (Điều 35).

18. Phân tích điều kiện bảo hộ đối với chương trình phát sóng, tín hiệu vệ tinh mang
chương trình được mã hóa?
Các chương trình phát sóng, tín hiệu vệ tinh mang chương trình được mã hoá thuộc các trường
hợp cụ thể sẽ được bảo hộ quyền liên quan đến quyền tác giả theo quy định.

Theo đó, theo quy định tại Khoản 3 Điều 17 Luật Sở hữu trí tuệ 2019 thì chương trình phát sóng,
tín hiệu vệ tinh mang chương trình được mã hoá được bảo hộ nếu thuộc một trong các trường
hợp sau đây:

- Chương trình phát sóng, tín hiệu vệ tinh mang chương trình được mã hoá của tổ chức phát sóng

13
có quốc tịch Việt Nam;

- Chương trình phát sóng, tín hiệu vệ tinh mang chương trình được mã hoá của tổ chức phát sóng
được bảo hộ theo điều ước quốc tế mà Cộng hoà xã hội chủ nghĩa Việt Nam là thành viên.

Các chương trình phát sóng, tín hiệu vệ tinh mang chương trình được mã hoá chỉ được bảo hộ
quyền liên quan đến quyền tác giả với điều kiện không gây phương hại đến quyền tác giả.

19. Phân tích các đặc điểm và nội dung của quyền nhân thân thuộc quyền tác giả?
Điều 19.

- là các quyền mang yếu tố tinh thần của chủ thể đối với tác phẩm=> bản chất là các quyền luôn
gắn liền với chủ thể nhất định mà không thể chuyển dịch được.
- có những quyền tuy được xác định là quyền nhân thân nhưng nó vốn dĩ lại là cơ sở để chủ thể
có các quyển đó thực hiện các quyền khác về tài sản=> muốn thực hiện các quyền về tài sản,
người có quyền nhân thân này phải chuyển giao quyền đó cho chủ thể khác.
Quyền nhân thân đối với tác phẩm được phân chia thành hai loại:
+ Quyền nhân thân không thể chuyển dịch
+ quyển nhân thân có thể chuyển dịch.
Theo quy định của pháp luật sở hữu trí tuệ, quyền nhân thân của chủ thể đối với tác phẩm bao
gồm:
a. Quyền đặt tên cho tác phẩm
- nhằm cá biệt hóa tác phẩm mà còn thể hiện dấu ấn cá nhân, cá tính sáng tạo của tác giả.
- cơ sở để người đón nhận tác phẩm có thể hình dung sơ bộ nội dung tư tưởng của tác phẩm ngay
từ khi vừa biết đến tên của tác phẩm. => quyền đặt tên cho tác phẩm là quyển luôn gắn liền với
tác giả và không thể chuyển dịch sang người khác được.
b. Quyền đứng tên tác giả đối với tác phẩm
- được quy định tại điểm khoản 2 Điều 738 Bộ luật dân sự và quy định tại khoản 2 Điều 19 Luật
sở hữu trí tuệ như sau: "Đứng tên thật hoặc bút danh trên tác phẩm, được nêu tên thật hoặc bút
danh khi tác phẩm được công bố, sử dụng". Theo quyền này, tác giả được tùy ý lựa chọn đúng
tên như thế nào đối với tác phẩm, tác giả có thể đứng tên thật của mình với đầy đủ họ, tên, học
hàm học vị, chức vụ, cũng có thể chỉ đứng tên.
- Trong trường hợp vì một lí do nào đó tác giả không muốn đứng tên thật của mình trên tác phẩm
thì tác giả có quyền chỉ để bút danh, bí danh, thậm chí tác giả có quyền không đứng tên đối với
tác phẩm.
- Dù không nêu tên của mình hoặc chỉ đứng bút danh, bí danh trong tác phẩm thì quyền lợi của
tác giả đối với tác phẩm vẫn được bảo vệ, miễn là sau khi tác phẩm được công bố, phổ biến,
sử dụng tác giả chứng minh được tác phẩm đó là do mình sáng tạo ra và có quyền yêu cầu
người sử dụng tác phẩm phải thực hiện các nghĩa vụ đối với các quyển của mình. Đây chính
là quyền được nêu tên thật hoặc bút danh khi tác phẩm được công bố, sử dụng. Quyển đứng tên
tác giả đối với tác phẩm là quyển thân thân luôn gắn liền với tác giả và không thể chuyển dịch.
c. Quyền bảo vệ sự toàn vẹn tác phẩm, không cho người khác sửa chữa, cắt xén hoặc xuyên
tạc tác phẩm dưới bất kỳ hình thức nào gây phương hại đến danh dự và uy tín của tác giả
(khoản 4 điều 19)
=> chỉ có tác giả mới có quyền sửa đổi, bổ sung nội dung tác phẩm của mình. Cũng chỉ có tác
giả mới có quyền cho phép người khác sửa đổi, bổ sung nội dung của tác phẩm.

14
Vì vậy, bất cứ người nào sửa đổi tác phẩm mà không có sự đồng ý của tác giả làm cho chủ đề tư
tưởng; giá trị nghệ thuật, văn hóa, khoa học của tác phẩm bị thay đổi so với đổ của tác giả đều bị
coi là có hành vi xâm phạm quyền tác giả và do đó tác giả có quyển yêu cầu người đó phải
chấm dứt hành vi đó, xin lỗi, hoàn lại sự toàn vẹn của tác phẩm và phải bồi thường thiệt hại
nếu hằng vi đó đã gây ra thiệt hại cho tác giả.
d. Quyền công bố hoặc cho người khác công bố tác phẩm
- theo quy định của pháp luật hiện hành về sở hữu trí tuệ của nước ta thì thuật ngữ công bố tác
phẩm được hiểu diện tương đối hẹp, chỉ bao gồm việc xuất bản các tác phẩm viết, nhân bản và
phát hành các các tác phẩm khác mà vật mang tin của tác phẩm là băng ghi âm, ghi hình; đĩa ghi
âm, ghi hình và các phương tiện kỹ thuật tương tự khác.
- là quyền lợi tinh thần vì mọi hành vi công bố tác phẩm đều không được làm ảnh hưởng đến
danh dự, tiếng tăm của tác giả.
- là cơ sở pháp lý để tác giả bảo vệ danh dự của mình khi người khác xâm hại do công bố tác
phẩm của mình.
20. Phân tích các đặc điểm và nội dung của quyền tài sản thuộc quyền tác giả?
- Điều 20
1. Quyền tài sản bao gồm các quyền sau đây:
a) Làm tác phẩm phái sinh;
- Tác giả được quyền được phép từ tác phẩm của mình tạo ra tác phẩm phái sinh khác
- Hoặc cho phép hoặc không cho phép người khác tạo ra tác phẩm phái sinh như dịch thuật, cải
biên,.....
b) Biểu diễn tác phẩm trước công chúng;
- độc quyền quyền sở hữu của tác giả
- người khác có quyền biểu diễn tác phẩm mà không cần sự đồng ý của chủ sở hữu tác giả nếu
tác phẩm đã được công bố, đã nêu tên tác giả và đã trả thù lao cho tác giả nếu việc biểu diễn
mang mục đích thương mại.
c) Sao chép tác phẩm;
- nếu tác phẩm chưa được công bố thì sao chép tác phẩm là độc quyền thuộc về chủ sở hữu
quyền tác giả
- khi tác phẩm được công bố người khác có quyền sao chép tác phẩm mà không cần xin phép
hoặc trả thù lao trong 2 trường hợp: tự sao chép 1 bản để nghiên cứu khoa học hoặc lưu trữ trong
thư viện.
d) Phân phối, nhập khẩu bản gốc hoặc bản sao tác phẩm;
- Là độc quyền thuộc về chủ sở hữu quyền tác giả trong suốt thời hạn tác phẩm được bảo hộ dù
tác phẩm được công bố hay chưa.
đ) Truyền đạt tác phẩm đến công chúng bằng phương tiện hữu tuyến, vô tuyến, mạng thông
tin điện tử hoặc bất kỳ phương tiện kỹ thuật nào khác;
- là việc truyền tải đến công chúng bằng phương tiện vô tuyến, hữu tuyến, mạng thông tin điện tử
hay bất kì phương tiện nào khác
e) Cho thuê bản gốc hoặc bản sao tác phẩm điện ảnh, chương trình máy tính.

15
- nếu tác phẩm là chương trình máy tính độc lập, nghĩa là chương trình máy tính đó là đối tượng
chủ yếu để cho thuê thì chủ sở hữu quyền tác giả mới có quyền cho thuê bản gốc hoặc bản sao
tác phẩm.

21. Phân biệt quyền sao chép và quyền công bố tác phẩm?

quyền sao chép quyền công bố tác phẩm

Khái niệm Sao chép là việc tạo ra một hoặc là việc phát hành tác phẩm đến công
nhiều bản sao của tác phẩm hoặc chúng với số lượng bản sao đủ để
bản ghi âm, ghi hình bằng bất kỳ đáp ứng nhu cầu hợp lý của công
phương tiện hay hình thức nào, bao chúng tuỳ theo bản chất của tác
gồm cả việc tạo bản sao dưới hình phẩm, do tác giả, chủ sở hữu quyền
thức điện tử. (Quy định tại khoản 10 tác giả thực hiện hoặc do cá nhân,
điều 4) tổ chức khác thực hiện với sự đồng
ý của tác giả, chủ sở hữu quyền tác
giả. (khoản 3 Điều 19)

22. Phân biệt quyền công bố và quyền phân phối tác phẩm?

Quyền công bố Quyền phân phối

Tính chất là quyền nhân thân có thể quyền tài sản


chuyển nhượng được cho
người khác - đối tượng sở
hữu quyền công bố tác
phẩm có thể trao đổi, giao
dịch quyền này.
=> quyền công bố tác
phẩm là quyền nhân thân
duy nhất gắn với tài sản.

23. Phân tích nội dung quyền của người biểu diễn?
Điều 29
– Đối với người biểu diễn đồng thời là chủ đầu tư luôn thì có đầy đủ quyền nhân thân và
quyền tài sản.

– Đối với người biểu diễn không đồng thời là chủ đầu tư thì:

+ Người biểu diễn có các quyền nhân thân đối với cuộc biểu diễn.

+ Chủ đầu tư có các quyền tài sản đối với cuộc biểu diễn.
- Quyền của người biểu diễn được bảo hộ 50 năm tính từ năm tiếp theo năm cuộc biểu
diễn được định hình (khoản 1 điều 34)
- Ví dụ: Ca khúc “Ngày đầu tiên” của nhạc sĩ Khắc Hưng được Đức Phúc hát thì Đức Phúc
16
chính là người biểu diễn.
- Người biểu diễn có thể là ca sĩ, vũ công….
24. Phân tích nội dung quyền của nhà sản xuất bản ghi âm, ghi hình?
- Điều 30
- Nhà sản xuất bản ghi âm, ghi hình là tổ chức, cá nhân định hình lần đầu âm thanh, hình
ảnh của cuộc biểu diễn hoặc các âm thanh, hình ảnh khác. Bản ghi âm, ghi hình được tổ
chức, cá nhân sản xuất bằng chính thời gian, tài chính, cơ sở vật chất- kỹ thuật của mình.
- Khi sử dụng tác phẩm chưa được công bố thì phải xin phép tác giả hoặc chủ sở hữu tác
phẩm thông qua việc giao kết hợp đồng sử dụng tác phẩm.
- Quyền của nhà sản xuất bản ghi âm, ghi hình được bảo hộ năm mươi năm tính từ năm
tiếp theo năm công bố hoặc năm mươi năm kể từ năm tiếp theo năm bản ghi âm, ghi hình
được định hình nếu bản ghi âm, ghi hình chưa được công bố.
25. Phân tích nội dung quyền của tổ chức phát sóng?
- điều 31
- Tổ chức phát sóng là tổ chức khởi xướng và thực hiện việc phát sóng.
- Quyền của tổ chức phát sóng được bảo hộ năm mươi năm tính từ năm tiếp theo năm
chương trình phát sóng được thực hiện.
- không áp dụng đối với tác phẩm điện ảnh.
- Tổ chức, cá nhân sử dụng tác phẩm theo quy định không được làm ảnh hưởng đến việc
khai thác bình thường tác phẩm, không gây phương hại đến các quyền của tác giả, chủ sở
hữu quyền tác giả; phải thông tin về tên tác giả và nguồn gốc, xuất xứ của tác phẩm.

26. Phân tích thời hạn bảo hộ quyền tác giả?

Thời hạn bảo hộ được quy định tại Điều 27 Luật Sở hữu trí tuệ . Theo đó, có thể xác định về thời
hạn bảo hộ như sau:

Bảo hộ vô thời hạn : Bảo hộ suốt cuộc đời của tác giả và vĩnh viễn sau khi tác giả chết

- các quyền nhân thân gắn liền với tác giả không thể chuyển dịch, bao gồm: Quyền đặt tên cho
tác phẩm, quyền đứng tên thật hoặc bút danh trên tác phẩm….

Bảo hộ có thời hạn : bảo hộ trong một khoảng thời gian nhất định

Các quyền đối với tác phẩm được pháp luật bảo hộ có thời hạn bao gồm quyền nhân thân có thể
chuyển dịch (quyền công bố hoặc cho người khác công bố tác phẩm) và các quyền tài sản. Theo
quy định tại Điều 27 Luật sở hữu trí tuệ có hai cách tính thời hạn bảo hộ quyền tác giả:

+ Đối với tác phẩm điện ảnh, nhiếp ảnh, mĩ thuật ứng dụng chưa được công bố trong thời hạn 25
năm, kể từ khi tác phẩm được định hình thì thời hạn bảo hộ là 100 năm, kể từ khi tác phẩm được
định hình. Việc xác định thời hạn bảo hộ đối với tác phẩm điện ảnh, nhiếp ảnh, mĩ thuật ứng
dụng trong trường hợp này được hiểu là nếu tác phẩm đó đã được công bố trong vòng 25 năm kể
từ khi tác phẩm được định hình trên một hình thái vật chất thì thời hạn bảo hộ là 75 năm; nếu
càng công bố muộn thì thời hạn bảo hộ sẽ ít đi.

Ví dụ: Tác phẩm đã được định hình nhưng đến năm thứ 40 tác giả mới công bố thì thời hạn bảo
hộ chỉ còn sáu mươi năm kể từ khi công bố.
17
+ Trong thời hạn năm mươi năm, kể từ khi tác phẩm điện ảnh, tác phẩm sân khấu được định
hình, nếu tác phẩm chưa được công bố thì thời hạn bảo hộ được tính từ khi tác phẩm đó được
định hình. Việc xác định thời hạn bảo hộ đối với tác phẩm điện ảnh, tác phẩm sân khấu trong
trường hợp này được hiểu là nếu tác phẩm điện ảnh, tác phẩm sân khấu đã được định hình theo
hình thái vật chất nhất định nhưng chưa được công bố thì thời điểm bắt đầu để xác định thời hạn
bảo hộ (50 năm ) là thời điểm tác phẩm đó được định hình. Nếu hết 50 năm đó tác phẩm mới
được công bố thì sẽ không được bảo hộ nữa.

+ Tác phẩm khuyết danh là tác phẩm mà tác giả không đứng tên trên tác phẩm đó hoặc hoặc chỉ
đề ký hiệu trên tác phẩm những kí hiệu đó không đủ cơ sở để xác định chính xác về tác giả của
tác phẩm. Trong thời hạn trên, người được hưởng quyền đối với tác phẩm khuyết danh là Nhà
nước, trong trường hợp tác phẩm khuyết danh cho các tổ chức, cá nhân đang quản lý thì tổ chức,
cá nhân đó được hưởng quyền đối với tác phẩm.

Khi các thông tin về tác giả xuất hiện trong thời hạn 50 năm kể từ khi tác phẩm khuyết danh
được công bố lần đầu tiên đủ để xác định danh tính của tác giả, chủ sở hữu tác phẩm khuyết danh
thì các quyền đối với tác phẩm đó thuộc về họ và sẽ được bảo hộ kể từ ngày danh tính của họ
được xác định cho đến năm mươi năm sau khi họ chết (nếu tác phẩm khuyết danh đó không
thuộc tác phẩm điện ảnh, sân khấu, mĩ thuật ứng dụng).

– Đối với tác phẩm di cảo thì thời hạn bảo hộ là 50 năm kể từ khi tác phẩm đó được công bố lần
đầu tiên. “di cảo” được hiểu là bản thảo tác phẩm của người chết để lại. Theo đó, có thể hiểu tác
phẩm di cảo là tác phẩm chỉ được phát hiện sau khi tác giả của tác phẩm đã chết. Vì vậy, khi tác
giả còn sống thì tác phẩm đã được định hình theo hình thái vật chất nhất định nhưng chưa ai biết
về tác phẩm đó nên thời hạn bảo hộ đối với tác phẩm này không được xác định theo thời điểm
tác phẩm được định hình.

– Đối với tác phẩm thuộc các loại hình khác thì thời hạn bảo hộ là suốt cuộc đời tác giả và 50
năm tiếp theo năm tác giả chết. Trong đó, đối với tác phẩm do một tác giả sáng tạo thì thời điểm
bắt đầu để tính năm mươi năm tiếp theo năm tác giả chết là thời điểm tác giả đó chết hoặc được
coi là đã chết. Đối với tác phẩm do các đồng tác giả cùng sáng tạo thì thời điểm bắt đầu để tính
năm mươi năm tiếp theo năm tác giả chết là thời điểm tác giả cuối cùng chết hoặc được coi là đã
chết.

Việc pháp luật quy định về bảo hộ quyền tác giả năm mươi năm sau khi tác giả chết nhằm xác
định quyền để lại thừa kế của tác giả đối với tác phẩm cho những người thừa kế của họ. Vì vậy,
khi tác giả chết quyền tác giả cũng là loại di sản thừa kế và được chuyển dịch theo quy định của
pháp luật thừa kế.

Lưu ý, Thời hạn bảo hộ theo quy định nêu trên chấm dứt vào thời điểm 24 giờ ngày 31 tháng 12
của năm chấm dứt thời hạn bảo hộ quyền tác giả.
27. Phân tích thời hạn bảo hộ quyền liên quan? ( Điều 34 luật SHTT )

Thời hạn bảo hộ quyền liên quan có thời hạn bảo hộ chung là 50 năm , không có sự phân biệt đó
là quyền nhân thân hay tài sản ( chủ yếu thiên về bảo hộ các quyền tài sản của người biểu diễn;
nhà sx bản ghi âm ,ghi hình ; tổ chức phát sóng ) .

Lưu ý: Thời hạn bảo hộ chấm dứt vào thời điểm 24 giờ ngày 31/12 của năm chấm dứt thời hạn
18
bảo hộ các quyền liên quan.

Như vậy, quyền liên quan cũng được bảo hộ mà không cần đăng ký. Tuy nhiên, đối tượng quyền
liên quan đến tác giả không phải là các tác phẩm văn học nghệ thuật giống như quyền tác giả.

28. Phân tích các trường hợp giới hạn quyền tác giả?
Giới hạn quyền tác giả là những hạn chế về quyền của tác giả, chủ sở hữu quyền tác giả hay còn
được hiểu như là các ngoại lệ của quyền tác giả.
Giới hạn quyền tác giả là quy định liên quan tới việc khai thác, sử dụng tác phẩm, cuộc biểu
diễn, bảng ghi âm hình, chương trình sóng các trường hợp đặc biệt, cụ thể nhằm đáp ứng các nhu
cầu chính đáng phục vụ công tác thông tin, nghiên cứu khoa học, giảng dạy và chính sách xã hội.
Các trường hợp giới hạn quyền tác giả
- Giới hạn quyền tác giả về không gian: Quyền tác giả chỉ được bảo hộ trong phạm vi một
quốc gia, có thể nói việc bảo hộ quyền tác giả có tính chất lãnh thổ, tác phẩm chỉ được
hưởng sự bảo hộ quyền tác giả nếu đáp ứng được các tiêu chuẩn pháp lý được quy định
bởi pháp luật về quyền tác giả của một quốc gia nhất định\
- Giới hạn quyền tác giả về thời gian: Chỉ có thời gian nhất định chứ không có thời gian
vĩnh viễn, với mỗi tác phẩm khác nhau thì sẽ có những quy định thời hạn khác nhau
- Giới hạn quyền tác giả trong trường hợp sử dụng tác phẩm đã công bố không phải xin
phép, không phải trả tiền nhuận bút, thù lao: Theo công ước Berne điều ước quốc tế da
phương đầu tiên về quyền tác giả cũng quy định trong những trường hợp nhất định quốc
gia thành viên có quyền quy định việc sao chép, trích dẫn, in lại…với mức độ phù hợp
không nhằm mục đích kinh doanh, không xâm phạm tới việc khai thác bình thường tác
phẩm, không gây thiệt hại cho những quyền lợi hợp pháp của tác giả thì việc sử dụng đó
không phải xin phép.
- Giới hạn quyền tác giả trong trường hợp sử dụng tác phẩm đã công bố không phải xin
phép nhưng phải trả tiền nhuận bút, thù lao: theo điều 11 công ước Berne cho phép pháp
luật quốc gia thành viên có quyền quy định việc sử dụng tác phẩm đã công bố để phát
sóng thì không phải xin phép nhưng phải trả tiền bản quyền. Và việc sử dụng tác phẩm
trong trường hợp không phải xin phép nhưng phải trả tiền nhuận bút, thù lao không áp
dụng đối với tác phẩm điện ảnh.
29. Phân tích các trường hợp giới hạn quyền liên quan?
Trường hợp sử dụng quyền liên quan không cần phải xin phép , không phải trả
nhuận bút ,thù lao.
Nhằm tăng khả năng tiếp cận của công chúng đối với các đối tượng của quyền liên quan ,
tạo đkien cho họ trong việc sử dụng vào mục đích phi thương mại và phải đáp ứng các
điều kiện chung : sử dụng với mục đích phi thương mại , sử dụng trong một phạm vi hợp
lý , không gây tổn hại đến quyền , lợi ích hợp pháp của chủ sở hữu . Với các trường hợp
cụ thể :( quy định tại điều 32 LSHTT )
+ Tự sao chép một bản với mục đích nghiên cứu , giảng dạy (VD: Giảng viên sao chép
một phần trương trình phát sóng trên truyền hình để làm tư liệu giảng dạy )
+ Trích dẫn hợp lý để cung cấp thông tin ( ví dụ : trong chương trình “ tin tức văn hóa “
đài truyền hình VN phát sóng một số hình ảnh ( trích dẫn ) từ các chương trình nghệ thuật
đã diễn ra ở Vn hay trên TG để đưa tin )
+ Tổ chức phát sóng tự làm bản sao tạm thời để phát sóng khi được hưởng quyền phát
sóng ( ví dụ : để chuẩn bị cho buổi phát sóng của mình , đài truyền hình làm các bản sao
tạm thời trên phương tiện của họ để thực hiện việc phát sóng )
19
Trường hợp sử dụng quyền liên quan không phải xin phép nhưng phải trả tiền nhuận
bút , thù lao ( điều 33 LSHTT )
+ Tổ chức cá nhân sử dụng trực tiếp hoặc gián tiếp bản ghi âm , ghi hình đã được công
bố nhằm mục đích thương mại để phát sóng có tài trợ hay không tài trợ , quảng cáo hoặc
thu tiền hay không thu tiền dưới bất kỳ hình thức nào không phải xin phép nhưng phải
trả tiền nhuận bút ,thù lao ( được hiểu là tổ chức , cá nhân khi sử dụng bản ghi âm , ghi
hình để thực hiện chương trình phát sóng, quảng cáo hoặc thu tiền hoặc không thu tiền
dưới bất kỳ hình thức nào đều phải trả tiền nhuận bút thù lao cho chủ sở hữu bản ghi âm ,
ghi hình)
+ Tổ chức , cá nhân sử dụng bản ghi âm , ghi hình đã công bố trong hoạt động kinh
doanh , thương mại không phải xin phép nhưng phải trả tiền nhuận bút , thù lao
30. Phân tích khái niệm tác giả, đồng tác giả?
Tác giả: Là người trực tiếp sáng tạo ra một phần hoặc toàn bộ tác phẩm văn học, nghệ thuật và
khoa học.

Đồng tác giả là những tác giả cùng trực tiếp sáng tạo ra một phần hoặc toàn bộ tác phẩm văn
học, nghệ thuật và khoa học”

Điều 8 Nghị định số 100/2006/NĐ-CP đã chỉ rõ:

– Tác giả là người trực tiếp sáng tạo ra một phần hoặc toàn bộ tác phẩm văn học, nghệ thuật và
khoa học bao gồm:

a) Cá nhân Việt Nam có tác phẩm được bảo hộ quyền tác giả:

b) Cá nhân nước ngoài có tác phẩm được sáng tạo và thể hiện dưới hình thức vật chất nhất định
tại Việt Nam;

c) Cá nhân nước ngoài có tác phẩm được công bố lần đầu tiên tại Việt Nam;

d) Cá nhân nước ngoài có tác phẩm được bảo hộ tại Việt Nam theo điều ước quốc tế về quyền
tác giả mà Việt Nam là thành viên.

– Tổ chức, cá nhân làm công việc hỗ trợ, góp ý kiến hoặc cung cấp tư liệu cho người khác sáng
tạo ra tác phẩm không được công nhận là tác giả.” Hệ quả pháp lý của quy định này được thể
hiện: Quyền đặt tên cho tác phẩm, quyền đứng tên đối với tác phẩm, quyền bảo vệ sự toàn vẹn
của tác phẩm là không thể chuyển giao và tồn tại vĩnh viễn.

Tác giả chỉ có thể từ chối tư cách là chủ sở hữu tác phẩm mà không thể hủy bỏ tác phẩm khi tác
phẩm đã được công bố, nói một cách chính xác: Tác giả không thể hủy bỏ quyền nhân thân
(không thể chuyển giao) của mình đối với tác phẩm khi nó đã được công bố, ngay cả trong
trường hợp bản gốc của tác phẩm (bản định hình lần đầu của tác phẩm) không còn tồn tại.

Pháp luật Việt Nam về sở hữu trí tuệ cũng không định nghĩa thuật ngữ “đồng tác giả” mà mặc
nhiên quan niệm trong trường hợp có từ hai tác giả trở lên cùng sáng tạo nên một tác phẩm thì họ
là các đồng tác giả của tác phẩm đó. Quan niệm này chỉ điều chỉnh được mối quan hệ về quyền
tài sản đối với tác phẩm giữa các đồng tác giả đối với trường hợp:

20
– Tác phẩm được coi là đồng sở hữu chung duy nhất;

– Tác phẩm được coi là đồng sở hữu chung theo phần, trường hợp này được điều chỉnh bởi
Khoản 2 Điều 38 Luật Sở hữu trí tuệ: các đồng tác giả sáng tạo ra tác phẩm, nếu có phần riêng
biệt có thể tách ra sử dụng độc lập mà không làm phương hại đến phần của các đồng tác giả khác
thì có các quyền nhân thân và quyền tài sản đối với phần riêng biệt đó. Quy định như trên là
không phổ quát, bởi lẽ nó không thể điều chỉnh được quyền nhân thân đối với tác phẩm mà các
ví dụ sau đây là minh chứng:

– Một bài thơ được công bố, sau đó một nhạc sĩ phổ nhạc cho bài thơ thành bài hát, giả định rằng
tác giả bài thơ chỉ biết đến bài hát khi nó được công bố. Nếu coi bài hát (bao gồm phần nhạc và
phần lời) là một tác phẩm đồng tác giả thì pháp luật không thể điều chỉnh được khi xảy ra tranh
chấp về quyền nhân thân giữa các đồng tác giả, bởi lẽ ngoài việc mỗi đồng tác giả có các quyền
nhân thân đối với phần riêng biệt của mình thì họ còn có quyền nhân thân chung đối với toàn bộ
tác phẩm đồng tác giả:

– Tác giả của một bản nhạc không lời đã chết, một người viết thêm lời vào bản nhạc thành bài
hát, nếu quan niệm như trên thì phải coi bài hát là một tác phẩm đồng tác giả vì đã có hai tác giả
cùng sáng tạo nên một tác phẩm.

Tham khảo quy định về tác phẩm đồng tác giả trong Luật Quyền tác giả của Hoa Kỳ: “Tác phẩm
đồng tác giả là tác phẩm được sáng tạo bởi hai hoặc nhiều tác giả với chủ ý là sự đóng góp của
họ được kết hợp thành các phần không thể tách rời và phụ thuộc lẫn nhau trong một tổng thể
hoàn chỉnh”, trong đó nhất thiết các đồng tác giả phải chủ ý cùng sáng tạo nên một tác phẩm
chung.

31. Phân tích khái niệm chủ sở hữu quyền tác giả?
Theo điều 36 Luật sở hữu trí tuệ Chủ sở hữu quyền tác giả là tổ chức, cá nhân nắm giữ một, một
số hoặc toàn bộ các quyền tài sản
Chủ sở hữu quyền tác giả là:
- là tác giả khi họ sử dụng thời gian, tài chính, cơ sở vật chất - kỹ thuật của mình để sáng
tạo ra tác phẩm. Lúc này chủ sở hữu quyền tác giả có toàn bộ quyền nhân thân và quyền
tài sản.
- là các đồng tác giả nếu sử dụng thời gian, tài chính, cơ sở vật chất - kỹ thuật của mình để
cùng sáng tạo ra tác phẩm.
- Tác giả là người trực tiếp sáng tạo ra một phần hoặc toàn bộ tác phẩm văn học, nghệ
thuật và khoa học.
- Đồng tác giả là những tác giả cùng trực tiếp sáng tạo ra một phần hoặc toàn bộ tác phẩm
văn học, nghệ thuật và khoa học.
- Là Tổ chức giao nhiệm vụ sáng tạo tác phẩm cho tác giả là người thuộc tổ chức mình
hoặc giao kết hợp đồng với tác giả sáng tạo ra tác phẩm là chủ sở hữu các quyền tài sản
và quyền công bố tác phẩm/ cho phép người khác công bố tác phẩm
- Là Tổ chức, cá nhân được thừa kế quyền tác giả theo quy định của pháp luật về thừa kế là
chủ sở hữu các quyền tài sản và quyền công bố tác phẩm/ cho phép người khác công bố
tác phẩm.
- Là Tổ chức, cá nhân được chuyển giao một, một số hoặc toàn bộ các quyền tài sản và
quyền công bố tác phẩm/ cho phép người khác công bố tác phẩm theo thoả thuận trong

21
hợp đồng là chủ sở hữu quyền tác giả
- Nhà nước là chủ sở hữu quyền tác giả đối với các tác phẩm sau đây: Tác phẩm khuyết
danh, trừ trường hợp có tổ chức, cá nhân đang quản lý tác phẩm khuyết danh; Tác phẩm
còn trong thời hạn bảo hộ mà chủ sở hữu quyền tác giả chết không có người thừa kế,
người thừa kế từ chối nhận di sản hoặc không được quyền hưởng di sản; Tác phẩm được
chủ sở hữu quyền tác giả chuyển giao quyền sở hữu cho Nhà nước.
32. Phân tích cơ chế xác lập quyền tác giả, quyền liên quan?
- Quyền tác giả: được bảo hộ theo cơ chế tự động có nghĩa là quyền tác giả phát sinh kể từ
khi tác phẩm được sáng tạo và được thể hiện dưới một hình thức vật chất nhất định, mà
không phân biệt nội dung, chất lượng, hình thức…, không phụ thuộc vào việc tác giả đã
công bố hay chưa công bố, đã đăng ký hay chưa đăng ký.. Như vậy cơ chế xác lập quyền
tác giả là sự thể hiện tác phẩm.
Thể hiện nếu tác phẩm được định hình dưới hình thức vật chất nhất định ví dụ: viết ra giấy, được
ghi băng, ghi đĩa…tức là bằng một cách nào đó mà các giác quan của con người có thể nhận biết
được, đây chính là bằng chứng cho sự tồn tại của tác phẩm, qua đó người tạo ra tác phẩm sẽ có
quyền đối với các tác phẩm của mình
Mặc dù quyền tác giả được phát sinh ngay sau khi tác phẩm được thể hiện dưới hình thức vật
chất nhất định nhưng thực tế trong nhiều trường hợp để xác định được chính xác thời điểm tác
phẩm được thể hiện như vậy cũng không phải đơn giản, vì vậy pháp luật quy định cho phép các
chủ thể quyền được gia tăng bảo hộ quyền của mình bằng cách đăng ký tại cơ quan nhà nước có
thẩm quyền.
+ Đăng ký QTG là việc tác giả, chủ sở hữu QTG nộp đơn và hồ sơ kèm theo cho cơ quan
nhà nước có thẩm quyền để ghi nhận các thông tin về tác giả, tác phẩm, chủ sở hữu
QTG .
- Quyền liên quan: phát sinh từ khi cuộc biểu diễn, bản ghi âm, ghi hình, chương trình phát
sóng, tín hiệu vệ tinh mang chương trình được mã hoá được định hình hoặc thực hiện mà
không cản trở và gây phương hại đến việc tác giả thực hiện quyền tác giả. Cơ chế xác lập
quyền liên quan chính là sự định hình hoặc thực hiện các buổi biểu diễn, bản ghi âm, buổi
phát sóng, đó chính là minh chứng cho việc tồn tại của các đối tượng của QLQ từ đó làm
phát sinh quyền của các chủ thể quyền
Để đảm bảo một cách chắc chắn quyền của các chủ thể quyền, ngăn chặn hiệu quả các hành vi vi
phạm cũng như tạo thuận lợi cho công tác quản lý, pháp luật có các quy định rõ ràng về vấn đề
đăng ký QLQ
33. Phân tích khái niệm quyền sở hữu công nghiệp?
Quyền sở hữu công nghiệp là quyền đối với các thành quả lao động sáng tạo trí tuệ của cong
người trong lĩnh vực công nghiệp, kinh doanh và thương mại.
Là một bộ phận cấu thành quan trọng của SHTT, quyền SHCN cũng mang đầy đủ các đặc điểm
của quyền SHTT
- Đối tượng phải là sản phẩm sáng tạo trí tuệ của con người, những sự vật, hiện tượng vốn
có thuộc về thế giới tự nhiên không phải do con người tạo ra bằng trí tuệ của mình thì
không được bảo hộ
- Quyền SHCN trao cho chủ sở hữu độc quyền trong việc khai thác sử dụng đối tượng
SHCN và quyền ngăn cấm người khác sử dụng đối tượng SHCN đó
- Quyền SHCN mang tính lãnh thổ
- Quyền SHCN là quyền được bảo hộ có thời hạn, mỗi quốc gia đều có quy định cụ thể về

22
thời hạn được hưởng độc quyền đối với từng đối tượng quyền SHCn
- Quyền SHCN là quyền đối với tài sản trí tuệ, do vậy có thể thực hiện các giao dịch đối
với các quyền SHCN
34. So sánh quyền tác giả và quyền sở hữu công nghiệp?
Về đối tượng bảo hộ
- Đối tượng quyền tác giả bao gồm tác phẩm văn học, nghệ thuật, khoa học; đối tượng
quyền liên quan đến quyền tác giả bao gồm cuộc biểu diễn, bản ghi âm, ghi hình, chương
trình phát sóng, tín hiệu vệ tinh mang chương trình được mã hoá. Đối tượng của quyền
tác giả chủ yếu được áp dụng trong các hoạt động giải trí tinh thần.
- Còn các đối tượng của quyền sở hữu công nghiệp lại được ứng dụng trong các hoạt động
sản xuất, kinh doanh thương mại. Bao gồm sáng chế, kiểu dáng công nghiệp, thiết kế bố
trí mạch tích hợp bán dẫn, bí mật kinh doanh, nhãn hiệu, tên thương mại và chỉ dẫn địa
lý.
Về đối tượng không được bảo hộ
- Các đối tượng không thuộc phạm vi bảo hộ quyền tác giả quy định tại Điều 15 Luật sở
hữu trí tuệ bao gồm: tin tức thời sự thuần túy đưa tin, văn bản quy phạm pháp luật, văn
bản hành chính, văn bản khác thuộc lĩnh vực tư pháp và bản dịch chính thức của văn bản
đó.
- Các đối tượng không thuộc phạm vi bảo hộ của quyền sở hữu công nghiệp, được chia
thành đối với sáng chế, kiểu dáng công nghiệp. thiết kế bố trí, nhãn hiệu, tên thương mại,
chỉ dẫn địa lý và bí mật kinh doanh.
Về điều kiện bảo hộ:
- Quyền tác giả phát sinh kể từ khi tác phẩm được sáng tạo và được thể hiện dưới một hình
thức vật chất nhất định, không phân biệt nội dung, chất lượng, hình thức, phương tiện,
ngôn ngữ, đã công bố hay chưa công bố, đã đăng ký hay chưa đăng ký (khoản 1 Điều 6
Luật sở hữu trí tuệ 2005, sửa đổi bổ sung 2009). Pháp luật về quyền tác giả không quy
định về nội dung đối với tác phẩm được bảo hộ.
- Đối với quyền sở hữu công nghiệp: đối tượng quyền sở hữu công nghiệp được bảo hộ khi
đáp ứng được các điều kiện bảo hộ mà Luật sở hữu trí tuệ quy định. Tức là, pháp luật về
quyền sở hữu công nghiệp bảo hộ nội dung của đối tượng.
+ Sáng chế: có tính mới, tính sáng tạo, khả năng áp dụng công nghiệp.
+ Kiểu dáng công nghiệp: có tính mới, tính sáng tạo, khả năng áp dụng công
nghiệp.
+ Nhãn hiệu: dấu hiệu nhìn thấy được, có tính phân biệt (Điều 72 Luật sở hữu trí
tuệ).
+ Tên thương mại: Có khả năng phân biệt giữa các chủ thể kinh doanh (Điều 76
Luật sở hữu trí tuệ).
+ Chỉ dẫn địa lý: Sản phẩm mang chỉ dẫn địa lý có nguồn gốc địa lý từ khu vực, địa
phương, vùng lãnh thổ hoặc nước tương ứng với chỉ dẫn địa lý; có danh tiếng,
chất lượng hoặc đặc tính chủ yếu do điều kiện địa lý ở nơi đó mang lại (Điều 79
Luật sở hữu trí tuệ).
+ Mạch tích hợp bán dẫn: có tính nguyên gốc, tính thương mại (Điều 68 Luật sở
hữu trí tuệ)
+ Bí mật kinh doanh: Không phải là hiểu biết thông thường và không dễ dàng có
được; khi sử dụng trong kinh doanh sẽ tạo cho người sở hữu nó có lợi thế; được
chủ sở hữu bảo mật bằng các biện pháp cần thiết (Điều 84 Luật sở hữu trí tuệ).

23
Căn cứ xác lập quyền
- Quyền tác giả được xác lập kể từ khi tác phẩm được sáng tạo và định hình dưới một hình
thức vật chất nhất định; dựa vào chính hành vi tạo ra tác phẩm của tác giả, không phụ
thuộc vào thể thức, thủ tục nào.
- Đối với quyền sở hữu công nghiệp: được xác lập dựa trên quyết định của cơ quan nhà
nước có thẩm quyền thông qua việc xét và cấp văn bằng bảo hộ cho chủ sở hữu các đối
tượng đó (trừ các đối tượng sở hữu công nghiệp được xác lập một cách tự động)
Về thời điểm phát sinh
- Quyền tác giả phát sinh kể từ khi tác phẩm được sáng tạo và được thể hiện dưới một hình
thức vật chất nhất định, không phân biệt nội dung, chất lượng, hình thức, phương tiện,
ngôn ngữ, đã công bố hay chưa công bố, đã đăng ký hay chưa đăng ký. Quyền tác giả
phát sinh một cách mặc nhiên và nó được thiết lập từ thời điểm tác phẩm đó được thể
hiện dưới hình thức khách quan mà người khác có thể nhận biết được.
- Quyền sở hữu công nghiệp phát sinh tại từng thời điểm khác nhau tùy thuộc vào đối
tượng được bảo hộ
Về yêu cầu văn bằng bảo hộ
- Quyền tác giả không cần phải có văn bằng bảo hộ. Văn bằng bảo hộ của quyền tác giả -
quyền liên quan là giấy chứng nhận đăng ký quyền tác giả, giấy chứng nhận đăng ký
quyền liên quan (được cấp bởi Cục bản quyền tác giả thuộc Bộ văn hóa thể thao và du
lịch). Quyền tác giả được bảo hộ một cách tự động, bảo hộ không phụ thuộc vào thủ tục
đăng ký.
- Với quyền sở hữu công nghiệp, việc đăng kí quyền sở hữu công nghiệp là thủ tục bắt
buộc. Quyền sở hữu công nghiệp chỉ được pháp luật bảo hộ khi chúng đã được cơ quan
nhà nước chính thức cấp văn bằng bảo hộ. Văn bằng bảo hộ của quyền sở hữu công
nghiệp đối với từng đối tượng như sau:
Đối với sáng chế: bằng độc quyền sáng chế
Đối với kiểu dáng công nghiệp: bằng độc quyền kiểu dáng công nghiệp
Đối với chỉ dẫn địa lý, nhãn hiệu, thiết kế bố trí mạch tích hợp bán dẫn: giấy chứng nhận đăng ký
chỉ dẫn địa lý, nhãn hiệu, thiết kế bố trí mạch tích hợp bán dẫn.
(Được cấp bởi Cục Sở hữu trí tuệ thuộc Bộ khoa học và công nghệ)
Về thời hạn bảo hộ
- Pháp luật về quyền tác giả chỉ bảo hộ hình thức chứa đựng tác phẩm khi nó được tạo ra
và thể hiện dưới hình thức nhất định mà không bảo hộ nội dung tác phẩm.
- Trong khi đó quyền sở hữu công nghiệp bảo hộ nội dung của đối tượng. Đối tượng sở
hữu công nghiệp phải đáp ứng được các điều kiện nhất định
Về nội dung
- Quyền tác giả được bảo hộ về quyền nhân thân, quyền tài sản.
- Nội dung quyền sở hữu công nghiệp bao gồm tổng hợp các quyền của chủ thể sở hữu
công nghiệp được pháp luật ghi nhận và bảo hộ. Quyền sở hữu công nghiệp được bảo hộ
về quyền tài sản, trường hợp sáng chế, kiểu dáng công nghiệp, thiết kế bố trí được bảo hộ
cả quyền của tác giả.
Về giới hạn bảo hộ
- Giới hạn bảo hộ của quyền tác giả thể hiện ở các quy định: Các trường hợp sử dụng tác
phẩm đã công bố (Điều 25 và 26) và Các trường hợp sử dụng quyền liên quan không phải
xin phép (Điều 32 và 33). Theo quy định của hai điều luật này, ngoại lệ chỉ dành cho một
số trường hợp sử dụng tác phẩm, sử dụng quyền liên quan đáp ứng được ba điều kiện sau:

24
Việc sử dụng hoàn toàn vào mục đích phi thương mại
Việc sử dụng không làm ảnh hưởng đến việc khai thác bình thường tác phẩm, cuộc biểu diễn,
bản ghi, chương trình phát sóng, không gây phương hại đến quyền tác giả và quyền liên quan;
Khi sử dụng phải tôn trọng các quyền của tác giả, chủ thể của quyền liên quan (như: thông tin về
tác giả, tác phẩm, người biểu diễn,… )
- Giới hạn quyền sở hữu công nghiệp quy định tại Mục 2 Chương IX Luật sở hữu trí tuệ.
Thứ nhất, chủ sở hữu đối tượng sở hữu công nghiệp (vẫn thực hiện các quyền của mình
nhưng lị không được hoàn toàn tự do ý chí, họ phải thực hiện quyền đó theo mệnh lệnh
bắt buộc của cơ quan nhà nước có thẩm quyền. Thứ hai, các chủ thể trong những trường
hợp nhất định pháp luật cho phép được tự ý sử dụng các đối tượng sở hữu công nghiệp
thuộc quyền của người khác mà không cần phải xin phép hay trả thù lao.
Về hành vi xâm phạm
- Hành vi xâm phạm quyền tác giả được bảo hộ quy định tại Điều 28 Luật sở hữu trí tuệ.
- Hành vi xâm phạm quyền sở hữu công nghiệp được bảo hộ, tùy từng đối tượng được bảo
hộ mà nó có thể là hành vi xâm phạm quyền đối với sáng chế, kiểu dáng công nghiệp,
thiết kế bố trí (Điều 126), hay hành vi xâm phạm quyền đối với nhãn hiệu, tên thương
mại, và chỉ dẫn địa lý (Điều 129) hay hành vi cạnh tranh không lành mạnh (Điều 130).

35. Phân tích khái niệm sáng chế


Sáng chế là một độc quyền được cấp cho một giải pháp là sản phẩm hoặc quy trình nói chung,
cung cấp cách thức mới để thực hiện một điều gì đó hoặc đưa ra một giải pháp công nghệ mới
cho một vấn đề
Sáng chế là sản phẩm được thể hiện dưới các dạng sau đây:
- Dạng vật thể: được thể hiện bằng một tập hợp các thông tin xác định một sản phẩm nhân
tạo được đặc trưng bởi các dấu hiệu, chi tiết về kết cấu
- Dạng chất thể: được thể hiện bằng một tập hợp các thông tin xác định một sản phẩm nhân
tạo được đặc trưng bởi các dấu hiệu, đặc điểm về sự hiện diện, tỉ lệ và trạng thái các phân
tử tạo thành và có một chức năng nhất định: vật liệu, chất liệu, thực phẩm….
- Dạng vật liệu sinh học: được thể hiện bằng tập hợp các thông tin về một sản phẩm có
chứa thông tin di truyền bị biến đổi dưới tác động của cong người, có khả năng tự tái tạo
như gen thực vật/động vật….
36. Phân tích điều kiện bảo hộ sáng chế: tính mới.
Điều 60 Luật Sở hữu trí tuệ 2005 được sửa đổi, bổ sung tại khoản 2 Điều 2 Luật Kinh doanh bảo
hiểm, Luật Sở hữu trí tuệ 2019 quy định sáng chế có tính mới nếu:

- Chưa bị bộc lộ công khai dưới hình thức sử dụng, mô tả bằng văn bản hoặc bất kỳ hình thức
nào khác ở trong nước hoặc ở nước ngoài trước ngày nộp đơn đăng ký sáng chế hoặc trước ngày
ưu tiên trong trường hợp đơn đăng ký sáng chế được hưởng quyền ưu tiên.

- Sáng chế được coi là chưa bị bộc lộ công khai nếu chỉ có một số người có hạn được biết và có
nghĩa vụ giữ bí mật về sáng chế đó.

- Sáng chế không bị coi là mất tính mới nếu được người có quyền đăng ký quy định tại Điều 86
Luật Sở hữu trí tuệ 2005 hoặc người có được thông tin về sáng chế một cách trực tiếp hoặc gián

25
tiếp từ người đó bộc lộ công khai với điều kiện đơn đăng ký sáng chế được nộp tại Việt Nam
trong thời hạn mười hai tháng kể từ ngày bộc lộ
37. Phân tích điều kiện bảo hộ sáng chế: trình độ sáng tạo
Theo điều 61 Luật SHTT, “Sáng chế được coi là có trình độ sáng tạo nếu căn cứ vào các
giải pháp kỹ thuật đã được bộc lộ công khai dưới hình thức sử dụng, mô tả bằng văn bản hoặc
dưới bất kỳ hình thức nào khác ở trong nước hoặc ở nước ngoài trước ngày nộp đơn hoặc trước
ngày ưu tiên của đơn đăng ký sáng chế trong trường hợp đơn đăng ký sáng chế được hưởng
quyền ưu tiên, sáng chế đó là một bước tiến sáng tạo, không thể được tạo ra một cách dễ dàng
đối với người có hiểu biết trung bình về lĩnh vực kỹ thuật tương ứng”.
- Là bước tiến sáng tạo, không thể được tạo ra một cách dễ dàng (không hiển nhiên) đối
với người có hiểu biết trung bình về lĩnh vực kỹ thuật tương ứng
- Bản chất: đánh giá tập hợp dấu hiệu kỹ thuật cơ bản khác biệt của giải pháp yêu cầu bảo
hộ có bị coi là hiển nhiên (có được tạo ra một cách dễ dàng) đối người có hiểu biết trung
bình trong lĩnh vực kỹ thuật tương ứng hay không?
- Sáng chế bị coi là không có trình độ sáng tạo nếu:
+ Tập hợp dấu hiệu kỹ thuật cơ bản khác biệt đã được bộc lộ dưới dạng đồng nhất
hoặc tương đương trong giải pháp kỹ thuật đã biết và có căn cứ để người có hiểu
biết trung bình trong lĩnh vực kỹ thuật tương ứng kết hợp các giải pháp kỹ thuật
đã biết để tạo ra giải pháp yêu cầu bảo hộ.
+ Là sự kết hợp đơn giản của các giải pháp đã biết với chức năng và/hoặc hiệu quả
chỉ là sự kết hợp đơn giản chức năng/hiệu quả của từng giải pháp đã biết.
Sáng tạo không cần phải mới tuyệt đối.
38. Phân tích điều kiện bảo hộ sáng chế: khả năng áp dụng công nghiệp
Theo điều 62 Luật SHTT, “Sáng chế được coi là có khả năng áp dụng công nghiệp nếu
có thể thực hiện được việc chế tạo, sản xuất hàng loạt sản phẩm hoặc áp dụng lặp đi lặp lại quy
trình là nội dung của sáng chế và thu được kết quả ổn định”.
Một số ví dụ về đối tượng bị coi là không có khả năng áp dụng công nghiệp:
- Các sản phẩm hoặc quy trình hoạt động trái với các quy luật của tự nhiên và các nguyên
lý cơ bản của khoa học (ví dụ động cơ vĩnh cửu);
- Không ứng dụng được trong thực tế;
- Cần có kỹ năng đặc biệt để thực hiện đối tượng;
- Chỉ áp dụng được trong những điều kiện tự nhiên nhất định.
39. Phân tích các đối tượng không được bảo hộ với danh nghĩa sáng chế
Theo điều 59 Luật SHTT, “Các đối tượng sau đây không được bảo hộ với danh nghĩa
sáng chế:
1. Phát minh, lý thuyết khoa học, phương pháp toán học;
2. Sơ đồ, kế hoạch, quy tắc và phương pháp để thực hiện các hoạt động trí óc, huấn luyện vật
nuôi, thực hiện trò chơi, kinh doanh; chương trình máy tính;
3. Cách thức thể hiện thông tin;
Cách thức thể hiện thông tin: những đối tượng này rõ ràng chỉ là sự thuần tuý thể hiện thông tin
chứ không phải là một giải pháp kỹ thuật, tức là chúng cũng không có khả năng ứng dụng công
nghiệp. Hơn nữa, các đối tượng như “Sơ đồ, kế hoạch, quy tắc và phương pháp để thực hiện các
hoạt động trí óc, huấn luyện vật nuôi, thực hiện trò chơi, kinh doanh; chương trình máy tính”
được bảo hộ theo quyền tác giả.
26
4. Giải pháp chỉ mang đặc tính thẩm mỹ;
5. Giống thực vật, giống động vật;
6. Quy trình sản xuất thực vật, động vật chủ yếu mang bản chất sinh học mà không phải là quy
trình vi sinh;
7. Phương pháp phòng ngừa, chẩn đoán và chữa bệnh cho người và động vật”.
Phát minh, lý thuyết khoa học, phương pháp toán học, giải pháp chỉ mang đặc tính thẩm
mỹ là những đối tượng không đảm bảo đầy đủ được khả năng áp dụng công nghiệp của sáng chế.
Những đối tượng trên không thể được áp dụng trực tiếp trên đời sống, như phát minh, về bản
chất, phát minh là việc nhận ra những quy luật tồn tại một cách khách quan. Phát minh không thể
trực tiếp áp dụng vào đời sống mà phải thông qua sáng chế. Hơn nữa, cùng với sự phát triển của
khoa học, kỹ thuật, sáng chế có thể cũ đi, giá trị giảm dần theo thời gian, trong khi đó phát minh,
hay lý thuyết khoa học, phương pháp toán học lại tồn tại mãi mãi, là cơ sở để từ tạo ra sáng chế.
Sơ đồ, kế hoạch, quy tắc và phương pháp để thực hiện các hoạt động trí óc, huấn luyện
vật nuôi, thực hiện trò chơi, kinh doanh; chương trình máy tính; cách thức thể hiện thông tin. Các
đối tượng này chỉ thuần túy là sự thể hiện thông tin chứ không mang là một giải pháp kỹ thuật.
Do đó, không thể có khả năng áp dụng chúng vào sản xuất công nghiệp trong thực tiễn được.
Các đối tượng “sơ đồ, kế hoạch, quy tắc và phương pháp để thực hiện các hoạt động trí óc, huấn
luyện vật nuôi, thực hiện trò chơi, kinh doanh, chương trình máy tính được” bảo hộ theo quy
định quyền tác giả. Vì vậy, các đối tượng này không được bảo hộ với danh nghĩa sáng chế.
Giống thực vật, giống động vật; Quy trình sản xuất thực vật, động vật chủ yếu mang bản
chất sinh học mà không phải quy trình vi sinh. Các đối tượng này được bảo hộ trong lĩnh vực
khác bởi những đặc điểm khác biệt của các đối tượng này. Ví dụ như Quy trình để tạo ra một
giống cây trồng mới thường mất rất nhiều thời gian và tiền bạc. Trong khi đó, việc sao chép
giống cây trồng lại có thể được thực hiện nhanh chóng theo nhiều cách thức khác nhau, như:
chiết cây, giâm cây hoặc gieo hạt … Việc thời hạn hiệu lực của bằng độc quyền sáng chế (tối đa
chỉ 20 năm) lại không phù hợp đối với đối tượng giống cây trồng mới (thời gian khai thác giống
cây trồng có thể kéo dài đến 25 năm hoặc hơn thế). Do vậy, đòi hỏi các cần phải xây dựng cơ chế
bảo hộ hữu hiệu, riêng biệt cho đối tượng giống cây trồng mới, đảm bảo cho người tạo giống cây
trồng có thể khai thác lợi nhuận và tái đầu tư cho các hoạt động sáng tạo của mình.
Phương pháp phòng ngừa, chẩn đoán, chữa bệnh cho người và động vật. Loại trừ đối
tượng này ra khỏi phạm vi sáng chế được bảo hộ bởi vì việc tìm ra phương pháp phòng và chữa
bệnh cần phải mở rộng phạm vi áp dụng vì mục đích nhân đạo có tầm quan trọng rất lớn đến lợi
ích cộng đồng, nhu cầu phát triển kinh tế - xã hội của đất nước và không thể đưa ra để tư nhân
hóa hoặc thương mại hóa được.
Ngoài ra, những sáng chế trái với đạo đức xã hội, trật tự công cộng, có hại cho quốc
phòng, an ninh (theo khoản 1, điều 8 Luật Sở hữu trí tuệ). Quy định này nhằm mục đích “bảo vệ
trật tự công cộng và đạo đức xã hội, bảo vệ cuộc sống và sức khỏe của con người cũng như các
sinh vật khác để tránh gây nghiêm trọng cho môi trường”. Điều kiện này đặc biệt có ý nghĩa
trong những năm gần đây với sự phát triển của công nghệ gen. Cần thiết phải loại trừ việc cấp
bằng sáng chế cho quy trình nhân bản người và thay đổi cấu trúc gen nhận dạng người cũng như
các loại động vật khác.
40. Phân tích nguyên tắc cân bằng lợi ích trong bảo hộ quyền sở hữu công nghiệp đối với sáng
chế
27
Việc cấp bằng độc quyền bằng sáng chế được coi là động lực để đầu tư vào các hoạt động
sáng tạo và kiến thức chuyên sâu nhưng không phải lúc nào cũng đạt được mục đích là tăng cho
lợi ích công cộng, để cân bằng quyền lợi giữa chủ sở hữu sáng chế, bên thứ ba và công chúng,
phạm vi quyền độc quyền có thể có những ngoại lệ và hạn chế quyền. Quy định về các trường
hợp chủ sở hữu sáng chế không được quyền cấm người khác sử dụng sáng chế tại khoản 2 Điều
125 Luật SHTT. Quy định về bảo hộ sáng chế của pháp luật SHTT thì quy định về giới hạn
quyền sở hữu công nghiệp thể hiện rõ nhất nguyên tắc cân bằng lợi ích theo điều 134 Luật SHTT
về quyền của người sử dụng trước sáng chế sau khi văn bằng bảo hộ được cấp, người đó có
quyền tiếp tục sử dụng sáng chế, kiểu dáng công nghiệp trong phạm vi và khối lượng đã sử dụng
hoặc đã chuẩn bị để sử dụng mà không phải xin phép hoặc trả tiền đền bù cho chủ sở hữu sáng
chế nhưng để không ảnh hưởng tới quyền của chủ sở hữu sáng chế thì quyền của người sử dụng
trước đối với sáng chế chỉ được áp dụng khi đáp ứng điều kiện: sáng chế phải được tạo ra một
cách độc lập trước ngày nộp đơn hoặc ngày ưu tiên của đơn đăng ký sáng chế; sáng chế phải
đồng nhất với sáng chế đã được bảo hộ; sáng chế đã được sử dụng hoặc chuẩn bị các điều kiện
cần thiết để sử dụng.
41. Phân tích khái niệm nhãn hiệu.
Theo khoản 16 điều 4 Luật SHTT, “Nhãn hiệu là dấu hiệu dùng để phân biệt hàng hóa,
dịch vụ của các tổ chức, cá nhân khác nhau”.
Nhãn hiệu bao gồm những dấu hiệu mang dẫn chứng, nguồn gốc về thương mại (bản chất
là chỉ dẫn thương mại).
Nhãn hiệu chỉ được bảo hộ khi thỏa mãn các quy định của pháp luật và phải tiến hành
đăng ký tại cơ quan nhà nước có thẩm quyền.
42. Phân biệt nhãn hiệu với thương hiệu
Thương hiệu (brands) theo định nghĩa của Tổ chức sở hữu trí tuệ thế giới (WIPO): là một
dấu hiệu (hữu hình và vô hình) đặc biệt để nhận biết một sản phẩm hàng hoá hay một dịch vụ
nào đó được sản xuất hay được cung cấp bởi một cá nhân hay một tổ chức.
Nhãn hiệu (marks) theo định nghĩa tại Khoản 16 Điều 4 Luật SHTT là dấu hiệu dùng để
phân biệt hàng hoá, dịch vụ của các tổ chức, cá nhân khác nhau.
● Về mặt pháp lý
Ở góc độ pháp lý, sử dụng thuật ngữ nhãn hiệu hàng hóa là đúng như trong quy định của
pháp luật Việt Nam nhưng ở góc độ quản trị doanh nghiệp thì người ta sử dụng thuật ngữ thương
hiệu nhiều hơn.
Luật SHTT chưa có định nghĩa về thương hiệu mà chỉ đưa ra định nghĩa về nhãn hiệu, do
đó chỉ có nhãn hiệu mới về đối tượng được bảo hộ quyền sở hữu trí tuệ tại Việt Nam.
Vì chỉ có nhãn hiệu là đối tượng của Luật SHTT Việt Nam nên nhãn hiệu được các cơ
quan quản lý Nhà nước công nhận và bảo hộ còn thương hiệu là kết quả phấn đấu lâu dài của
doanh nghiệp và người tiêu dùng chính là người công nhận.
● Về khía cạnh vật chất
Nói đến thương hiệu là nói đến hình tượng về hàng hóa trong tâm trí người tiêu dùng. Ví
dụ như khi nói tới điện thoại Nokia, người dùng sẽ hình dung ra một sản phẩm bền, điện thoại
Iphone thì “sang chảnh”,…
Còn nhãn hiệu lại có thể là từ ngữ, hình ảnh, biểu tượng…giúp khách hàng nhận diện bên
ngoài của hàng hóa.
● Về thời gian tồn tại
Thương hiệu tồn tại lâu hơn nhãn hiệu. Có những thương hiệu nổi tiếng mãi theo thời
gian nhưng nhãn hiệu thì thay đổi theo những yếu tố tác động bên ngoài nhất định như thị hiếu

28
người tiêu dùng… Hơn nữa nhãn hiệu được bảo hộ trong thời gian có hạn, còn thương hiệu được
định vị lâu dài trong tâm trí của người tiêu dùng.
Tiêu chí Thương hiệu Nhãn hiệu

Về mặt pháp lý Thương hiệu không là đối tượng Nhãn hiệu là đối tượng được bảo
được bảo hộ quyền sở hữu trí tuệ hộ quyền sở hữu trí tuệ tại Việt
tại Việt Nam Nam

Về khía cạnh vật Tồn tại trong tâm trí người tiêu Người tiêu dùng nhận diện qua
chất dùng hình ảnh, từ ngữ, biểu tượng,…

Về thời gian tồn Lâu dài Có thời hạn


tại

43. Phân biệt nhãn hiệu với nhãn hàng hóa


Nhãn hiệu là dấu hiệu có khả năng phân biệt hàng hóa, dịch vụ do một doanh nghiệp sản
xuất hoặc cung ứng với hàng hóa, dịch vụ của các doanh nghiệp khác. (Theo khoản 16, Điều 4,
Luật SHTT). Nhãn hiệu chỉ được bảo hộ khi thỏa mãn các quy định của pháp luật và phải tiến
hành đăng ký tại cơ quan nhà nước có thẩm quyền.
Nhãn hàng hóa là bản viết, bản in, bản vẽ, bản chụp của chữ, hình vẽ, hình ảnh được dán,
in, đính, đúc, chạm, khắc trực tiếp trên hàng hóa, bao bì, thương phẩm của hàng hóa hoặc trên
các chất liệu khác được gắn trên hàng hóa, bao bì, thương phẩm của hàng hóa. Theo quy định
của pháp luật thì hàng hóa lưu thông trong nước, hàng hóa xuất khẩu phải bắt buộc ghi nhãn.
Nhãn hàng hóa phải được gắn trên hàng hóa, bao bì, thương phẩm của hàng hóa ở vị trí khi quan
sát có thể nhận biết được dễ dàng, đầy đủ các nội dung quy định của nhãn mà không phải tháo
rời các chi tiết, các thành phần của hàng hóa. Nhãn hàng hóa không phải đăng ký với cơ quan
nhà nước có thẩm quyền. Nhãn hàng hóa được sử dụng với nhiều mục đích khác nhau : chỉ dẫn
nhà sản xuất, năm sản xuất, nước sản xuất hay các thông số về hàng, kích thước, thành phần hoặc
các đặc tính khác của hàng hóa. Nhãn hàng hóa có thể bao gồm cả nhãn hiệu.
Như vậy, sự khác nhau cơ bản giữa nhãn hàng hóa và nhãn hiệu là nhãn hàng hóa có
chức năng chủ yếu là ghi thông tin của hàng hóa còn nhãn hiệu để phân biệt hàng hóa, dịch vụ
của doanh nghiệp này với hàng hóa, dịch vụ cùng loại doanh nghiệp khác.
44. Phân tích khái niệm nhãn hiệu tập thể, nhãn hiệu liên kết, nhãn hiệu chứng nhận
Theo khoản 17,18,19 điều 4 Luật SHTT
“Nhãn hiệu tập thể là nhãn hiệu dùng để phân biệt hàng hóa, dịch vụ của các thành viên của tổ
chức là chủ sở hữu nhãn hiệu đó với hàng hóa, dịch vụ của tổ chức, cá nhân không phải là thành
viên của tổ chức đó.
Nhãn hiệu chứng nhận là nhãn hiệu mà chủ sở hữu nhãn hiệu cho phép tổ chức, cá nhân khác sử
dụng trên hàng hóa, dịch vụ của tổ chức, cá nhân đó để chứng nhận các đặc tính về xuất xứ,
nguyên liệu, vật liệu, cách thức sản xuất hàng hóa, cách thức cung cấp dịch vụ, chất lượng, độ
chính xác, độ an toàn hoặc các đặc tính khác của hàng hóa, dịch vụ mang nhãn hiệu.
Nhãn hiệu liên kết là các nhãn hiệu do cùng một chủ thể đăng ký, trùng hoặc tương tự nhau dùng
cho sản phẩm, dịch vụ cùng loại hoặc tương tự nhau hoặc có liên quan với nhau”.

29
Như vậy, Nhãn hiệu tập thể là nhãn hiệu có nhiều chủ đồng sở hữu và sử dụng; Nhãn
hiệu liên kết là các nhãn hiệu giống hoặc tương tự nhau do cùng 1 chủ sở hữu đăng ký để sử
dụng trên các sản phẩm có nhiều phiên bản khác nhau; Nhãn hiệu chứng nhận dùng để chứng
nhận về đặc tính của sản phẩm như nguồn gốc xuất xứ, nguyên liệu, phương pháp sản xuất, tiêu
chuẩn chất lượng…
45. Phân loại nhãn hiệu theo hình thức của nhãn hiệu
Theo khoản 16, Điều 4 Luật SHTT 2005: “Nhãn hiệu là dấu hiệu dùng để phân biệt hàng hóa,
dịch vụ của các tổ chức, cá nhân khác nhau.” Việc phân loại nhãn hiệu cho phép xác định đặc
trưng của một loại nhãn hiệu mà khi đó đặc trưng này không chỉ ảnh hưởng tới yếu tố cảm nhận
về nhãn hiệu đối với người tiêu dùng, tính chất của việc sử dụng nhãn hiệu mà còn ảnh hưởng tới
quá trình đăng ký nhãn hiệu.
Theo hình thức thể hiện nhãn hiệu có thể ở dạng từ ngữ, chữ cái, chữ số, hình ảnh, hình khối
hoặc là sự kết hợp các yếu tố này thể hiện trên một hoặc nhiều màu sắc. Nhãn hiệu cũng có thể là
những dấu hiệu đặc biệt như dấu hiệu âm thanh mùi vị, ánh sáng.
- Nhãn hiệu từ ngữ: được phân làm 02 loại (nhãn hiệu từ ngữ đơn thuần thể hiện ý nghĩa và
nhãn hiệu từ ngữ được thực hiện với phong cách đồ họa đặc biệt) bao gồm những dấu hiệu là các
từ, sự kết hợp giữa các chữ cái có thể phát âm, cụm từ, câu, những đơn vị tiếng khác cũng như sự
kết hợp giữa chúng. Nhãn hiệu là từ ngữ mang trong mình những nội dung có ý nghĩa đặc biệt,
trong sự kết hợp với hiệu ứng phát âm của từ ngữ có khả năng tạo ra trong người tiêu dùng
những cảm nhận tích cực về nhà sản xuất hay bên cung ứng sản phẩm, dịch vụ. Nhãn hiệu từ ngữ
đang dần trở nên chiếm ưu thế hơn so với nhãn hiệu hình ảnh.
- Nhãn hiệu chữ cái, chữ số: Có thể là sự kết hợp giữa các chữ cái đơn lẻ (BMW), con số (333).
Ở một số quốc gia nhãn hiệu là chữ cái và chữ số không được chấp nhận với lý do không có khả
năng phân biệt, tuy nhiên trong thực tiễn bảo hộ, các nhãn hiệu dạng này vẫn có thể được chấp
nhận đăng ký nếu chúng được trình bày dưới dạng thức độc đáo, có khả năng phân biệt. Thông
thường các chữ số được coi là không có khả năng phân biệt, tuy nhiên số lượng thể hiện dưới
dạng chữ thì lại được coi là có khả năng phân biệt.
- Nhãn hiệu hình vẽ, hình ảnh (dấu hiệu hình): Là những nhãn hiệu mà khả năng phân biệt của
chúng đạt được thông qua những cách trình bày mang tính nghệ thuật và chủ yếu hướng tới
những cảm nhận về thị giác của người tiêu dùng. Nhãn hiệu hình tượng là những dấu hiệu ở các
dạng hình vẽ, là sự kết hợp giữa các đường nét dấu hiệu trên mặt phẳng; đặc trưng cơ bản của
các dấu hiệu này là sử dụng các yếu tố đồ họa, xây dựng biểu tượng theo nguyên tắc bố cục mỹ
thuật màu sắc. (VD: biểu tượng hãng xe Jaguar,...)
- Nhãn hiệu hình khối: Là những dấu hiệu được hình thành dựa trên sự kết hợp giữa các yếu tố
đường nét, hình ảnh, màu sắc trên không gian 3 chiều, có khả năng phân biệt rất lớn. Những dấu
hiệu này có thể là hình dáng đặc biệt của sản phẩm (viên hút mùi, thỏi socola,..) hoặc hình dáng
của bao bì sản phẩm (lọ nước hoa, vỏ chai,...) VD: hình dáng chai COCA-COLA. Một dấu hiệu
là hình khối có thể được bảo hộ với tư cách là kiểu dáng công nghiệp.
- Nhãn hiệu kết hợp: được tạo ra trên cơ sở kết hợp giữa những yếu tố về từ ngữ, chữ cái, chữ số
và những yếu tố hình ảnh, hình khối; trong đó phần hình ảnh và từ ngữ thường minh họa lẫn
nhau.
- Nhãn hiệu đặc biệt (âm thanh, hình ảnh, ánh sáng): Khác với các dấu hiệu trên, nhãn hiệu
đặc biệt là những dấu hiệu có thể được cảm nhận bởi các giác quan khác của con người như
thính giác, vị giác, khứu giác. Việc đăng ký và bảo hộ những dấu hiệu này không phải là phổ
biến và chủ yếu được dùng để phân biệt trong lĩnh vực dịch vụ, đó có thể là mùi hương đặc biệt
trong thuốc cho trẻ em, bản nhạc hiệu của một chương trình truyền hình (VD: đoạn intro sư tử

30
gầm của MGM), …
46. Phân loại nhãn hiệu theo mức độ khả năng phân biệt của nhãn hiệu.
- Khả năng phân biệt của nhãn hiệu được quy định tại Điều 74 Luật SHTT 2005, trong đó
có thể phân loại như sau:
- Nhãn hiệu không có khả năng tự phân biệt (điều này xuất phát từ đặc điểm của nhãn
hiệu đó), bao gồm các dấu hiệu sau:
+ Hình và hình hình học đơn giản, chữ số, chữ cái, chữ thuộc các ngôn ngữ không
thông dụng, trừ trường hợp các dấu hiệu này đã được sử dụng và thừa nhận rộng
rãi với danh nghĩa một nhãn hiệu;
+ Dấu hiệu, biểu tượng quy ước, hình vẽ hoặc tên gọi thông thường của hàng hóa,
dịch vụ bằng bất kỳ ngôn ngữ nào đã được sử dụng rộng rãi, thường xuyên, nhiều
người biết đến;
+ Dấu hiệu chỉ thời gian, địa điểm, phương pháp sản xuất, chủng loại, số lượng,
chất lượng, tính chất, thành phần, công dụng, giá trị hoặc các đặc tính khác mang
tính mô tả hàng hóa, dịch vụ, trừ trường hợp dấu hiệu đó đã đạt được khả năng
phân biệt thông qua quá trình sử dụng trước thời điểm nộp đơn đăng ký nhãn
hiệu;
+ Dấu hiệu mô tả hình thức pháp lý, lĩnh vực kinh doanh của chủ thể kinh doanh;
+ Dấu hiệu chỉ nguồn gốc địa lý của hàng hóa, dịch vụ, trừ trường hợp dấu hiệu đó
đã được sử dụng và thừa nhận rộng rãi với danh nghĩa một nhãn hiệu hoặc được
đăng ký dưới dạng nhãn hiệu tập thể hoặc nhãn hiệu chứng nhận quy định tại
Luật này;
- Nhãn hiệu không có khả năng phân biệt với nhãn hiệu khác: là nhãn hiệu có các dấu
hiệu trùng hoặc tương tự đến mức gây nhầm lẫn về cấu trúc, nội dung, cách phát âm, ý
nghĩa và hình thức thể hiện với hàng hóa, dịch vụ của chủ thể khác dưới đây được coi là
không có khả năng phân biệt:
+ Dấu hiệu không phải là nhãn hiệu liên kết trùng hoặc tương tự đến mức gây nhầm
lẫn với nhãn hiệu đã được đăng ký cho hàng hóa, dịch vụ trùng hoặc tương tự
trên cơ sở đơn đăng ký có ngày nộp đơn hoặc ngày ưu tiên sớm hơn trong trường
hợp đơn đăng ký được hưởng quyền ưu tiên, kể cả đơn đăng ký nhãn hiệu được
nộp theo điều ước quốc tế mà Cộng hòa xã hội chủ nghĩa Việt Nam là thành viên;
+ Dấu hiệu trùng hoặc tương tự đến mức gây nhầm lẫn với nhãn hiệu của người
khác đã được sử dụng và thừa nhận rộng rãi cho hàng hóa, dịch vụ trùng hoặc
tương tự từ trước ngày nộp đơn hoặc ngày ưu tiên trong trường hợp đơn được
hưởng quyền ưu tiên;
+ Dấu hiệu trùng hoặc tương tự đến mức gây nhầm lẫn với nhãn hiệu của người
khác đã đăng ký cho hàng hóa, dịch vụ trùng hoặc tương tự mà đăng ký nhãn
hiệu đó đã chấm dứt hiệu lực chưa quá năm năm, trừ trường hợp hiệu lực bị
chấm dứt vì lý do nhãn hiệu không được sử dụng theo quy định tại điểm d khoản 1
Điều 95 của Luật này;
+ Dấu hiệu trùng hoặc tương tự đến mức gây nhầm lẫn với nhãn hiệu được coi là
nổi tiếng của người khác đăng ký cho hàng hóa, dịch vụ trùng hoặc tương tự với
hàng hóa, dịch vụ mang nhãn hiệu nổi tiếng hoặc đăng ký cho hàng hóa, dịch vụ
không tương tự, nếu việc sử dụng dấu hiệu đó có thể làm ảnh hưởng đến khả năng
phân biệt của nhãn hiệu nổi tiếng hoặc việc đăng ký nhãn hiệu nhằm lợi dụng uy
tín của nhãn hiệu nổi tiếng;

31
- Nhãn hiệu không có khả năng phân biệt với đối tượng sở hữu công nghiệp khác:
+ Dấu hiệu trùng hoặc tương tự với tên thương mại đang được sử dụng của người
khác, nếu việc sử dụng dấu hiệu đó có thể gây nhầm lẫn cho người tiêu dùng về
nguồn gốc hàng hóa, dịch vụ;
+ Dấu hiệu trùng hoặc tương tự với chỉ dẫn địa lý đang được bảo hộ nếu việc sử
dụng dấu hiệu đó có thể làm cho người tiêu dùng hiểu sai lệch về nguồn gốc địa
lý của hàng hóa;
+ Dấu hiệu trùng với chỉ dẫn địa lý hoặc có chứa chỉ dẫn địa lý hoặc được dịch
nghĩa, phiên âm từ chỉ dẫn địa lý đang được bảo hộ cho rượu vang, rượu mạnh
nếu dấu hiệu được đăng ký để sử dụng cho rượu vang, rượu mạnh không có
nguồn gốc xuất xứ từ khu vực địa lý mang chỉ dẫn địa lý đó;
+ Dấu hiệu trùng hoặc không khác biệt đáng kể với kiểu dáng công nghiệp của
người khác được bảo hộ trên cơ sở đơn đăng ký kiểu dáng công nghiệp có ngày
nộp đơn hoặc ngày ưu tiên sớm hơn so với ngày nộp đơn, ngày ưu tiên của đơn
đăng ký nhãn hiệu.
47. Phân tích điều kiện bảo hộ nhãn hiệu: dấu hiệu nhìn thấy được
- Điều 72. Điều kiện chung đối với nhãn hiệu được bảo hộ
Nhãn hiệu được bảo hộ nếu đáp ứng các điều kiện sau đây:
1. Là dấu hiệu nhìn thấy được dưới dạng chữ cái, từ ngữ, hình vẽ, hình ảnh, kể cả hình ba chiều
hoặc sự kết hợp các yếu tố đó, được thể hiện bằng một hoặc nhiều màu sắc;
2. Có khả năng phân biệt hàng hóa, dịch vụ của chủ sở hữu nhãn hiệu với hàng hóa, dịch vụ của
chủ thể khác
- Đối với tiêu chí “dấu hiệu nhìn thấy được” thì các dấu hiệu của nhãn hiệu phải nhìn thấy được,
có thể “tri giác” được. Theo quy định của pháp luật thì đối với đặc điểm có thể nhìn thấy được,
có nghĩa là con người chỉ có thể nhận thức, nắm bắt được về chúng thông qua khả năng thị giác
(VD: người tiêu dùng thông qua quan sát, nhìn ngắm để lựa chọn hàng hóa có gắn với nhãn hiệu
để mua). Theo pháp luật của một số nước trên thế giới thì việc bảo hộ đối với nhãn hiệu được áp
dụng đối với nhãn hiệu đặc biệt mà con người cảm nhận được thông qua khứu giác, thính giác
hay vị giác (tham khảo kỹ hơn ở câu 45).
- Nhãn hiệu có thể được thể hiện thông qua các dấu hiệu chữ cái, từ ngữ, hình ảnh, kể cả hình ba
chiều hoặc sự kết hợp giữa các yếu tố đó. Yếu tố màu sắc là không thể thiếu đối với nhãn hiệu
hàng hóa bởi ưu điểm gây ấn tượng mạnh với thị giác con người, thông qua đó giúp cho nhãn
hiệu thực hiện được chức năng phân biệt của mình. Pháp luật Việt Nam không chấp nhận bảo hộ
dấu hiệu chỉ đơn thuần là màu sắc như mảng màu, vệt màu mà không được kết hợp với các dấu
hiệu chữ hoặc dấu hiệu hình hoặc không được thể hiện dưới dạng dấu hiệu chữ hoặc hình; tuy
nhiên những dấu hiệu này vẫn có thể được chấp nhận bảo hộ nếu như chúng đã và đang được sử
dụng với chức năng nhãn hiệu và được người tiêu dùng biết đến rộng rãi, nhờ đó mà nhãn hiệu
đạt được khả năng phân biệt đối với hàng hóa và dịch vụ liên quan.
Bên cạnh đó, Luật SHTT cũng có quy định các dấu hiệu bị loại trừ khi xem xét cấp bằng bảo hộ
đối với nhãn hiệu tại Điều 73 như sau:
“Điều 73. Dấu hiệu không được bảo hộ với danh nghĩa nhãn hiệu
Các dấu hiệu sau đây không được bảo hộ với danh nghĩa nhãn hiệu:

32
1. Dấu hiệu trùng hoặc tương tự đến mức gây nhầm lẫn với hình quốc kỳ, quốc huy của các
nước;
2. Dấu hiệu trùng hoặc tương tự đến mức gây nhầm lẫn với biểu tượng, cờ, huy hiệu, tên viết tắt,
tên đầy đủ của cơ quan nhà nước, tổ chức chính trị, tổ chức chính trị - xã hội, tổ chức chính trị
xã hội - nghề nghiệp, tổ chức xã hội, tổ chức xã hội - nghề nghiệp của Việt Nam và tổ chức quốc
tế, nếu không được cơ quan, tổ chức đó cho phép;
3. Dấu hiệu trùng hoặc tương tự đến mức gây nhầm lẫn với tên thật, biệt hiệu, bút danh, hình
ảnh của lãnh tụ, anh hùng dân tộc, danh nhân của Việt Nam, của nước ngoài;
4. Dấu hiệu trùng hoặc tương tự đến mức gây nhầm lẫn với dấu chứng nhận, dấu kiểm tra, dấu
bảo hành của tổ chức quốc tế mà tổ chức đó có yêu cầu không được sử dụng, trừ trường hợp
chính tổ chức này đăng ký các dấu đó làm nhãn hiệu chứng nhận;
5. Dấu hiệu làm hiểu sai lệch, gây nhầm lẫn hoặc có tính chất lừa dối người tiêu dùng về nguồn
gốc xuất xứ, tính năng, công dụng, chất lượng, giá trị hoặc các đặc tính khác của hàng hóa, dịch
vụ.”
48. Phân tích điều kiện bảo hộ nhãn hiệu: có khả năng phân biệt
- Được quy định tại khoản 2, Điều 72 về điều kiện chung đối với nhãn hiệu được bảo hộ thì nhãn
hiệu phải: “Có khả năng phân biệt hàng hóa, dịch vụ của chủ sở hữu nhãn hiệu với hàng hóa,
dịch vụ của chủ thể khác”. Cụ thể điều này được giải đáp trong Điều 74 cùng bộ luật.
Điều 74. Khả năng phân biệt của nhãn hiệu
1. Nhãn hiệu được coi là có khả năng phân biệt nếu được tạo thành từ một hoặc một số yếu tố dễ
nhận biết, dễ ghi nhớ hoặc từ nhiều yếu tố kết hợp thành một tổng thể dễ nhận biết, dễ ghi nhớ
và không thuộc các trường hợp quy định tại khoản 2 Điều này.
2. Nhãn hiệu bị coi là không có khả năng phân biệt nếu nhãn hiệu đó là dấu hiệu thuộc một
trong các trường hợp sau đây:
a) Hình và hình hình học đơn giản, chữ số, chữ cái, chữ thuộc các ngôn ngữ không thông dụng,
trừ trường hợp các dấu hiệu này đã được sử dụng và thừa nhận rộng rãi với danh nghĩa một
nhãn hiệu;
b) Dấu hiệu, biểu tượng quy ước, hình vẽ hoặc tên gọi thông thường của hàng hóa, dịch vụ bằng
bất kỳ ngôn ngữ nào đã được sử dụng rộng rãi, thường xuyên, nhiều người biết đến;
c) Dấu hiệu chỉ thời gian, địa điểm, phương pháp sản xuất, chủng loại, số lượng, chất lượng,
tính chất, thành phần, công dụng, giá trị hoặc các đặc tính khác mang tính mô tả hàng hóa, dịch
vụ, trừ trường hợp dấu hiệu đó đã đạt được khả năng phân biệt thông qua quá trình sử dụng
trước thời điểm nộp đơn đăng ký nhãn hiệu;
d) Dấu hiệu mô tả hình thức pháp lý, lĩnh vực kinh doanh của chủ thể kinh doanh;
đ) Dấu hiệu chỉ nguồn gốc địa lý của hàng hóa, dịch vụ, trừ trường hợp dấu hiệu đó đã được sử
dụng và thừa nhận rộng rãi với danh nghĩa một nhãn hiệu hoặc được đăng ký dưới dạng nhãn
hiệu tập thể hoặc nhãn hiệu chứng nhận quy định tại Luật này;
e) Dấu hiệu không phải là nhãn hiệu liên kết trùng hoặc tương tự đến mức gây nhầm lẫn với
nhãn hiệu đã được đăng ký cho hàng hóa, dịch vụ trùng hoặc tương tự trên cơ sở đơn đăng ký có
ngày nộp đơn hoặc ngày ưu tiên sớm hơn trong trường hợp đơn đăng ký được hưởng quyền ưu

33
tiên, kể cả đơn đăng ký nhãn hiệu được nộp theo điều ước quốc tế mà Cộng hòa xã hội chủ nghĩa
Việt Nam là thành viên;
g) Dấu hiệu trùng hoặc tương tự đến mức gây nhầm lẫn với nhãn hiệu của người khác đã được
sử dụng và thừa nhận rộng rãi cho hàng hóa, dịch vụ trùng hoặc tương tự từ trước ngày nộp đơn
hoặc ngày ưu tiên trong trường hợp đơn được hưởng quyền ưu tiên;
h) Dấu hiệu trùng hoặc tương tự đến mức gây nhầm lẫn với nhãn hiệu của người khác đã đăng
ký cho hàng hóa, dịch vụ trùng hoặc tương tự mà đăng ký nhãn hiệu đó đã chấm dứt hiệu lực
chưa quá năm năm, trừ trường hợp hiệu lực bị chấm dứt vì lý do nhãn hiệu không được sử dụng
theo quy định tại điểm d khoản 1 Điều 95 của Luật này;
i) Dấu hiệu trùng hoặc tương tự đến mức gây nhầm lẫn với nhãn hiệu được coi là nổi tiếng của
người khác đăng ký cho hàng hóa, dịch vụ trùng hoặc tương tự với hàng hóa, dịch vụ mang nhãn
hiệu nổi tiếng hoặc đăng ký cho hàng hóa, dịch vụ không tương tự, nếu việc sử dụng dấu hiệu đó
có thể làm ảnh hưởng đến khả năng phân biệt của nhãn hiệu nổi tiếng hoặc việc đăng ký nhãn
hiệu nhằm lợi dụng uy tín của nhãn hiệu nổi tiếng;
k) Dấu hiệu trùng hoặc tương tự với tên thương mại đang được sử dụng của người khác, nếu
việc sử dụng dấu hiệu đó có thể gây nhầm lẫn cho người tiêu dùng về nguồn gốc hàng hóa, dịch
vụ;
l) Dấu hiệu trùng hoặc tương tự với chỉ dẫn địa lý đang được bảo hộ nếu việc sử dụng dấu hiệu
đó có thể làm cho người tiêu dùng hiểu sai lệch về nguồn gốc địa lý của hàng hóa;
m) Dấu hiệu trùng với chỉ dẫn địa lý hoặc có chứa chỉ dẫn địa lý hoặc được dịch nghĩa, phiên
âm từ chỉ dẫn địa lý đang được bảo hộ cho rượu vang, rượu mạnh nếu dấu hiệu được đăng ký để
sử dụng cho rượu vang, rượu mạnh không có nguồn gốc xuất xứ từ khu vực địa lý mang chỉ dẫn
địa lý đó;
n) Dấu hiệu trùng hoặc không khác biệt đáng kể với kiểu dáng công nghiệp của người khác được
bảo hộ trên cơ sở đơn đăng ký kiểu dáng công nghiệp có ngày nộp đơn hoặc ngày ưu tiên sớm
hơn so với ngày nộp đơn, ngày ưu tiên của đơn đăng ký nhãn hiệu.
- Mục 39.6 Thông tư 01 quy định về khả năng phân biệt của một số dấu hiệu kết hợp thỏa mãn
các trường hợp sau:
+ Dấu hiệu chữ và dấu hiệu hình đều có khả năng phân biệt và tạo thành tổng thể có khả
năng phân biệt
+ Thành phần mạnh của nhãn hiệu (yếu tố tác động mạnh vào cảm giác của người tiêu
dùng, gây chú ý và ấn tượng về nhãn hiệu khi quan sát) là dấu hiệu chữ hoặc dấu hiệu
hình có khả năng phân biệt, mặc dù thành phần còn lại không có hoặc ít có khả năng
phân biệt.
+ Trường hợp dấu hiệu kết hợp gồm các dấu hiệu chữ và dấu hiệu hình không có hoặc ít có
khả năng phân biệt những cách thức kết hợp độc đáo của các dấu hiệu đó tạo ra một ấn
tượng riêng biệt thì tổng thể kết hợp đó vẫn được coi là có khả năng phân biệt.
+ Dấu hiệu kết hợp gồm các thành phần chữ và hình không có hoặc ít có khả năng phân
biệt nhưng tổng thể kết hợp đó đã đạt được khả năng phân biệt trong quá trình sử dụng.
- Tuy vậy trên thực tế, khả năng phân biệt của nhãn hiệu phải được đánh giá trong tổng thể
sự kết hợp giữa các yếu tố tạo thành nhãn hiệu chứ không phải chỉ dựa trên việc xem xét
khả năng phân biệt của từng yếu tố một cách riêng biệt.
49. Phân tích các dấu hiệu không được bảo hộ với danh nghĩa nhãn hiệu
34
Điều 73. Dấu hiệu không được bảo hộ với danh nghĩa nhãn hiệu
Các dấu hiệu sau đây không được bảo hộ với danh nghĩa nhãn hiệu:
1. Dấu hiệu trùng hoặc tương tự đến mức gây nhầm lẫn với hình quốc kỳ, quốc huy của các
nước;
2. Dấu hiệu trùng hoặc tương tự đến mức gây nhầm lẫn với biểu tượng, cờ, huy hiệu, tên viết tắt,
tên đầy đủ của cơ quan nhà nước, tổ chức chính trị, tổ chức chính trị - xã hội, tổ chức chính trị
xã hội - nghề nghiệp, tổ chức xã hội, tổ chức xã hội - nghề nghiệp của Việt Nam và tổ chức quốc
tế, nếu không được cơ quan, tổ chức đó cho phép;
3. Dấu hiệu trùng hoặc tương tự đến mức gây nhầm lẫn với tên thật, biệt hiệu, bút danh, hình
ảnh của lãnh tụ, anh hùng dân tộc, danh nhân của Việt Nam, của nước ngoài;
4. Dấu hiệu trùng hoặc tương tự đến mức gây nhầm lẫn với dấu chứng nhận, dấu kiểm tra, dấu
bảo hành của tổ chức quốc tế mà tổ chức đó có yêu cầu không được sử dụng, trừ trường hợp
chính tổ chức này đăng ký các dấu đó làm nhãn hiệu chứng nhận;
5. Dấu hiệu làm hiểu sai lệch, gây nhầm lẫn hoặc có tính chất lừa dối người tiêu dùng về nguồn
gốc xuất xứ, tính năng, công dụng, chất lượng, giá trị hoặc các đặc tính khác của hàng hóa, dịch
vụ.
50. Phân tích các tiêu chí xác định hai nhãn hiệu là tương tự đến mức gây nhầm lẫn?
Việc xác định một nhãn hiệu có “trùng” với một nhãn hiệu khác hay không thì chỉ cần xem xét
liệu có sự copy paste y nguyên nhau không, thế nhưng đối với tính “tương tự tới mức gây nhầm
lẫn” giữa các nhãn hiệu thì lại khá phức tạp do có nhiều yếu tố ảnh hưởng đến sự nhầm lẫn hoặc
có khi là không yếu tố nào giữ vai trò quyết định hơn các yếu tố còn lại.
Các tiêu chí chính để xác định sự tương đồng đến mức gây nhầm lẫn giữa hai nhãn hiệu bao
gồm:
- Phát âm: cách viết khác nhau nhưng phát âm giống nhau (VD: Windy - Windee)
- Cấu trúc: Cấu trúc, cách sắp xếp từ ngữ giống nhau, các từ có ý nghĩa và phát âm giống
nhau nhưng khác kiểu chữ thể hiện (VD: lays - LAYS)
- Nghĩa/Ý nghĩa: Trình bày dưới dạng ngôn ngữ khác nhau tuy nhiên lại có cùng ý nghĩa
(VD: Clown Z - Chú hề Z)
- Ấn tượng tổng thể: Nhãn hiệu sử dụng hình ảnh tổng thể tương tự nhau dẫn đến sự nhầm
lẫn cho người tiêu dùng về xuất xứ/nguồn gốc sản phẩm (VD: 2 nhãn hiệu cùng sử dụng
01 hình ảnh quả dừa thay cho tên sản phẩm, nhưng nhãn hiệu đối chứng thì sử dụng với
mục đích bán nước dừa đóng lon còn nhãn hiệu dự định đăng ký sử dụng để bán chè dừa
dầm)
- Sản phẩm và dịch vụ tương tự/liên quan: Tên nhãn hiệu giống nhau về ý nghĩa, cấu
trúc, có thể gây nhầm lẫn vì sản phẩm và dịch vụ tương tự nhau. (VD: Nhà hàng tổ chức
tiệc cưới - Nhà hàng cung cấp dịch vụ tiệc cưới)
- Theo mục 39.8, Điều 39 của Thông tư 01/2007/TT-CP, sửa đổi bổ sung 2016) thì:
39.8 Đánh giá sự tương tự đến mức gây nhầm lẫn của dấu hiệu yêu cầu đăng ký với nhãn
hiệu khác
a) Để đánh giá dấu hiệu yêu cầu đăng ký nêu trong đơn có trùng hoặc tương tự đến mức gây
nhầm lẫn với một nhãn hiệu khác (sau đây gọi là “nhãn hiệu đối chứng”) hay không, cần phải so
sánh về cấu trúc, nội dung, cách phát âm (đối với dấu hiệu chữ), ý nghĩa và hình thức thể hiện
của dấu hiệu (đối với cả dấu hiệu chữ và dấu hiệu hình), đồng thời phải tiến hành so sánh hàng
35
hóa, dịch vụ mang dấu hiệu với hàng hoá, dịch vụ mang nhãn hiệu đối chứng theo quy định tại
điểm này.
b) Dấu hiệu trùng với nhãn hiệu đối chứng: dấu hiệu bị coi là trùng với nhãn hiệu đối chứng nếu
dấu hiệu đó giống hệt nhãn hiệu đối chứng về cấu trúc, nội dung, ý nghĩa và hình thức thể hiện.
c) Dấu hiệu bị coi là tương tự đến mức gây nhầm lẫn với nhãn hiệu đối chứng nếu:
(i) Dấu hiệu đó gần giống với nhãn hiệu đối chứng về cấu trúc hoặc/và nội dung hoặc/và cách
phát âm hoặc/và ý nghĩa hoặc/và hình thức thể hiện đến mức làm cho người tiêu dùng tưởng lầm
rằng hai đối tượng đó là một hoặc đối tượng này là biến thể của đối tượng kia hoặc hai đối tượng
đó có cùng một nguồn gốc;
(ii) Dấu hiệu chỉ là bản phiên âm hoặc dịch nghĩa từ nhãn hiệu đối chứng nếu nhãn hiệu đối
chứng là nhãn hiệu nổi tiếng.
51. Phân tích các điều kiện bảo hộ tên thương mại
52. Theo khoản 21, Điều 4 Luật SHTT 2005 thì: “Tên thương mại là tên gọi của tổ chức,
cá nhân dùng trong hoạt động kinh doanh để phân biệt chủ thể kinh doanh mang tên
gọi đó với chủ thể kinh doanh khác trong cùng lĩnh vực và khu vực kinh doanh.” Như
vậy, chức năng chính của tên thương mại là nhằm phân biệt, cá thể hóa chủ thể kinh
doanh này với chủ thể kinh doanh khác và đây cũng là điều kiện để đảm bảo tính
phân biệt của tên thương mại khi đặt trong yêu cầu bảo hộ.
Điều 76. Điều kiện chung đối với tên thương mại được bảo hộ
Tên thương mại được bảo hộ nếu có khả năng phân biệt chủ thể kinh doanh mang tên thương
mại đó với chủ thể kinh doanh khác trong cùng lĩnh vực và khu vực kinh doanh.
- Theo như Điều 76 thì cùng lĩnh vực kinh doanh và khu vực kinh doanh là 2 điều kiện luôn đi
đôi với nhau, nếu thiếu một trong hai yếu tố này thì tên thương mại sẽ không được bảo hộ (VD: 2
tên thương mại trùng nhau nhưng hai chủ thể kinh doanh không hoạt động trên cùng khu vực địa
lý hoặc không cùng kinh doanh trong một lĩnh vực). Điều kiện bảo hộ của tên thương mại cũng
được đánh giá là đơn giản hơn rất nhiều so với điều kiện bảo hộ nhãn hiệu, điều này cũng được
đồng thời thể hiện ở các đối tượng không được bảo hộ dưới danh nghĩa tên thương mại theo Điều
77.
“Điều 77. Đối tượng không được bảo hộ với danh nghĩa tên thương mại
Tên của cơ quan nhà nước, tổ chức chính trị, tổ chức chính trị - xã hội, tổ chức chính trị xã hội -
nghề nghiệp, tổ chức xã hội, tổ chức xã hội - nghề nghiệp hoặc chủ thể khác không liên quan đến
hoạt động kinh doanh thì không được bảo hộ với danh nghĩa tên thương mại.”
53. Phân tích các điều kiện bảo hộ chỉ dẫn địa lý
- Khoản 22, Điều 4 Luật SHTT 2005 quy định: “Chỉ dẫn địa lý là dấu hiệu dùng để chỉ sản phẩm
có nguồn gốc từ khu vực, địa phương, vùng lãnh thổ hay quốc gia cụ thể.” Như vậy, chỉ dẫn địa
lý cho người tiêu dùng biết được hàng hóa đến từ khu vực nào với những nét đặc thù của khu
vực đó trong quá trình lựa chọn sản phẩm.
- Điều kiện bảo hộ đối với chỉ dẫn địa lý (Điều 79 Luật SHTT 2005) cần đáp ứng các điều kiện
sau:
“1. Sản phẩm mang chỉ dẫn địa lý có nguồn gốc địa lý từ khu vực, địa phương, vùng lãnh thổ
hoặc nước tương ứng với chỉ dẫn địa lý;

36
2. Sản phẩm mang chỉ dẫn địa lý có danh tiếng, chất lượng hoặc đặc tính chủ yếu do điều kiện
địa lý của khu vực, địa phương, vùng lãnh thổ hoặc nước tương ứng với chỉ dẫn địa lý đó quyết
định.”
- Chỉ dẫn địa lý là một dấu hiệu tuy nhiên dấu hiệu này phải thỏa mãn được các yêu cầu cụ thể:
tên gọi, biểu tượng hay hình ảnh được gắn chỉ dẫn địa lý phải có thật và chỉ thuộc về địa phương
đó. Hàng hóa, sản phẩm được gắn chỉ dẫn địa lý phải mang tính chất đặc thù về danh tiếng, chất
lượng có liên quan đến điều kiện địa lý, tự nhiên hoặc con người của địa phương đó. Đối với
những chỉ dẫn địa lý đạt được danh tiếng cao nhờ sự nổi tiếng về chất lượng và trở thành tài sản
có giá trị thương mại thì sự bảo hộ lại càng thiết yếu.
- Tính chất đặc thù về điều kiện tự nhiên và tính chất đặc thù về con người của môi trường địa lý
mang chỉ dẫn địa lý của hàng hóa, sản phẩm được quyết định hoàn toàn hoặc mức độ cơ bản ở
nơi mà chúng được sản xuất ra. VD: nước mắm Phú Quốc sử dụng nguyên liệu là cá cơm sống ở
biển thuộc khu vực này, kết hợp với nguồn nước, điều kiện ánh sáng, kỹ thuật quy trình sản xuất
truyền thống của người dân địa phương để làm ra nước mắm với những kinh nghiệm bao đời của
người dân sống trên đảo.
Đồng thời, luật cũng quy định về các đối tượng không được bảo hộ với danh nghĩa chỉ dẫn địa lý
tại Điều 80 như sau:
“Các đối tượng sau đây không được bảo hộ với danh nghĩa chỉ dẫn địa lý:
1.[24] Tên gọi, chỉ dẫn đã trở thành tên gọi chung của hàng hóa theo nhận thức của người tiêu
dùng có liên quan trên lãnh thổ Việt Nam;
2. Chỉ dẫn địa lý của nước ngoài mà tại nước đó chỉ dẫn địa lý không được bảo hộ, đã bị chấm
dứt bảo hộ hoặc không còn được sử dụng;
3.[25] Chỉ dẫn địa lý trùng hoặc tương tự với một nhãn hiệu đang được bảo hộ hoặc đã được
nộp theo đơn đăng ký nhãn hiệu có ngày nộp đơn hoặc ngày ưu tiên sớm hơn, nếu việc sử dụng
chỉ dẫn địa lý đó được thực hiện thì có khả năng gây nhầm lẫn về nguồn gốc thương mại của
hàng hóa;
4. Chỉ dẫn địa lý gây hiểu sai lệch cho người tiêu dùng về nguồn gốc địa lý thực của sản phẩm
mang chỉ dẫn địa lý đó.”
54. Phân tích các điều kiện bảo hộ bí mật kinh doanh

Theo quy định tại khoản 23 Điều 4 Luật sở hữu trí tuệ 2005 sửa đổi bổ sung 2019 quy định Bí
mật kinh doanh là thông tin thu được từ hoạt động đầu tư tài chính, trí tuệ, chưa được bộc lộ và
có khả năng sử dụng trong kinh doanh. Bí mật kinh doanh là đối tượng quyền sở hữu công
nghiệp, nên điều kiện để bí mật kinh doanh được bảo hộ theo quy định của pháp luật đó là:

Căn cứ theo quy định tại Điều 84. Điều kiện chung đối với bí mật kinh doanh được bảo hộ
Luật sở hữu trí tuệ 2005 sửa đổi bổ sung 2019 quy định:

Bí mật kinh doanh được bảo hộ nếu đáp ứng các điều kiện sau đây:

1. Không phải là hiểu biết thông thường và không dễ dàng có được;

2. Khi được sử dụng trong kinh doanh sẽ tạo cho người nắm giữ bí mật kinh doanh lợi thế so với

37
người không nắm giữ hoặc không sử dụng bí mật kinh doanh đó;

3. Được chủ sở hữu bảo mật bằng các biện pháp cần thiết để bí mật kinh doanh đó không bị bộc
lộ và không dễ dàng tiếp cận được. (Luật sở hữu trí tuệ)

+ Thứ nhất, không phải là hiểu biết thông thường hoặc không dễ dàng có được : Đây có thể
coi là điều kiện về tính sáng tạo của bí mật kinh doanh. Tri thức, thông tin chỉ được bảo hộ là bí
mật kinh doanh nếu nó là thành quả của một quá trình đầu tư tài chính và trí tuệ của chủ sở hữu.

+ Thứ hai, có giá trị thương mại : Tri thức, thông tin được bảo hộ là bí mật kinh doanh nếu nó
tạo ra cho người nắm giữ bí mật kinh doanh lợi thế hơn so với người không nắm giữ hoặc không
sử dụng bí mật kinh doanh đó. Để có được bí mật kinh doanh , doanh nghiệp phải đầu tư rất
nhiều tiền của, công sức để thu thập, phát triển, bảo mật bí mật kinh doanh và đổi lại, bí mật kinh
doanh tạo ra những giá trị kinh tế độc lập cho người nắm giữ nó.

+ Thứ ba, tính bảo mật : Bí mật kinh doanh phải được chủ sở hữu bảo mật bằng các biện pháp
cần thiết để thông tin đó không bị bộc lộ và không dễ dàng tiếp cận được. Để được bảo hộ là bí
mật kinh doanh , một trong những điều kiện quan trọng là thông tin còn được tồn tại trong tình
trạng bí mật. Một thông tin cũng được coi là có tính bí mật nếu như chỉ có một phạm vi hạn chế
những người trực tiếp sử dụng thông tin đó trong doanh nghiệp biết được thông tin và có trách
nhiệm giữ bí mật.

Theo quy định của pháp luật, bí mật kinh doanh được xác lập trên cơ sở có được một cách hợp
pháp và thực hiện việc bảo mật bí mật kinh doanh đó. Vì vậy nếu bí mật kinh doanh của bạn là
do bạn có được một cách hợp pháp, đáp ứng đủ điều kiện để được bảo hộ và bạn có những chính
sách để bảo mật thì bí mật kinh doanh của bạn được bảo hộ mà không cần đăng ký.

Đồng thời Luật sở hữu trí tuệ cũng quy định những đối tượng không được bảo hộ với danh nghĩa
bí mật kinh doanh tại Điều 85 như sau:

Điều 85. Đối tượng không được bảo hộ với danh nghĩa bí mật kinh doanh Luật sở hữu trí
tuệ 2005 sửa đổi bổ sung 2019 quy định:

Các thông tin bí mật sau đây không được bảo hộ với danh nghĩa bí mật kinh doanh:

1. Bí mật về nhân thân;

2. Bí mật về quản lý nhà nước;

3. Bí mật về quốc phòng, an ninh;

4. Thông tin bí mật khác không liên quan đến kinh doanh.

Vì vậy để được bảo hộ với bí mật kinh doanh phải là bí mật kinh doanh theo quy định và đáp
ứng được những điều kiện chung để bảo hộ đồng thời không thuộc những đối tượng không được
bảo hộ với danh nghĩa bí mật kinh doanh.

55. Phân tích các điều kiện bảo hộ kiểu dáng công nghiệp

38
Kiểu dáng công nghiệp là hình dáng bên ngoài của sản phẩm, được thể hiện bằng đường nét,
hình khối, màu sắc hoặc sự kết hợp những yếu tố này. Sản phẩm mang kiểu dáng công nghiệp
được hiểu là đồ vật, dụng cụ, thiết bị, phương tiện... thuộc mọi lĩnh vực, có kết cấu và chức năng
nhất định, được sản xuất và lưu thông độc lập.

Điều 63 Luật SHTT quy định kiểu dáng công nghiệp được bảo hộ nếu đáp ứng đủ các điều
kiện sau đây :

● Có tính mới;
● Có tính sáng tạo;
● Có khả năng áp dụng công nghiệp.

a, Về tính mới của kiểu dáng công nghiệp

Kiểu dáng công nghiệp được coi là có tính mới nếu kiểu dáng công nghiệp đó khác biệt đáng kể
với những kiểu dáng công nghiệp đã bị bộc lộ công khai dưới hình thức sử dụng, mô tả bằng văn
bản hoặc bất ký hình thức nào khác ở trong nước hoặc ở nước ngoài trước ngày nộp đơn hoặc
trước ngày ưu tiên nếu đơn đăng ký kiểu dáng công nghiệp được hưởng quyền ưu tiên. Cụ thể:

+ Nếu chỉ khác biệt về những đặc điểm tạo dáng không dễ dàng nhận biết, ghi nhớ và không thể
dùng phân biệt tổng thể hai kiểu dáng công nghiệp đó thì không được coi là khác biệt đáng kể.

+ Được coi là chưa bị bộc lộ công khai nếu chỉ có một số người có hạn được biết và có nghĩa vụ
giữ bí mật về kiểu dáng công nghiệp đó.

+ Được coi là không bị mất tính mới nếu được công bố trong các trường hợp được quy định tại
khoản 4, điều 66, Luật Sở hữu trí tuệ.

b, Về tính sáng tạo của kiểu dáng công nghiệp

Kiểu dáng công nghiệp được coi là có tính sáng tạo nếu căn cứ vào các kiểu dáng công nghiệp đã
được bộc lộ công khai dưới hình thức sử dụng, mô tả bằng văn bản hoặc bất kỳ hình thức nào
khác ở trong nước hoặc nước ngoài trước ngày nộp đơn hoặc trước ngày ưu tiên của đơn đăng ký
kiểu dáng công nghiệp trong trường hợp đơn được hưởng quyền ưu tiên, kiểu dáng công nghiệp
đó không thể được tạo ra một cách dễ dàng đối với người có hiểu biết trung bình về lĩnh vực
tương ứng.

c, Về khả năng áp dụng công nghiệp của kiểu dáng công nghiệp

Kiểu dáng công nghiệp được coi là có khả năng áp dụng công nghiệp nếu có thể dùng làm mẫu
để chế tạo hàng loạt sản phẩm có hình dáng bên ngoài là kiểu dáng công nghiệp đó bằng phương
pháp công nghiệp hoặc thủ công nghiệp.

56. Phân tích các đối tượng không được bảo hộ với danh nghĩa kiểu dáng công nghiệp?

Theo quy định tại Điều 64 Luật SHTT , những sản phẩm dưới đây sẽ không được bảo hộ kiểu
dáng công nghiệp:

39
– Sản phẩm có hình dáng bên ngoài do đặc tính kỹ thuật của sản phẩm bắt buộc phải có.

VD: hình tròn của bánh xe, hình xoắn ốc của đinh ốc, mặt phẳng của đĩa CD, hai đầu của chiếc
kim khâu,…

– Hình dáng bên ngoài của công trình xây dựng dân dụng hoặc công nghiệp

Đối tượng này không được bảo hộ kiểu dáng công nghiệp vì không có khả năng áp dụng công
nghiệp. Chúng ta sẽ không thể tạo ra hàng loạt những công trình xây dựng dân dụng hoặc công
nghiệp giống y hệt nhau từ hình dáng cho đến nguyên vật liệu, trang thiết bị, nội thất bên trong.
Bản vẽ thiết kế của các công trình xây dựng chỉ có giá trị thẩm mỹ và có thể được bảo hộ dưới
hình thức bản quyền tác giả.

VD: nhà xây, công trình cầu đường,…

Tuy nhiên, nếu sản phẩm là khung nhà có thể lắp ráp, di chuyển vẫn được bảo hộ dưới dạng kiểu
dáng công nghiệp.

– Hình dáng của sản phẩm không nhìn thấy được trong quá trình sử dụng sản phẩm

VD: hình dáng kem đánh răng, động cơ xe máy,…

– Sản phẩm có kiểu dáng chứa biểu tượng hoặc huy hiệu

VD: quốc kỳ, quốc huy của Việt Nam,…

– Sản phẩm có kiểu dáng trái với thuần phong mỹ tục.

57. Phân loại nhãn hiệu?

Nhãn hiệu là dấu hiệu dùng để phân biệt hàng hoá, dịch vụ của các tổ chức, cá nhân khác nhau.
Việc phân loại nhãn hiệu có thể căn cứ vào một số khía cạnh, cụ thể:

*Căn cứ vào dấu hiệu của nhãn hiệu

- Nhãn hiệu chữ : bao gồm những dấu hiệu là các từ, sự kết hợp giữa các chữ cái có thể
phát âm, cụm từ, câu, những đơn vị tiếng khác cũng như sự kết hợp giữa chúng. Những
dấu hiệu là từ ngữ phổ biến hơn do những đặc tính phân biệt có thể có trong ý nghĩa của
những từ ngữ.

VD: ADIDAS, NIKE, Puma,..

- Nhãn hiệu hình : bao gồm hình vẽ, ảnh chụp, biểu tượng, hình khối (hình không gian
ba chiều).

- Nhãn hiệu kết hợp : là nhãn hiệu có sự kết hợp cả cả từ ngữ và hình ảnh. Những nhãn
hiệu này có thể được thể hiện đen trắng hoặc kết hợp cả màu sắc.

*Căn cứ vào tính chất, chức năng của nhãn hiệu

40
- Nhãn hiệu dùng cho hàng hóa

Là những dấu hiệu để phân biệt hàng hóa của các cá nhân, tổ chức kinh doanh khác nhau. Nhãn
hiệu này có thể được gắn ngay trên chính hàng hóa hay trên bao bì của hàng hóa đó. Hàng hóa
được hiểu là những vật phẩm có nguồn gốc tự nhiên hay được sản xuất, chế tạo để bán. Các hàng
hóa đang lưu thông trên thị trường hiện nay chủ yếu là sản phẩm của lao động, còn những hàng
hóa có nguồn gốc tự nhiên thì không nhiều

- Nhãn hiệu dùng cho dịch vụ

Là những dấu hiệu để phân biệt dịch vụ của các cá nhân, tổ chức kinh doanh khác nhau. Dịch vụ
được hiểu là các hoạt động thực tế được thực hiện theo yêu cầu hay vì lợi ích của bên thuê dịch
vụ. Khái niệm dịch vụ được hiểu là những dịch vụ độc lập bao gôm một hay nhiều hành vi cụ thể
để thực hiện một yêu cầu nhất định, qua đó mang lại lợi ích của chủ thể phía bên kia.

Ví dụ: Việc sửa chữa đồ điện là một hoạt động dịch vụ nhưng việc sửa chữa các sản phẩm đồ
điện từ một quan hệ mua bán đã được thiết lập thông qua khâu bảo hành thì không phải là một
quan hệ dịch vụ.

Nhãn hiệu dịch vụ thường được gắn trên các bảng hiệu dịch vụ để người có nhu cầu hưởng dịch
vụ đó có thể dễ dàng nhận biết. Trong điều kiện đời sống kinh tế vật chất ngày càng được nâng
cao như hiện nay thì các loại hình dịch vụ ngày càng trở lên phong phú, tăng cả về số lượng, chất
lượng và mức độ cạnh tranh giữa các loại dịch vụ cũng trở nên gay gắt.

- Nhãn hiệu tập thể

Nhãn hiệu tập thể là nhãn hiệu dùng để phân biệt hàng hoá, dịch vụ của các thành viên của tổ
chức là chủ sở hữu nhãn hiệu đó với hàng hoá, dịch vụ của tổ chức, cá nhân không phải là thành
viên của tổ chức đó. (Khoản 17 Điều 4 Luật Sở hữu trí tuệ 2005)

- Nhãn hiệu chứng nhận

Nhãn hiệu chứng nhận là nhãn hiệu mà chủ sở hữu nhãn hiệu cho phép tổ chức, cá nhân khác sử
dụng trên hàng hóa, dịch vụ của tổ chức, cá nhân đó để chứng nhận các đặc tính về xuất xứ,
nguyên liệu, vật liệu, cách thức sản xuất hàng hoá, cách thức cung cấp dịch vụ, chất lượng, độ
chính xác, độ an toàn hoặc các đặc tính khác của hàng hoá, dịch vụ mang nhãn hiệu. (Khoản 18
Điều 4 Luật Sở hữu trí tuệ 2005)

- Nhãn hiệu liên kết

Nhãn hiệu liên kết là các nhãn hiệu do cùng một chủ thể đăng ký, trùng hoặc tương tự nhau dùng
cho sản phẩm, dịch vụ cùng loại hoặc tương tự nhau hoặc có liên quan với nhau.(Khoản 19 Điều
4 Luật Sở hữu trí tuệ 2005)

- Nhãn hiệu nổi tiếng

Nhãn hiệu nổi tiếng là nhãn hiệu được người tiêu dùng biết đến rộng rãi trên toàn lãnh thổ Việt
Nam. (Khoản 120 Điều 4 Luật Sở hữu trí tuệ 2005)

41
58. Phân tích khái niệm nhãn hiệu nổi tiếng và cơ chế bảo hộ đặc thù dành cho nhãn hiệu nổi
tiếng
* Khái niệm nhãn hiệu nổi tiếng
Căn cứ theo Khoản 20 Điều 4 Luật Sở hữu trí tuệ năm 2005, sửa đổi bổ sung năm 2009, 2019,
Nhãn hiệu nổi tiếng là nhãn hiệu được người tiêu dùng biết đến rộng rãi trên toàn lãnh thổ
Việt Nam. Thông qua quá trình kinh doanh, những nhãn hiệu này đã trở nên phổ biến và được
thừa nhận rộng rãi trên thị trường. Người tiêu dùng do quen thuộc với nhãn hiệu nổi tiếng nên
biết rõ các sản phẩm, dịch vụ này khi nhắc đến tổng quan về thương hiệu nổi tiếng.
* Cơ chế bảo hộ đặc thù dành cho nhãn hiệu nổi tiếng là gì?
Căn cứ Điểm a Khoản 3 Điều 6 Luật Sở hữu trí tuệ năm 2005, sửa đổi bổ sung năm 2009, 2019:
Quyền sở hữu đối với nhãn hiệu nổi tiếng được xác lập trên cơ sở sử dụng không phụ thuộc vào
thủ tục đăng ký nhãn hiệu tại Cục Sở hữu trí tuệ. Nhãn hiệu nổi tiếng sẽ có cơ chế bảo hộ đặc thù
mà không phải thông qua thủ tục đăng ký để được pháp luật bảo hộ. Tuy nhiên, để được bảo hộ
đặc thù, nhãn hiệu nổi tiếng cần đáp ứng đầy đủ theo các tiêu chí đánh giá của Luật Sở hữu trí
tuệ.
59. Phân tích các tiêu chí xác định nhãn hiệu nổi tiếng.
Tại Điều 75 Luật sở hữu trí tuệ 2005 quy định một số tiêu chí để đánh giá một nhãn hiệu nổi
tiếng bao gồm:
a) Số lượng người tiêu dùng liên quan đã biết đến nhãn hiệu thông qua việc mua bán, sử dụng
hàng hoá, dịch vụ mang nhãn hiệu hoặc thông qua quảng cáo
Một nhãn hiệu được nhiều người biết đến thông qua nhiều hoạt động khác nhau như mua bán, sử
dụng hoặc quảng cáo thì có thể thỏa mãn một trong các tiêu chí trở thành nhãn hiệu nổi tiếng
b) Phạm vi lãnh thổ mà hàng hóa, dịch vụ mang nhãn hiệu đó đã được lưu hành
Nhãn hiệu nổi tiếng thì không chỉ lưu hành ở trong một số địa phương nhất định. Trong một
quốc gia, nhãn hiệu cho hàng hóa, dịch vụ đó được lưu hành phổ biến, rộng rãi ở các địa phương
trên các nước, được nhiều người biết đến và công nhận.
c) Doanh số từ việc bán hàng hoá hoặc cung cấp dịch vụ mang nhãn hiệu hoặc số lượng hàng hoá
đã được bán ra, lượng dịch vụ đã được cung cấp
Điều này thể hiện việc bán ra hàng hóa và cung cấp dịch vụ. Một nhãn hiệu cho hàng hóa, dịch
vụ nổi tiếng thì sẽ được nhiều người quan tâm, mua sắm và cung cấp dịch vụ.. Doanh số bán ra
và cung cấp càng nhiều thì rất có thể càng chứng tỏ nhãn hiệu đó được nhiều người biết đến và
sử dụng.
d) Thời gian sử dụng liên tục nhãn hiệu
Một nhãn hiệu nếu có uy tín và phổ biến, được nhiều người ủng hộ sẽ có thể sẽ duy trì thời gian
sử dụng rất lâu. Trên thế giới, có những nhãn hiệu đã tồn tại hàng trăm năm và vẫn duy trì sự nổi
tiếng đó cho đến tận ngày nay. Một nhãn hiệu chỉ duy trì trong thời gian ngắn thì có thể chưa đủ
tin cậy và chỗ đứng trong thị trường.
e) Uy tín rộng rãi của hàng hóa, dịch vụ mang nhãn hiệu
Uy tín rộng rãi của nhãn hiệu được thể hiện qua việc sự lựa chọn của người tiêu dùng khi muốn
mua sắm hay dùng một dịch vụ nào đó. Ví dụ một người khi muốn mua tủ lạnh thì sẽ tin tưởng
và chọn những nhãn hiệu uy tín như LG, Samsung,… thay vì một nhãn hiệu non trẻ, chưa khẳng
42
định được uy tín.
f) Số lượng quốc gia bảo hộ nhãn hiệu
Nhãn hiệu nếu càng phổ biến thì quy mô phát triển càng rộng lớn, vượt ra ngoài phạm vi lãnh thổ
của một quốc gia.
g) Số lượng quốc gia công nhận nhãn hiệu là nổi tiếng
Nhãn hiệu nếu được nhiều quốc gia công nhận là nhãn hiệu nổi tiếng thì càng chứng minh được
sự phổ biến, rộng rãi của mình trên thị trường. Do đó trở thành tiêu chí để đánh giá nhãn hiệu nổi
tiếng
h) Giá chuyển nhượng, giá chuyển giao quyền sử dụng, giá trị góp vốn đầu tư của nhãn hiệu
Mọi sự đánh giá nên được thể hiện qua con số. Nhãn hiệu được định giá cao thì nhãn hiệu đó
mang lại lợi ích lớn cho người tiêu dùng.
60. So sánh nhãn hiệu và tên thương mại.

Về cơ bản, nhãn hiệu và tên thương mại là hai đối tượng hoàn toàn khác nhau. Tuy nhiên, do
những điểm giống nhau nhất định về mặt hình thức mà mọi người vẫn thường nhầm lẫn. Để có
thể so sánh một cách đầy đủ chúng ta cần phải căn cứ vào các quy định của pháp luật một cách
chặt chẽ nhất.

Giống nhau:

- Đều là các chỉ dẫn thương mại xuất hiện trên hàng hóa, giúp người tiêu dùng phân biệt.

- Phải là những dấu hiệu nhìn thấy được.

- Có khả năng phân biệt.

Khác nhau:

Tiêu chí Nhãn hiệu Tên thương mại

“ Nhãn hiệu là dấu hiệu dùng để phân “Tên thương mại là tên gọi của tổ chức,
biệt hàng hóa, dịch vụ của các tổ cá nhân dùng trong hoạt động kinh
Khái niệm chức , cá nhân khác nhau” – Khoản doanh để phân biệt chủ thể kinh doanh
16 Điều 4 Luật SHTT năm 2005 sửa mang tên đó với chủ thể kinh doanh
đổi bổ sung năm 2019. khác trong cùng lĩnh vực và khu vực
kinh doanh” – Khoản 21 Điều 4 Luật
SHTT sửa đổi bổ sung năm 2019.

-Đăng ký đối với nhãn hiệu thông


thường.
-Không đăng ký đối với nhãn hiệu

43
nổi tiếng .
Căn cứ xác Không cần đăng ký.
lập

- Có thể là những từ ngữ hình


ảnh, biểu tượng, là sự kết hợp Chỉ là dấu hiệu từ ngữ, không bảo hộ
giữa ngôn ngữ và hình ảnh. màu sắc, hình ảnh.
- Không bảo hộ những cụm từ,
dấu hiệu quy định tại khoản 2 Gồm 2 thành phần :
Dấu hiệu Điều 74 Luật SHTT. - Mô tả
- Phân biệt

Số lượng Một chủ thể kinh doanh có thể đăng Một chủ thể sản xuất kinh doanh chỉ có
ký sở hữu nhiều nhãn hiệu. thể có một tên thương mại.

Điều kiện Phải đăng ký và được cấp văn bằng Chỉ cần sử dụng hợp pháp tên thương
bảo hộ. mại.

Phạm vi Bảo hộ trên phạm vi toàn quốc Bảo hộ trong lĩnh vực và khu vực kinh
bảo hộ doanh.

Thời gian 10 năm , có thể gia hạn , mỗi lần gia Bảo hộ không xác định thời hạn, chấm
bảo hộ hạn là 10 năm. dứt khi không còn sử dụng.

Nhãn hiệu có thể là đối tượng của hợp Chỉ có thể là đối tượng của hợp đồng
đồng chuyển nhượng và hợp đồng chuyển nhượng với điều kiện là việc
Chuyển chuyển quyền sử dụng. chuyển nhượng tên thương mại kèm
giao theo việc chuyển nhượng toàn bộ cơ sở
sản xuất kinh doanh.

61. Phân tích khái niệm chỉ dẫn thương mại, phân biệt chỉ dẫn thương mại với nhãn hiệu và tên
thương mại.
*Khái niệm chỉ dẫn thương mại
Theo pháp luật Việt Nam khái niệm chỉ dẫn thương mại lần đầu tiên được quy định tại Nghị định
54/2000/NĐ-CP về bảo hộ quyền sở hữu công nghiệp đối với bí mật kinh doanh, chỉ dẫn địa lý,
tên thương mại và bảo hộ quyền chống cạnh tranh không lành mạnh liên quan đến sở hữu công
nghiệp:

“Chỉ dẫn thương mại” là các dấu hiệu, thông tin nhằm hướng dẫn thương mại, hàng hóa, dịch
vụ, gồm nhãn hiệu hàng hóa, tên thương mại, biểu tượng kinh doanh, khẩu hiệu kinh doanh, chỉ
dẫn địa lý, kiểu dáng bao bì của hàng hóa, nhãn hàng hóa,…” (khoản 1 Điều 4 Nghị định
54/2000).

44
Và đã được pháp điển hóa trong Luật Sở hữu trí tuệ năm 2005 được sửa đổi bổ sung năm 2019 .

*Phân biệt chỉ dẫn thương mại với nhãn hiệu và tên thương mại

Chỉ dẫn thương mại quy định tại khoản 2 Điều 130 của Luật Sở hữu trí tuệ gồm các đối tượng
sở hữu công nghiệp (nhãn hiệu, tên thương mại, chỉ dẫn địa lý) và các đối tượng sau đây:

+ "Nhãn hàng hóa" là bản viết, bản in, bản vẽ, bản chụp của chữ, hình vẽ, hình ảnh được
dán, in, đính, đúc, chạm, khắc trực tiếp trên hàng hóa, bao bì thương phẩm của hàng hoá
hoặc trên các chất liệu khác được gắn trên hàng hoá, bao bì thương phẩm của hàng hóa
thể hiện nội dung cơ bản, cần thiết về hàng hóa để người tiêu dùng nhận biết, làm căn cứ
lựa chọn, tiêu thụ và sử dụng; để nhà sản xuất, kinh doanh quảng bá cho hàng hoá của
mình và để các cơ quan chức năng thực hiện việc kiểm tra, kiểm soát;

+ "Khẩu hiệu kinh doanh" là một nhóm từ ngữ xuất hiện bên cạnh tên doanh nghiệp hoặc
nhãn hiệu của sản phẩm của doanh nghiệp nhằm nhấn mạnh mục đích hoặc tiêu chí kinh
doanh của doanh nghiệp hoặc đối tượng khách hàng mà sản phẩm hướng tới.

+ "Biểu tượng kinh doanh" là ký hiệu, chữ viết, hình vẽ, hình khối được thiết kế một cách
độc đáo và được coi là biểu tượng của doanh nghiệp sử dụng trong hoạt động kinh doanh;

+ “Kiểu dáng bao bì hàng hóa” là thiết kế, trang trí bao bì hàng hóa, gồm hình dạng, đường
nét, hình vẽ, chữ, số, màu sắc, cách trình bày, cách phối hợp màu sắc, cách bố trí, kết hợp
giữa các yếu tố nói trên tạo nên ấn tượng riêng hay nét đặc trưng của bao bì hàng hóa.
Như vậy , chỉ dẫn thương mại sẽ bao gồm cả nhãn hiệu và tên thương mại nên nó có đầy đủ đặc
điểm của nhãn hiệu và tên thương mại .

62. Trình bày chủ sở hữu đối tượng sở hữu công nghiệp?
Theo Luật SHTT 2019, Điều 121 “Chủ sở hữu đối tượng sở hữu công nghiệp”
Chủ sở hữu đối với các đối tượng sở hữu công nghiệp bao gồm:
● Chủ sở hữu sáng chế, kiểu dáng công nghiệp, thiết kế bố trí: Bao gồm các tổ chức, cá
nhân được cơ quan có thẩm quyền cấp văn bằng bảo hộ các đối tượng sở hữu công
nghiệp tương ứng.
● Chủ sở hữu nhãn hiệu: Tổ chức, cá nhân được cơ quan có thẩm quyền cấp văn bằng bảo
hộ nhãn hiệu hoặc có nhãn hiệu đã đăng ký quốc tế được cơ quan có thẩm quyền công
nhận hoặc có nhãn hiệu nổi tiếng.
● Chủ sở hữu tên thương mại: Tổ chức, cá nhân sử dụng hợp pháp tên thương mại đó
trong hoạt động kinh doanh.
● Chủ sở hữu bí mật kinh doanh: Bao gồm các tổ chức, cá nhân có được bí mật kinh
doanh hợp pháp và có biện pháp để bảo mật bí mật kinh doanh đó. Bí mật kinh doanh
mà bên làm thuê, bên thực hiện nhiệm vụ được giao có được trong khi thực hiện công
việc được thuê hoặc được giao thuộc quyền sở hữu của bên thuê hoặc bên giao việc, trừ
trường hợp các bên có thoả thuận khác.
● Chủ sở hữu chỉ dẫn địa lý của Việt Nam: Chủ sở hữu chỉ dẫn địa lý là Nhà nước Việt
Nam. Nhà nước trao quyền sử dụng chỉ dẫn địa lý cho tổ chức, cá nhân tiến hành việc
sản xuất sản phẩm mang chỉ dẫn địa lý tại địa phương tương ứng và đưa sản phẩm đó ra

45
thị trường. Nhà nước trực tiếp thực hiện quyền quản lý chỉ dẫn địa lý hoặc trao quyền
quản lý chỉ dẫn địa lý cho tổ chức đại diện quyền lợi của tất cả các tổ chức, cá nhân
được trao quyền sử dụng chỉ dẫn địa lý.
Ngoài ra chủ sở hữu các đối tượng sở hữu công nghiệp còn là người được chuyển giao quyền
sở hữu công nghiệp thông qua hợp đồng chuyển nhượng quyền sở hữu công nghiệp hoặc
thông qua nhận di sản thừa kế.

63. Phân tích nội dung các quyền của chủ sở hữu đối tượng sở hữu công nghiệp
Căn cứ Điều 123. Quyền của chủ sở hữu đối tượng sở hữu công nghiệp
1. Chủ sở hữu đối tượng sở hữu công nghiệp có các quyền tài sản sau đây:
a) Sử dụng, cho phép người khác sử dụng đối tượng sở hữu công nghiệp theo quy định tại
Điều 124 và Chương X của Luật này;
b) Ngăn cấm người khác sử dụng đối tượng sở hữu công nghiệp theo quy định tại Điều 125
của Luật này;
c) Định đoạt đối tượng sở hữu công nghiệp theo quy định tại Chương X của Luật này.
2. Tổ chức, cá nhân được Nhà nước trao quyền sử dụng, quyền quản lý chỉ dẫn địa lý theo
quy định tại khoản 4 Điều 121 của Luật này có các quyền sau đây:
a) Tổ chức được trao quyền quản lý chỉ dẫn địa lý có quyền cho phép người khác sử dụng chỉ
dẫn địa lý đó theo quy định tại điểm a khoản 1 Điều này;
b) Tổ chức, cá nhân được trao quyền sử dụng hoặc tổ chức được trao quyền quản lý chỉ dẫn
địa lý có quyền ngăn cấm người khác sử dụng chỉ dẫn địa lý đó theo quy định tại điểm b
khoản 1 Điều này.
Như vậy, ta có thể hiểu như sau:
❖ Quyền sử dụng đối với sở hữu công nghiệp
Sử dụng, đưa đối tượng sở hữu công nghiệp vào khai thác để thu được các lợi ích từ chúng mang
lại có thể được xem như một trong những quyển năng quan trọng nhất của chủ sở hữu đối tượng
sở hữu công nghiệp. Trên thực tế có rất nhiều cách thức khai thác khác nhau nhưng có thể đưa ra
những hành vi sử dụng chủ yếu sau đây đối với các đối tượng sở hữu công nghiệp:

Đối với sáng Đối với kiểu Đối với thiết kế Đối với bí mật
chế dáng công bố trí mạch kinh doanh
nghiệp

Hành vi tiến Là hành vi sản Là sản xuất sản Là sao chép Là sản xuất sản
hành sản xuất xuất sản phẩm có thiết kế phẩm,cung
các sản phẩm phẩm,áp dụng hình dáng bên bố trí,sản xuất ứng dịch
thuộc đối tượng quy trình được ngoài được bảo mạch tích hợp vụ,thương mại
sở hữu công cấp bằng độc hộ dưới danh bản dẫn theo hàng hoá theo
nghiệp quyền; nghĩa là kiểu thiết kế bố trí; các thông tin
dáng thuộc bí mật
công nghiệp; kinh doanh.

46
Hành vi khai Là việc khai Là gắn nhãn Là dùng tê Gắn chỉ dẫn địa
thác công dụng thác công dụng hiệu được bảo thương mại để lí được bảo hộ
của các đối của sản phẩm hộ lên hàng hóa, xưng danh trong lên hàng hóa,
tượng sở hữu được bảo hộ bao bì hàng hóa, các hoạt động bao bì hàng hóa,
công nghiệp hoặc sản phẩm phương tiện kinh doanh, thể phương tiện
được bảo hộ được sản xuất kinh doanh, hiện tên thương kinh doanh,
theo quy trình phương tiện mại trong các giấy tờ giao
được bảo hộ dịch vụ, giấy tờ giấy tờ giao dịch trong hoạt
giao dịch trong dịch, biển hiệu, động kinh
hoạt động kinh sản phẩm, hàng doanh.
doanh hóa, bao bì hàng
hóa là phương
tiện cung cấp
dịch vụ, quảng
cáo.

Hành vi lưu Là việc lưu Là bán, cho Là bán, quảng Là lưu thông,
thông thương thông, quảng thuê, quảng cáo, cáo để bán, tàng chào bán, quảng
mại, nhập khẩu cáo, chào hàng, chào hàng hoặc trữ để bán, nhập cáo để bán, tàng
các đối tượng tàng trữ để lưu tàng trữ hoặc khẩu sản phẩm trữ và nhập
sở hữu công thông sản phẩm nhập khẩu các được sản xuất khẩu hàng hóa
nghiệp được và nhập khẩu bản sao thiết kế do áp dụng bí có mang nhãn
bảo hộ các sản phẩm bố trí, mạch tích mật kinh doanh hiệu, chỉ dẫn địa
đó được bảo hộ hợp bán dẫn sản lí được bảo hộ.
theo sáng chế xuất theo thiết
và kiểu dáng kế bố trí hoặc
công nghiệp đó hàng hóa chứa
mạch tích hợp
bán dẫn sản
xuất theo thiết
kế bố trí được
bảo hộ

❖ Quyền tạm thời đối với các đối tượng sở hữu công nghiệp
Quyền tạm thời đối với đối tượng sở hữu công nghiệp là quyền của người đăng kí bảo hộ
các đối tượng đó được phát sinh từ ngày đơn đăng kí bảo hộ được nộp tại cơ quan nhà
nước có thẩm quyền hoặc được công bố công khai đến ngày được cấp bằng độc quyền
đối với các đối tượng đó.
Quyền tạm thời này phát sinh trên các điều kiện:
- Có việc sử dụng sáng chế, kiểu dáng công nghiệp, thiết kế bố trí nêu trong đơn
đăng kí đã nộp tại cơ quan nhà nước có thẩm quyền.
- Có thông báo bằng văn bản của người đã nộp đơn cho người sử dụng biết về việc
nộp đơn và việc đơn đã được công bố công khai đối với sáng chế, kiểu dáng công
nghiệp.

47
- Quyền này thực sự phát sinh khi ngời nộp đơn đã được cấp bằng độc quyền đối
tượng sở hữu công nghiệp
❖ Quyền chuyển quyền sử dụng đối tượng sở hữu công nghiệp
Trong thời gian bảo hộ, chủ sở hữu có thể chuyển quyền sử dụng cho đối tượng sở hữu
công nghiệp cho người khác tuy nhiên phải được thực hiện dưới hình thức kí kết hợp
đồng bằng văn bản phù hợp các quy định của pháp luật về hợp đồng dân sự, kinh tế.
❖ Quyền định đoạt đối tượng sở hữu công nghiệp
Có nhiều cách thức để chủ sở hữu thực hiện quyền định đoạt của mình đối với các đối
tượng sở hữu công nghiệp
- Thứ nhất: Chuyển toàn bộ quyền sở hữu của mình cho người khác
- Thứ hai: Từ bỏ quyền sở hữu của mình đối với các đối tượng sở hữu công nghiệp.
- Thứ ba: Chủ sở hữu các đối tượng sở hữu công nghiệp còn có thể để lại thừa kế cho
người khác
- Thứ tư: Dịch chuyển quyền theo sự sát nhập, hợp nhất, chia tách pháp nhân

64. Phân tích thời hạn bảo hộ quyền sở hữu công nghiệp
Thời hạn bảo hộ quyền sở hữu công nghiệp là thời hạn do pháp luật quy định trong đó Nhà nước
bảo hộ quyền của chủ sở hữu, quyền của tác giả đối tượng sở hữu công nghiệp đã được cấp văn
bằng bảo hộ. Thời hạn bảo hộ quyền sở hữu công nghiệp được tính từ ngày nộp đơn hợp lệ.

Có thể chia thời hạn bảo hộ thành ba loại:


– Thứ nhất, thời hạn bảo hộ được xác định và không được gia hạn. Loại thời hạn này được áp
dụng đối với sáng chế là 20 năm tính từ ngày nộp đơn hợp lệ; với giải pháp hữu ích là 10 năm,
với thiết kế bố trí mạch tích hợp là 10 năm kể từ ngày đăng ký hoặc ngày có quyền nộp đơn khai
thác, cho phép người khác khai thác thương mại tại bất kì nơi nào trên thế giới hoặc 15 năm tính
từ ngày tạo ra thiết kế bố trí; 20 năm đối với giống cây trồng tính từ ngày cấp; với kiểu dáng
công nghiệp là 5 năm tính từ ngày nộp đơn hợp lệ.
– Thứ hai, thời hạn bảo hộ được xác định và có thể gia hạn. Loại thời hạn này được áp dụng đối
với nhãn hiệu là 10 năm tính từ ngày nộp đơn hợp lệ, có thể gia hạn liên tiếp nhiều lần mỗi lần
10 năm. Đối với các đối tượng nêu trên, việc bảo hộ trong thời hạn xác định chỉ có hiệu lực khi
chủ sở hữu văn bằng bảo hộ nộp lệ phí duy trì hiệu lực của văn bằng.
+ Đối với các đối tượng nêu trên, việc bảo hộ trong thời hạn xác định chỉ có hiệu lực khi chủ
sở hữu văn bằng bảo hộ nộp lệ phí duy trì hiệu lực của văn bằng.
– Thứ ba, thời hạn bảo hộ không xác định. Loại thời hạn này được áp dụng đối với tên thương
mại, chỉ dẫn địa lí, bí mật kinh doanh cho đến khi nào còn đáp ứng được điều kiện bảo hộ.

65. Phân tích các trường hợp giới hạn quyền sở hữu công nghiệp
Quyền sở hữu công nghiệp của chủ sở hữu không phải là quyền tuyệt đối mà bị giới hạn bởi các
yếu tố được quy định cụ thể trong pháp luật sở hữu trí tuệ.
Các yếu tố hạn chế quyền sở hữu công nghiệp được ghi nhận tại Điều 132 Luật Sở hữu trí tuệ
năm 2005 sửa đổi bổ sung 2019 quy định: Quyền của người sử dụng trước đối với sáng chế, kiểu
dáng công nghiệp; Các nghĩa vụ của chủ sở hữu, bao gồm: trả thù lao cho tác giả sáng chế, kiểu
dáng công nghiệp, thiết kế bố trí và sử dụng sáng chế, nhãn hiệu; Chuyển giao quyền sử dụng
sáng chế theo quyết định của cơ quan nhà nước có thẩm quyền.
Chủ thể quyền sở hữu trí tuệ chỉ được thực hiện quyền của mình trong phạm vi và thời hạn bảo
hộ theo quy định của Luật Sở hữu trí tuệ 2019 quy định cụ thể. Với từng yếu tố trên, pháp luật đã
quy định chi tiết từ Điều 133 đến Điều 137 Luật Sở hữu trí tuệ 2019 tạo điều kiện cho các cơ
48
quan, tổ chức, cá nhân hiểu rõ phạm vi quyền và giới hạn của mình trong sở hữu công nghiệp.
– Chủ sở hữu phải chuyển giao quyền sử dụng sáng chế theo quyết định của cơ quan Nhà nước
trong trường hợp phục vụ vì mục đích công cộng, phi thương mại, quốc phòng an ninh, phòng
bệnh, chữa bệnh, dinh dưỡng cho nhân dân và đáp ứng các nhu cầu cấp thiết khác của xã hội.
– Quyền sử dụng trước đối với sáng chế, kiểu dáng công nghiệp được quy định tại Điều 134 Luật
Sở hữu trí tuệ: Trường hợp trước ngày đơn đăng ký sáng chế, kiểu dáng công nghiệp được công
bố mà có người đã sử dụng hoặc chuẩn bị các điều kiện cần thiết để sử dụng sáng chế, kiểu dáng
công nghiệp đồng nhất với sáng chế, kiểu dáng công nghiệp trong đơn đăng ký nhưng được tạo
ra một cách độc lập (sau đây gọi là người có quyền sử dụng trước) thì sau khi văn bằng bảo hộ
được cấp, người đó có quyền tiếp tục sử dụng sáng chế, kiểu dáng công nghiệp trong phạm vi và
khối lượng đã sử dụng hoặc đã chuẩn bị để sử dụng mà không phải xin phép hoặc trả tiền đền bù
cho chủ sở hữu sáng chế, kiểu dáng công nghiệp được bảo hộ. Việc thực hiện quyền của người
sử dụng trước sáng chế, kiểu dáng công nghiệp không bị coi là xâm phạm quyền của chủ sở hữu
sáng chế, kiểu dáng công nghiệp. Theo quy định của pháp luật người có quyền sử dụng trước
sáng chế và các kiểu dáng công nghiệp không được phép chuyển giao quyền đó cho người khác,
chỉ trừ trường hợp chuyển giao quyền đó kèm theo việc chuyển giao cơ sở sản xuất, kinh doanh
nơi sử dụng hoặc chuẩn bị sử dụng sáng chế, kiểu dáng công nghiệp.

– Chủ sở hữu có nghĩa vụ trả thù lao cho tác giả sáng chế, kiểu dáng công nghiệp, thiết kế bố trí
với mức thù lao được quy định tại Điều 135 Luật Sở hữu trí tuệ, cụ thể: 10% số tiền làm lợi mà
chủ sở hữu thu được do sử dụng sáng chế, kiểu dáng công nghiệp, thiết kế bố trí; 15% tổng số
tiền mà chủ sở hữu nhận được trong mỗi lần nhận tiền thanh toán do chuyển giao quyền sử dụng
sáng chế, kiểu dáng công nghiệp, thiết kế bố trí. Với trường hợp có đồng tác giả thì chủ sở hữu
trả mức thù lao quy định như trên cho tất cả các đồng tác giả, các đồng tác giả tự thỏa thuận việc
phân chia tiền thù lao do chủ sở hữu chi trả. Nghĩa vụ này tồn tại trong suốt thời gian bảo hộ.
– Đồng thời, chủ sở hữu còn có nghĩa vụ sử dụng sáng chế, nhãn hiệu nhằm đáp ứng nhu cầu an
ninh quốc phòng, phòng bệnh, chữa bệnh, nhu cầu cấp thiết khác của xã hội. Chủ sở hữu phải sử
dụng liên tục nhãn hiệu đó, nếu không được sử dụng liên tục trong năm năm trở lên thì quyền sở
hữu nhãn hiệu sẽ bị chấm dứt.
– Ngoài ra, chủ sở hữu còn có nghĩa vụ cho phép người khác sử dụng sáng chế cơ bản để phục
vụ cho việc sử dụng sáng chế phụ thuộc trong trường hợp sáng chế phụ thuộc được coi là bước
tiến quan trọng về mặt kĩ thuật so với sáng chế cơ bản và có ý nghĩa kinh tế lớn. Nếu chủ sở hữu
không đáp ứng yêu cầu mà không có lí do chính đáng thì cơ quan Nhà nước có thẩm quyền có
thể chuyển giao quyền sử dụng sáng chế đó cho chủ sở hữu sáng chế phụ thuộc.
66. Phân tích khái niệm nhập khẩu song song và quy định của Luật Sở hữu trí tuệ Việt
Nam về nhập khẩu song song?
Theo Thông tư 37/2011/TT-BKHCN Hướng dẫn thi hành một số điều của Nghị định số
97/2010/NĐ-CP ngày 21/9/2010 của Chính phủ quy định xử phạt vi phạm hành chính trong lĩnh
vực sở hữu công nghiệp thì nhập khẩu song song được định nghĩa như sau:
Điều 10. Nhập khẩu song song
1. Nhập khẩu song song theo quy định tại tại khoản 2 Điều 28 của Nghị định 97/2010/NĐ-CP là
việc tổ chức, cá nhân nhập khẩu sản phẩm do chính chủ sở hữu hoặc tổ chức, cá nhân được
chuyển giao quyền sử dụng, kể cả chuyển giao quyền sử dụng theo quyết định bắt buộc, người có
quyền sử dụng trước đối tượng sở hữu công nghiệp đã đưa ra thị trường nước ngoài, mặc dù
không được sự đồng ý của chủ thể quyền sở hữu công nghiệp.

49
Ví dụ: Nhà sản xuất A sản xuất sản phẩm X, sản phẩm đã được bảo hộ quyền sở hữu công
nghiệp ở nước C1, sản phẩm đã được cấp số đăng ký và đang được bán trên thị trường của nước
C1 với giá P1. Sản phẩm X này của nhà sản xuất A cũng lại đang được bán ở một nước khác là
C2 với giá P2. Nhà nhập khẩu của nước C1 có thể nhập khẩu sản phẩm X này từ nước C2 về bán
ở nước C1 với giá P3, điều kiện là P3 thấp hơn giá P1.
67. Phân tích nguyên tắc nộp đơn đầu tiên trong xác lập quyền sở hữu công nghiệp?
Nguyên tắc này được quy định cụ thể tại điều 90 Luật SHTT 2019 (dài quá nên em trích sang
bên cạnh cho mn đỡ choáng)
Trong trường hợp có nhiều đơn đăng ký theo quy định nêu trên cùng đáp ứng các điều kiện để
được cấp văn bằng bảo hộ và cùng có ngày ưu tiên hoặc ngày nộp đơn sớm nhất thì văn bằng bảo
hộ chỉ được cấp cho đối tượng của một đơn duy nhất trong số các đơn đó theo thoả thuận của tất
cả những người nộp đơn; nếu không thoả thuận được thì các đối tượng tương ứng của các đơn đó
bị từ chối cấp văn bằng bảo hộ.
Theo đó, pháp luật Sở hữu trí tuệ tại VN quy định ngày ưu tiên là ngày nộp đơn hợp lệ tại Cục
Sở hữu trí tuệ hay tại một nước thành viên khác của điều ước quốc tế mà Việt Nam là thành viên
trước khi nộp đơn tại Cục Sở hữu trí tuệ trong thời hạn nhất định. Ngày ưu tiên mang ý nghĩa
quan trọng, bởi đây là căn cứ để cấp văn bằng bảo hộ nếu có 02 đơn cùng đăng ký bảo hộ cho
cùng kiểu dáng công nghiệp.
Có thể thấy, nguyên tắc nộp đơn đầu tiên mang ý nghĩa hết sức quan trọng, chẳng hạn:
- Thúc đẩy việc nộp đơn nhanh chóng và có hiệu quả;
- Đẩy mạnh các hoạt động nghiên cứu, sáng tạo;
- Nâng cao ý thức của chủ thể nắm giữ các đối tượng sở hữu công nghiệp để đạt được nhiều
quyền lợi tốt hơn, tránh trường hợp bị người khác trùng hay do chậm trễ mà ý tưởng thuộc
về người khác;
- Giúp cơ quan nhà nước dễ dàng trong việc quyết định chủ thể nào được cấp văn bằng và
cũng ít nảy sinh tranh chấp hơn.

68. Phân tích nguyên tắc ưu tiên trong xác lập quyền sở hữu công nghiệp?
Được quy định cụ thể tại Điều 91 Luật SHTT 2019
Quyền ưu tiên sẽ được áp dụng khi mà có từ 2 cá nhân, tổ chức trở lên cùng tiến hành đăng ký
bảo hộ cho một đối tượng bảo hộ tương tự nhau hoặc là trùng nhau. Tùy vào quy định của từng
quốc gia, mà việc áp dụng quyền ưu tiên sẽ khác nhau.
Lưu ý :
Trong một đơn đăng ký sáng chế, kiểu dáng công nghiệp hoặc nhãn hiệu, người nộp đơn có
quyền yêu cầu hưởng quyền ưu tiên trên cơ sở nhiều đơn khác nhau được nộp sớm hơn với điều
kiện phải chỉ ra nội dung tương ứng giữa các đơn nộp sớm hơn ứng với nội dung trong đơn.
Đơn đăng ký sở hữu công nghiệp được hưởng quyền ưu tiên có ngày ưu tiên là ngày nộp đơn của
đơn đầu tiên.
Trong đó, theo quy định tại Điều 10 Nghị định 103/2006/NĐ-CP, quyền ưu tiên đối với đơn
đăng ký sáng chế, kiểu dáng công nghiệp, nhãn hiệu sẽ được áp dụng như sau:
- Trong trường hợp người nộp đơn đăng ký sáng chế, kiểu dáng công nghiệp, nhãn hiệu muốn
50
hưởng quyền ưu tiên theo quy định của Công ước Paris, yêu cầu hưởng quyền ưu tiên của người
nộp đơn sẽ được chấp nhận nếu đáp ứng các điều kiện sau đây:
● Người nộp đơn là công dân Việt Nam hoặc công dân của nước Thành viên của Công ước
Paris hoặc cư trú, có cơ sở sản xuất, kinh doanh tại Việt Nam hoặc tại nước Thành viên
Công ước đó;
● Đơn đầu tiên đã được nộp tại Việt Nam hoặc tại nước Thành viên của Công ước Paris và
đơn đó có chứa phần tương ứng với yêu cầu hưởng quyền ưu tiên của đơn đăng ký sáng
chế, kiểu dáng công nghiệp, nhãn hiệu;
● Đơn đăng ký được nộp trong thời hạn sau đây kể từ ngày nộp đơn đầu tiên: sáu tháng đối
với đơn đăng ký kiểu dáng công nghiệp hoặc đơn đăng ký nhãn hiệu, mười hai tháng đối
với đơn đăng ký sáng chế;
● Trong đơn đăng ký sáng chế, kiểu dáng công nghiệp, nhãn hiệu, người nộp đơn có nêu rõ
yêu cầu hưởng quyền ưu tiên và có nộp bản sao đơn đầu tiên nêu tại điểm b khoản này
trong trường hợp nộp tại nước ngoài, trong đó có xác nhận của Cơ quan nhận đơn đầu
tiên;
● Nộp đủ lệ phí yêu cầu hưởng quyền ưu tiên.

- Trong trường hợp người nộp đơn đăng ký sáng chế, kiểu dáng công nghiệp, nhãn hiệu muốn
hưởng quyền ưu tiên theo điều ước quốc tế khác, yêu cầu hưởng quyền ưu tiên sẽ được chấp
nhận nếu đáp ứng các điều kiện về quyền ưu tiên quy định trong điều ước đó.
Nguyên tắc ưu tiên cũng mang ý nghĩa quan trọng trong việc đảm bảo quyền và lợi ích của chủ
sở hữu cũng như tác giả được bảo hộ một cách công bằng, cụ thể:
● Là căn cứ xác định điều kiện bảo hộ của sáng chế, nhãn hiệu hay kiểu dáng công nghiệp;
● Có quyền ưu tiên là một lợi thế khi chủ sỡ hữu muốn thực hiện quyền sở hữu trí tuệ của
mình tại quốc gia khác (bảo hộ quyền sở hữu trí tuệ chỉ có hiệu lực trong phạm vi lãnh
thổ quốc gia đăng ký).
● Áp dụng nguyên tắc ưu tiên tạo điều kiện cho việc bảo vệ sáng chế, kiểu dáng công
nghiệp và nhãn hiệu không chỉ ở trong nước mà còn mở rộng ra các quốc gia khác; góp
phần hạn chế các hành vi xâm phạm quyền đối với các đối tượng nói trên.
● Giải quyết các tranh chấp cũng như bồi thường thiệt hại khi có hành vi xâm phạm nếu
việc giải quyết có liên quan đến ngày ưu tiên.

69. Trình bày chủ thể có quyền đăng ký sáng chế, kiểu dáng công nghiệp, thiết kế bố trí?
Theo quy định của Luật SHTT 2019 và Nghị định 103/2006/NĐ-CP quy định, có 03
nhóm đối tượng có quyền đăng ký sáng chế, kiểu dáng công nghiệp và thiết kế bố trí, cụ thể:
Thứ nhất, là Tổ chức, cá nhân có quyền đăng ký sáng chế, kiểu dáng công nghiệp và thiết kế bố
trí. Quy định chi tiết tại khoản 1 điều 86 Luật SHTT 2019
Điều 86. Quyền đăng ký sáng chế, kiểu dáng công nghiệp, thiết kế bố trí
1. Tổ chức, cá nhân sau đây có quyền đăng ký sáng chế, kiểu dáng công nghiệp, thiết kế bố trí:
a) Tác giả tạo ra sáng chế, kiểu dáng công nghiệp, thiết kế bố trí bằng công sức và chi phí của
mình;
b) Tổ chức, cá nhân đầu tư kinh phí, phương tiện vật chất cho tác giả dưới hình thức giao việc,
thuê việc, trừ trường hợp các bên có thỏa thuận khác và thỏa thuận đó không trái với quy định
tại khoản 2 Điều này.

51
Thứ hai, Quyền đăng ký sáng chế, kiểu dáng công nghiệp, thiết kế bố trí thuộc về Nhà nước
Thứ ba, Nhiều tổ chức, cá nhân cùng nhau tạo ra hoặc đầu tư để để tạo ra sáng chế, kiểu dáng
công nghiệp và thiết kế bố trí. Quy định chi tiết tại Diều 9 Nghị định 103/2006 NĐ-CP
Đối với trường hợp có nhiều tổ chức, cá nhân cùng nhau tạo ra hoặc đầu tư để tạo ra sáng chế,
kiểu dáng công nghiệp, thiết kế bố trí thì các tổ chức, cá nhân đó đều có quyền đăng ký và quyền
đăng ký đó chỉ được thực hiện nếu được tất cả các tổ chức, cá nhân đó đồng ý.
Lưu ý: Đối với (nhiều) tổ chức, cá nhân có quyền đăng ký theo quy định của Luật SHTT sẽ có
quyền chuyển giao quyền đăng ký cho tổ chức, cá nhân khác dưới hình thức hợp đồng bằng văn
bản, để thừa kế hoặc kế thừa theo quy định của pháp luật, kể cả trường hợp đã nộp đơn đăng ký.
70. Trình bày chủ thể có quyền đăng ký nhãn hiệu?
Căn cứ Điều 87 Luật Sở hữu trí tuệ năm 2005, sửa đổi bổ sung năm 2009 quy định:
1. Tổ chức, cá nhân có quyền đăng ký nhãn hiệu dùng cho hàng hoá do mình sản xuất hoặc dịch
vụ do mình cung cấp.
2. Tổ chức, cá nhân tiến hành hoạt động thương mại hợp pháp có quyền đăng ký nhãn hiệu cho
sản phẩm mà mình đưa ra thị trường nhưng do người khác sản xuất với điều kiện người sản xuất
không sử dụng nhãn hiệu đó cho sản phẩm và không phản đối việc đăng ký đó.
3. Tổ chức tập thể được thành lập hợp pháp có quyền đăng ký nhãn hiệu tập thể để các thành
viên của mình sử dụng theo quy chế sử dụng nhãn hiệu tập thể; đối với dấu hiệu chỉ nguồn gốc
địa lý của hàng hóa, dịch vụ, tổ chức có quyền đăng ký là tổ chức tập thể của các tổ chức, cá
nhân tiến hành sản xuất, kinh doanh tại địa phương đó; đối với địa danh, dấu hiệu khác chỉ
nguồn gốc địa lý đặc sản địa phương của Việt Nam thì việc đăng ký phải được cơ quan nhà nước
có thẩm quyền cho phép.
4. Tổ chức có chức năng kiểm soát, chứng nhận chất lượng, đặc tính, nguồn gốc hoặc tiêu chí
khác liên quan đến hàng hóa, dịch vụ có quyền đăng ký nhãn hiệu chứng nhận với điều kiện
không tiến hành sản xuất, kinh doanh hàng hóa, dịch vụ đó; đối với địa danh, dấu hiệu khác chỉ
nguồn gốc địa lý đặc sản địa phương của Việt Nam thì việc đăng ký phải được cơ quan nhà nước
có thẩm quyền cho phép.
5. Hai hoặc nhiều tổ chức, cá nhân có quyền cùng đăng ký một nhãn hiệu để trở thành đồng chủ
sở hữu với những điều kiện sau đây:
a) Việc sử dụng nhãn hiệu đó phải nhân danh tất cả các đồng chủ sở hữu hoặc sử dụng cho hàng
hoá, dịch vụ mà tất cả các đồng chủ sở hữu đều tham gia vào quá trình sản xuất, kinh doanh;
b) Việc sử dụng nhãn hiệu đó không gây nhầm lẫn cho người tiêu dùng về nguồn gốc của hàng
hoá, dịch vụ.
6. Người có quyền đăng ký quy định tại các khoản 1, 2, 3, 4 và 5 trên này, kể cả người đã nộp
đơn đăng ký có quyền chuyển giao quyền đăng ký cho tổ chức, cá nhân khác dưới hình thức hợp
đồng bằng văn bản, để thừa kế hoặc kế thừa theo quy định của pháp luật với điều kiện các tổ
chức, cá nhân được chuyển giao phải đáp ứng các điều kiện đối với người có quyền đăng ký
tương ứng.
7. Đối với nhãn hiệu được bảo hộ tại một nước là thành viên của điều ước quốc tế có quy định
cấm người đại diện hoặc đại lý của chủ sở hữu nhãn hiệu đăng ký nhãn hiệu đó mà Cộng hoà xã
hội chủ nghĩa Việt Nam cũng là thành viên thì người đại diện hoặc đại lý đó không được phép

52
đăng ký nhãn hiệu nếu không được sự đồng ý của chủ sở hữu nhãn hiệu, trừ trường hợp có lý do
chính đáng.
Như vậy, những chủ thể có quyền đăng ký nhãn hiệu gồm: Tổ chức, cá nhân; Tổ chức, cá
nhân tiến hành hoạt động thương mại hợp pháp; Tổ chức tập thể được thành lập hợp pháp; Tổ
chức có chức năng kiểm soát, chứng nhận chất lượng, đặc tính, nguồn gốc hoặc tiêu chí khác liên
quan đến hàng hóa, dịch vụ; Hai hoặc nhiều tổ chức, cá nhân; Người có quyền đăng ký quy định
tại các khoản 1, 2, 3, 4 và 5;
71. Trình bày chủ thể có quyền đăng ký chỉ dẫn địa lý?
- Theo khoản 22 Điều 4 Luật Sở hữu trí tuệ 2005 (được sửa đổi bổ sung năm 2009, 2019) (sau
đây gọi là Luật Sở hữu trí tuệ) thì chỉ dẫn địa lý là dấu hiệu dùng để chỉ sản phẩm có nguồn
gốc từ khu vực, địa phương, vùng lãnh thổ hay quốc gia cụ thể.
- Chủ sở hữu chỉ dẫn địa lý của Việt Nam là Nhà nước. Nhà nước trao quyền sử dụng chỉ dẫn
địa lý cho tổ chức, cá nhân tiến hành việc sản xuất sản phẩm mang chỉ dẫn địa lý tại địa
phương tương ứng và đưa sản phẩm đó ra thị trường. Nhà nước trực tiếp thực hiện quyền
quản lý chỉ dẫn địa lý hoặc trao quyền quản lý chỉ dẫn địa lý cho tổ chức đại diện quyền lợi
của tất cả các tổ chức, cá nhân được trao quyền sử dụng chỉ dẫn địa lý.
- Như vậy, chủ thể có quyền đăng ký chỉ dẫn địa lý là tổ chức, cá nhân tiến hành việc sản xuất
sản phẩm mang chỉ dẫn địa lý tại địa phương tương ứng và đưa sản phẩm đó ra thị trường.
* Sử dụng chỉ dẫn địa lý là việc thực hiện các hành vi sau đây:
- Gắn chỉ dẫn địa lý được bảo hộ lên hàng hóa, bao bì hàng hóa, phương tiện kinh doanh,
giấy tờ giao dịch trong hoạt động kinh doanh;
- Lưu thông, chào bán, quảng cáo nhằm để bán, tàng trữ để bán hàng hóa có mang chỉ dẫn
địa lý được bảo hộ;
- Nhập khẩu hàng hóa có mang chỉ dẫn địa lý được bảo hộ.
(Khoản 4 Điều 121, khoản 7 Điều 124 Luật Sở hữu trí tuệ)
72. Trình bày khái quát quy trình đăng ký xác lập quyền sở hữu công nghiệp?
- Cơ sở pháp lý: Thông tư 01/2007/TT-BKHCN ngày 14/01/2007 hướng dẫn NĐ
103/2006/NĐ-CP
- Kiểu dáng công nghiệp là hình dáng bên ngoài của sản phẩm, được thể hiện bằng đường
nét, hình khối, màu sắc hoặc sự kết hợp những yếu tố này. Sản phẩm mang kiểu dáng
công nghiệp được hiểu là đồ vật, dụng cụ, thiết bị, phương tiện... thuộc mọi lĩnh vực, có
kết cấu và chức năng nhất định, được sản xuất và lưu thông độc lập
* Những tài liệu cần cung cấp để đăng ký chỉ dẫn địa lý:
- File tác phẩm (bản mô tả, ảnh chụp tác phẩm...);
- Bản sao y Giấy phép đăng ký kinh doanh;
- Thông tin cụ thể về tác giả: Nơi thường trú, Số điện thoại, Email, Fax;
- Thông tin liên quan đến tác phẩm: Tên tác phẩm, đã công bố hay chưa công bố, thời gian
công bố.
* Những công việc sẽ thực hiện:
- Soạn thảo hồ sơ liên quan đến việc đăng ký bảo hộ kiểu dáng công nghiệp;
- Nộp hồ sơ đăng ký bảo hộ kiểu dáng công nghiệp tới Cục sở hữu trí tuệ.
- Theo dõi, giải quyết các vấn đề phát sinh trong quá trình thực hiện đăng ký bảo hộ kiểu
53
dáng công nghiệp
* Thời hạn giải quyết của Cục sở hữu trí tuệ:
- Thẩm định hình thức: 01 tháng từ ngày nhận đơn;
- Công bố đơn: 02 tháng kể từ ngày chấp nhận đơn hợp lệ;
- Thẩm định nội dung: 06 tháng từ ngày công bố đơn.
* Nộp lệ phí nhà nước:
- Lệ phí nộp đơn (cho mỗi phương án)
+ Tài liệu đơn dạng giấy 180.000
+ Đơn kèm tài liệu điện tử mang toàn bộ nội dung tài liệu đơn 150.000
- Lệ phí yêu cầu hưởng quyền ưu tiên (mỗi đơn/yêu cầu) 600.000
- Lệ phí công bố đơn 120.000
- Phí thẩm định nội dung (cho mỗi phương án của từng sản phẩm) 300.000
- Phí tra cứu thông tin phục vụ việc thẩm định nội dung (cho mỗi phương án sản phẩm)
120.000
- Lệ phí đăng bạ Bằng độc quyền KDCN 120.000
- Lệ phí cấp Bằng độc quyền KDCN 120.000
- Lệ phí công bố Bằng độc quyền KDCN 120.000
- Lệ phí gia hạn hiệu lực 540.000
73. Phân tích các trường hợp chấm dứt hiệu lực của văn bằng bảo hộ quyền sở hữu công
nghiệp?
- Văn bằng bảo hộ bị chấm dứt hiệu lực trong các trường hợp sau:
+ Chủ văn bằng bảo hộ không nộp lệ phí duy trì hiệu lực hoặc gia hạn hiệu lực theo
quy định;
+ Chủ văn bằng bảo hộ tuyên bố từ bỏ quyền sở hữu công nghiệp. (Trong trường
hợp này, cơ quan quản lý nhà nước về quyền sở hữu công nghiệp quyết định chấm
dứt hiệu lực văn bằng kể từ ngày nhận được tuyên bố của chủ văn bằng bảo hộ.)
- Tổ chức, cá nhân có quyền yêu cầu cơ quan quản lý nhà nước về quyền sở hữu công
nghiệp chấm dứt hiệu lực văn bằng đối với các trường hợp dưới đây với điều kiện phải
nộp phí và lệ phí:
+ Chủ văn bằng bảo hộ không còn tồn tại hoặc chủ Giấy chứng nhận đăng ký nhãn
hiệu không còn hoạt động kinh doanh mà không có người kế thừa hợp pháp;
+ Nhãn hiệu không được chủ sở hữu hoặc người được chủ sở hữu cho phép sử dụng
trong thời hạn năm năm liên tục trước ngày có yêu cầu chấm dứt hiệu lực mà
không có lý do chính đáng, trừ trường hợp việc sử dụng được bắt đầu hoặc bắt
đầu lại trước ít nhất ba tháng tính đến ngày có yêu cầu chấm dứt hiệu lực;
+ Chủ Giấy chứng nhận đăng ký nhãn hiệu đối với nhãn hiệu tập thể không kiểm
soát hoặc kiểm soát không có hiệu quả việc thực hiện quy chế sử dụng nhãn hiệu
tập thể;
+ Chủ Giấy chứng nhận đăng ký nhãn hiệu đối với nhãn hiệu chứng nhận vi phạm
quy chế sử dụng nhãn hiệu chứng nhận hoặc không kiểm soát, kiểm soát không có
hiệu quả việc thực hiện quy chế sử dụng nhãn hiệu chứng nhận;
+ Các điều kiện địa lý tạo nên danh tiếng, chất lượng, đặc tính của sản phẩm mang
chỉ dẫn địa lý bị thay đổi làm mất danh tiếng, chất lượng, đặc tính của sản phẩm
đó.
+ Căn cứ kết quả xem xét đơn yêu cầu và ý kiến của các bên liên quan, cơ quan
54
quản lý nhà nước về quyền sở hữu công nghiệp quyết định chấm dứt hiệu lực văn
bằng hoặc thông báo từ chối chấm dứt hiệu lực văn bằng.
74. Trình bày về tác giả và quyền của tác giả sáng chế, kiểu dáng công nghiệp, thiết
kế bố trí?
- Theo quy định tại Điều 122 Luật Sở hữu trí tuệ 2005 thì tác giả và quyền của tác giả sáng
chế, kiểu dáng công nghiệp, thiết kế bố trí được quy định cụ thể như sau:
- Tác giả sáng chế, kiểu dáng công nghiệp, thiết kế bố trí là người trực tiếp sáng tạo ra đối
tượng sở hữu công nghiệp; trong trường hợp có hai người trở lên cùng nhau trực tiếp
sáng tạo ra đối tượng sở hữu công nghiệp thì họ là đồng tác giả.
- Quyền nhân thân của tác giả sáng chế, kiểu dáng công nghiệp, thiết kế bố trí gồm các
quyền sau đây:
+ Được ghi tên là tác giả trong Bằng độc quyền sáng chế, Bằng độc quyền giải pháp hữu
ích, Bằng độc quyền kiểu dáng công nghiệp và Giấy chứng nhận đăng ký thiết kế bố trí
mạch tích hợp bán dẫn;
+ Được nêu tên là tác giả trong các tài liệu công bố, giới thiệu về sáng chế, kiểu dáng
công nghiệp, thiết kế bố trí.
- Quyền tài sản của tác giả sáng chế, kiểu dáng công nghiệp, thiết kế bố trí là quyền nhận
thù lao theo quy định tại Điều 135 của Luật Sở hữu trí tuệ 2005.
75. Phân tích quyền của người sử dụng trước sáng chế, kiểu dáng công nghiệp?
- Quyền sử dụng trước đối tượng sở hữu công nghiệp được pháp luật đặt ra đối với sáng
chế và kiểu dáng công nghiệp. Việc đặt ra quy định này nhằm giải quyết tình huống nhiều
người cùng nghiên cứu và đều tìm ra các giải pháp kỹ thuật để giải quyết một vấn đề
chung nhưng một trong số họ không đăng ký bảo hộ, trong khi đó người khác nộp đơn
đăng ký bảo hộ cho cùng một sáng chế, kiểu dáng công nghiệp của họ và được ghi nhận
là chủ sở hữu. Trong trường hợp đó, quyền lợi của những người đã tìm ra sáng chế, kiểu
dáng công nghiệp trước ngày công bố đơn và đã đưa các đối tượng đó vào sử dụng trong
thực tế được pháp luật bảo hộ.
- Cụ thể, pháp luật quy định về quyền sử dụng trước tại Điều 134 Luật Sở hữu trí tuệ 2005
như sau:
- “Trường hợp trước ngày nộp đơn hoặc ngày ưu tiên (nếu có) của đơn đăng ký sáng chế,
kiểu dáng công nghiệp mà có người đã sử dụng hoặc chuẩn bị các điều kiện cần thiết để
sử dụng sáng chế, kiểu dáng công nghiệp đồng nhất với sáng chế, kiểu dáng công nghiệp
trong đơn đăng ký nhưng được tạo ra một cách độc lập thì sau khi văn bằng bảo hộ được
cấp, người đó có quyền tiếp tục sử dụng sáng chế, kiểu dáng công nghiệp trong phạm vi
và khối lượng đã sử dụng hoặc đã chuẩn bị để sử dụng mà không phải xin phép hoặc trả
tiền đền bù cho chủ sở hữu sáng chế, kiểu dáng công nghiệp được bảo hộ.
- Việc thực hiện quyền của người sử dụng trước sáng chế, kiểu dáng công nghiệp không bị
coi là xâm phạm quyền của chủ sở hữu sáng chế, kiểu dáng công nghiệp.”
- Như vậy, quyền sử dụng trước sáng chế, kiểu dáng công nghiệp được áp dụng khi có các
điều kiện:
+ Phải có hành vi sử dụng của người sử dụng trước diễn ra trên thực tế: Họ đã trực
tiếp khai thác sáng chế, kiểu dáng công nghiệp hoặc đã chuẩn bị đầu tư để khai
thác đối tượng đó ( lập nhà xưởng, mua thiết bị máy móc, vật tư, thuê mướn nhân
55
công,..)
+ Việc sử dụng, triển khai chuẩn bị sử dụng phải diễn ra trước ngày công bố đơn
đăng kí yêu cầu bảo hộ.
+ Sáng chế, kiểu dáng công nghiệp mà người sử dụng trước đưa vào khai thác được
tạo ra một cách độc lập.
76. Phân tích quyền tạm thời đối với sáng chế, kiểu dáng công nghiệp, thiết kế bố
trí?
- Điều 131, Luật sở hữu trí năm 2005 đã được sửa đổi, bổ sung năm 2009)
- 1. Trường hợp người nộp đơn đăng ký sáng chế, kiểu dáng công nghiệp biết rằng sáng
chế, kiểu dáng công nghiệp đang được người khác sử dụng nhằm mục đích thương mại
và người đó không có quyền sử dụng trước thì người nộp đơn có quyền thông báo bằng
văn bản cho người sử dụng về việc mình đã nộp đơn đăng ký, trong đó chỉ rõ ngày nộp
đơn và ngày công bố đơn trên Công báo sở hữu công nghiệp để người đó chấm dứt việc
sử dụng hoặc tiếp tục sử dụng.
- 2. Đối với thiết kế bố trí đã được người có quyền đăng ký hoặc người được người đó cho
phép khai thác thương mại trước ngày cấp Giấy chứng nhận đăng ký thiết kế bố trí mạch
tích hợp bán dẫn, nếu người có quyền đăng ký biết rằng thiết kế bố trí đó đang được
người khác sử dụng nhằm mục đích thương mại thì người đó có quyền thông báo bằng
văn bản về quyền đăng ký của mình đối với thiết kế bố trí đó cho người sử dụng để người
đó chấm dứt việc sử dụng thiết kế bố trí hoặc tiếp tục sử dụng.
- 3. Trong trường hợp đã được thông báo quy định tại khoản (1) và (2) Điều này mà người
được thông báo vẫn tiếp tục sử dụng sáng chế, kiểu dáng công nghiệp, thiết kế bố trí thì
khi Bằng độc quyền sáng chế, Bằng độc quyền giải pháp hữu ích, Bằng độc quyền kiểu
dáng công nghiệp, Giấy chứng nhận đăng ký thiết kế bố trí mạch tích hợp bán dẫn được
cấp, chủ sở hữu sáng chế, kiểu dáng công nghiệp, thiết kế bố trí có quyền yêu cầu người
đã sử dụng sáng chế, kiểu dáng công nghiệp, thiết kế bố trí phải trả một khoản tiền đền
bù tương đương với giá chuyển giao quyền sử dụng sáng chế, kiểu dáng công nghiệp,
thiết kế bố trí đó trong phạm vi và thời hạn sử dụng tương ứng.
77. Phân tích nghĩa vụ sử dụng sáng chế?
- Theo quy định tại Điều 23 Nghị định 103/2006/NĐ-CP Hướng dẫn Luật Sở hữu trí tuệ về
sở hữu công nghiệp thì nghĩa vụ sử dụng sáng chế được quy định cụ thể như sau:
- Khi có các nhu cầu quốc phòng, an ninh, phòng bệnh, chữa bệnh, dinh dưỡng cho nhân
dân hoặc các nhu cầu cấp thiết khác của xã hội mà người nắm độc quyền sử dụng sáng
chế không thực hiện nghĩa vụ sản xuất sản phẩm được bảo hộ hoặc áp dụng quy trình
được bảo hộ tại Việt Nam để đáp ứng các nhu cầu đó theo quy định tại khoản 1 Điều 136
và khoản 5 Điều 142 của Luật Sở hữu trí tuệ thì Bộ Khoa học và Công nghệ có thể cho
phép các tổ chức, cá nhân khác sử dụng sáng chế trên cơ sở ban hành quyết định bắt buộc
chuyển giao quyền sử dụng sáng chế theo quy định tại điểm b khoản 1 Điều 145 và đoạn
thứ nhất khoản 1 Điều 147 của Luật Sở hữu trí tuệ.
- Trong trường hợp các nhu cầu quốc phòng, an ninh, phòng bệnh, chữa bệnh, dinh dưỡng
cho nhân dân hoặc các nhu cầu cấp thiết khác của xã hội được đáp ứng bởi sản phẩm
nhập khẩu, sản phẩm do bên nhận chuyển quyền sử dụng sáng chế theo hợp đồng sản
56
xuất thì người nắm độc quyền sử dụng sáng chế không phải thực hiện nghĩa vụ sản xuất
sản phẩm được bảo hộ hoặc áp dụng quy trình được bảo hộ quy định tại khoản 1 Điều 23
Nghị định 103/2006/NĐ-CP.
78. Phân tích nghĩa vụ sử dụng nhãn hiệu? (điều 136)
Áp dụng điều 136 LSHTT:
- Sử dụng nhãn hiệu là nghĩa vụ của chủ sở hữu. Và người đó có nghĩa vụ sử dụng
nhãn hiệu liên tục.
- Thời hạn của đăng kí sử dụng nhãn hiệu là 10 năm, tuy nhiên, nếu không sử dụng
nhãn hiệu liên tục trong 5 năm thì nhãn hiệu ấy sẽ kết thúc trước thời hạn.
- Bên cạnh đó, chủ sở hữu nhãn hiệu có thể dùng vào những mục đích kinh tế, dịch
vụ, các hoạt động kinh doanh và để khác biệt với những sản phẩm khác.
- Chủ sở hữu nhãn hiệu buộc phải nằm trong phạm vi mà luật bảo hộ nhãn hiệu ấy
(điều 72)

79. Phân tích nghĩa vụ cho phép sử dụng sáng chế cơ bản nhằm sử dụng sáng chế phụ
thuộc? (điều 137)
- Về cơ bản, sáng chế phụ thuộc là sáng chế dựa theo 1 sáng chế khác (nếu không
có A thì sẽ không có B, và B chỉ tạo ra đc nếu có A)
- Theo luật định, nếu sáng chế phụ thuộc là 1 bước tiến quan trọng thì chủ sở hữu
sáng chế phụ thuộc này hoàn toàn có quyền yêu cầu người chủ sở hữu sáng chế gốc
chuyển giao với đk và giá cả hợp lý.
- Trong đk chủ sở hữu sáng chế cơ bản k đáp ứng yêu cầu của chủ sở hứu sáng chế
phụ thuộc thì cơ quan có thẩm quyền sẽ dùng điều 145, 146 (căn cứ bắt buộc chuyển
giao..)
- Phân tích: điều này tạo một điều kiện hết sức cho sự phát triển của xã hội, khi luật
đã tạo điều kiện cho các sáng chế mới dễ dàng được áp dụng vào cuộc sống hơn. Và
“nghĩa vụ cho phép” ở đây, có lẽ là nói với chủ sở hữu sáng chế cơ bản, và theo đó,
họ phải tạo điều kiện cho sáng chế phụ thuộc. Tất nhiên, cái giá thương mại phải hợp
lý và được 2 bên đồng ý, họ có nghĩa vụ phải giúp đỡ sáng chế phụ thuộc, nhưng
không vì thế mà bỏ qua lợi ích của họ, Trong trường hợp cần thiết, cơ quan NN có
thẩm quyền sẽ có yêu cầu theo điều 145 và 146

80. Nêu và phân tích các phương thức khai thác thương mại đối với quyền sở hữu trí tuệ
(có 4 phương thức)
1. Chuyển nhượng quyền shtt (mua bán quyền)
2. Chuyển quyền sử dụng (chuyển giấy phép - license (e quên tiếng anh là j r, nó là
giấy phép á)) là pp phổ biến nhất (điều 141)
3. Góp giá trị quyền sở hữu trí tuệ làm vốn góp trong kinh doanh ( hiểu nôm na là 1
bên góp tài sản trí tuệ để bên kia hiện thực hóa nó, góp vốn cùng kinh doanh 1 bên bỏ
não 1 bên bỏ tiền)
4. Nhượng quyền thương mại. (là bán tên thương mại đó – đc quy định trong điều
138, 141 – ngay 3) câu dưới

81. Phân biệt chuyển nhượng quyền sở hữu công nghiệp và chuyển quyền sử dụng đối
tượng sở hữu công nghiệp?

57
82. Phân tích trường hợp bắt buộc chuyển giao quyền sử dụng đối với sáng chế? (điều
145)
- Về cơ bản, điều này sẽ được áp dụng trong 1 số trường hợp nhất định như “sáng
chế phụ thuộc, sáng chế nhân danh nhà nước”, hay các vấn đề được luật định đề cập.
- Về ý a khoản 1, ta có thể thấy vc sử dụng sáng chế nhằm các mục đích công cộng
phi thương mại… và với mục đích giúp đỡ nhân dân (nhu cầu xh) là hoàn toàn hợp
lý, bởi nó phù hợp với tính chất nhân đạo, cx như hợp với ý chí vì dân, phục vụ- phát
triển quyền con người của nước nhà
- Ý b-c-d (bao gồm: k tạo đk, k thể thương thuyết với giá cả hợp lý, thực hiện hành
vi hạn chế cạnh tranh bị cấm). Các nhà làm luật đã đánh vào nghĩa vụ, quyền của
người độc quyền sáng chế không tạo điều kiện cũng như bỏ quên sáng chế của mình
sẽ bị bắt buộc chuyển giao. Điều này đảm bảo xã hội sẽ luôn phát triển và có thể tạo
điều kiện cho những người đi sau sử dụng sáng chế vì mục đích chung. Bên cạnh đó,
các nhà làm luật cũng đưa ra thời hiệu (3 năm, 4 năm) cho người nắm độc quyền, đây
là 1 khoảng thời gian hợp lý.

83. Phân tích các điều kiện hạn chế chuyển nhượng quyền sở hữu công nghiệp? (điều
139)
- Có tất cả 5 đk hạn chế
o Chỉ được chuyển nhượng trong phạm vi được bảo hộ SHTT là 1 ngành đặc thù,
vc chủ sh chỉ đc chuyển nhượng trong phạm vi được bảo hộ đã đảm bảo được quyền
và lợi ích của ng khác, bởi lữ, chủ sh chỉ có toàn quyền (nhân thân, tài sản) đối vs
những thứ trong phạm vi bảo hộ
o Quyền đối với chỉ dẫn địa lý k đc chuyển đây là điều cần có, bởi chỉ dẫn địa lý
phụ thuộc vào địa lý, là “dấu hiệu” để nhận biết sp của vùng đó, và nó là nguồn gốc.
vậy nên, ta k thể chuyển giao nó. VD: nước mắm Phú Quốc nếu k sx ở PQ thì k còn
là nó nữa, chỉ dẫn địa lý ở đây là PQ, dù có chuyển nhượng thế nào đi nữa thì ‘PQ” cx
k thể chuyển giao
o Nếu chuyển tên thương mại thì phải nhượng theo toàn bộ cơ sở kinh doanh và hd
kinh doanh dưới tên TM đó đây là điều cần thiết, để tránh 1sp khi đã chuyển tên
thương mại sẽ hoạt động sai với đăng kí (so vs cơ quan NN có thẩm quyền) tạo nên
58
sự khó khăn, ko thống nhất và rắc rối. Điều này cx góp phần tạo nên giá trị của “tên
thương mại”
o không được gây ra sự nhầm lẫn về đặc tính, nguồn gốc của hàng hóa, dịch vụ
mang nhãn hiệu. chuyển nhượng phải rõ ràng, minh bạch, không đc phép tạo nên sự
nhầm lẫn, tránh làm ảnh hưởng tới khách hàng và gây khó khăn tới cơ quan NN có
thẩm quyền (khsc biệt về sản phẩm…)
o cuối cùng là chỉ chuyển nhượng cho các cá nhân, tổ chức đáp ứng đủ các điều
kiện tránh cá lớn nuốt cá bé, dùng tiền bạc để phá hủy 1 nhãn hiệu…

84. Phân tích các trường hợp hạn chế chuyển giao quyền sử dụng đối tượng sở hữu công
nghiệp? (điuef 142)
- Bản chất của “chuyển giao quyền sd với đối tượng sh công nghiệp” chính là việc
mình cho 1 bên sử dụng SHCN của mình và trong phạm vi của mình. Chính vì lẽ ấy:
o K đc chuyển giao quyền sd chỉ dẫn địa lý, tên thương mại. Bởi lẽ theo điều 139,
chuyển giao tên tm sẽ kéo theo cơ sở kinh doanh và hoạt động kinh doanh, và chỉ dẫn
địa lý là cái “nguồn”, không đc chuyển giao dưới mọi trường hợp.
o Quyền sử dụng nhãn hiệu tập thể (khoản 17 điều 4) không được chuyển giao cho
tổ chức, cá nhân không phải là thành viên của chủ sở hữu nhãn hiệu tập thể đó. theo
định nghĩa thì điều này là hiển nhiên, nếu anh không là chủ sở hữu, anh k thể sử dụng
nhãn hiệu của chủ sở hữu (vì người ấy có cơ chế bảo vệ và có mọi quyền hạn với nó)
o Bên được chuyển quyền không được ký kết hợp đồng thứ cấp với bên thứ ba, trừ
trường hợp được bên chuyển quyền cho phép.
o Bên được chuyển quyền sử dụng nhãn hiệu có nghĩa vụ ghi chỉ dẫn trên hàng hóa,
bao bì hàng hóa về việc hàng hóa đó được sản xuất theo hợp đồng sử dụng nhãn hiệu.
đảm bảo quyền lợi của đôi bên theo hợp đồng (đây là 1 yếu tố bắt buộc);
o Bên được chuyển quyền sử dụng sáng chế theo hợp đồng độc quyền có nghĩa vụ
sử dụng sáng chế như chủ sở hữu sáng chế theo quy định tại khoản 1 Điều 136 của
Luật này

85. Phân tích các căn cứ xác định hành vi xâm phạm quyền sở hữu trí tuệ?
- Các hành vi:
o Điều 28: Xâm phạm quyền tác giả
Chiếm đoạt trái phép, mạo danh, công bô – phân phối mà k có sự đồng ý của tác
giả/đồng tác giả
Sửa chữa, xuyên tạc, sao chép tác phẩm
Các vấn đề về nhuận bút, thù lao…
o Điều 35: Hành vi xâm phạm các quyền liên quan
Như điều 28
Đối tượng là: “người biểu diễn, nhà sản xuất bản ghi âm, ghi hình, tổ chức phát
sóng.”
o Điều 126: Hành vi xâm phạm quyền đối với sáng chế, kiểu dáng công nghiệp,
thiết kế bố trí
Sử dụng kiểu dáng được bảo hộ hoặc k khác biệt đáng kể mà k có phép của chủ sở
hữu (thế nào là khác biệt k đáng kể?)
Sd mà k trả tiền
o Điều 129: Hành vi xâm phạm quyền đối với nhãn hiệu, tên thương mại và chỉ dẫn

59
địa lý
Sd dấu hiệu trùng với nhãn hiệu đc bảo hộ hoặc thuộc danh mục đăng ký trùng
với nhãn hiệu đó trên mọi mặt
Nếu có khả năng gây nhầm lẫn, trùng vs tên thương mại, chỉ dẫn thương mại cx k
đc xài
Đối với chỉ dẫn địa lý: sd chỉ dẫn địa lý nhưng mà sai xuất xứ, ảnh hưởng tới bản
gốc, nhằm mục đích lợi dụng danh tiếng, làm hiểu sai, gây hiểu nhầm cx k đc phép
o Điều 188: Hành vi xâm phạm quyền đối với giống cây trồng
Khai thác sử dụng các quyền của chủ mà k đc phép
Sd tên giống hoặc tương tự với các loại lq (gần gũi) vs giống đc bảo hộ
SD mà k trả tiền đền bù

86. Phân tích hành vi xâm phạm quyền tác giả?

Khái niệm: Xâm phạm quyền tác giả là hành vi chiếm đoạt, sao chép, mạo danh tác giả, công bố,
phân phối tác phẩm, sửa chữa, cắt xén hoặc xuyên tạc tác phẩm dưới bất kỳ hình thức nào gây
phương hại đến danh dự và uy tín của tác giả mà không có sự đồng ý của tác giả/chủ sở hữu
quyền tác giả.

Quy định pháp luật: có 16 hành vi xâm phạm quyền tác giả quy định tại Điều 28 Luật sở
hữu trí tuệ 2005 sửa đổi bổ sung năm 2009,2019.

1. Chiếm đoạt quyền tác giả đối với tác phẩm văn học, nghệ thuật, khoa học.

2. Mạo danh tác giả.

3. Công bố, phân phối tác phẩm mà không được phép của tác giả.

4. Công bố, phân phối tác phẩm có đồng tác giả mà không được phép của đồng tác giả
đó.

5. Sửa chữa, cắt xén hoặc xuyên tạc tác phẩm dưới bất kỳ hình thức nào gây phương hại
đến danh dự và uy tín của tác giả.

6. Sao chép tác phẩm mà không được phép của tác giả, chủ sở hữu quyền tác giả, trừ
trường hợp quy định tại điểm a và điểm đ khoản 1 Điều 25 của Luật này.

7. Làm tác phẩm phái sinh mà không được phép của tác giả, chủ sở hữu quyền tác giả
đối với tác phẩm được dùng để làm tác phẩm phái sinh, trừ trường hợp quy định tại điểm i
khoản 1 Điều 25 của Luật này.

8. Sử dụng tác phẩm mà không được phép của chủ sở hữu quyền tác giả, không trả tiền
nhuận bút, thù lao, quyền lợi vật chất khác theo quy định của pháp luật, trừ trường hợp quy định
tại khoản 1 Điều 25 của Luật này.

9. Cho thuê tác phẩm mà không trả tiền nhuận bút, thù lao và quyền lợi vật chất khác cho
tác giả hoặc chủ sở hữu quyền tác giả.

60
10. Nhân bản, sản xuất bản sao, phân phối, trưng bày hoặc truyền đạt tác phẩm đến công
chúng qua mạng truyền thông và các phương tiện kỹ thuật số mà không được phép của chủ sở
hữu quyền tác giả.

11. Xuất bản tác phẩm mà không được phép của chủ sở hữu quyền tác giả.

12. Cố ý hủy bỏ hoặc làm vô hiệu các biện pháp kỹ thuật do chủ sở hữu quyền tác giả
thực hiện để bảo vệ quyền tác giả đối với tác phẩm của mình.

13. Cố ý xóa, thay đổi thông tin quản lý quyền dưới hình thức điện tử có trong tác phẩm.

14. Sản xuất, lắp ráp, biến đổi, phân phối, nhập khẩu, xuất khẩu, bán hoặc cho thuê thiết
bị khi biết hoặc có cơ sở để biết thiết bị đó làm vô hiệu các biện pháp kỹ thuật do chủ sở hữu
quyền tác giả thực hiện để bảo vệ quyền tác giả đối với tác phẩm của mình.

15. Làm và bán tác phẩm mà chữ ký của tác giả bị giả mạo.

16. Xuất khẩu, nhập khẩu, phân phối bản sao tác phẩm mà không được phép của chủ sở
hữu quyền tác giả.

Cấu thành hành vi xâm phạm quyền tác giả:

(1) Đối tượng bị xem xét phải là đối tượng đang được bảo hộ bởi Luật SHTT

(2) có các hành vi theo như Điều 28 Luật sở hữu trí tuệ năm 2005 sửa đổi bổ sung năm
2009, 2019 nêu trên.

(3) Không thuộc vào trường hợp được phép sử dụng quyền tác giả

(4) Hành vi bị xem xét xảy ra tại Việt Nam. Hành vi bị xem xét cũng bị coi là xảy ra tại
Việt Nam nếu hành vi đó xảy ra trên mạng internet nhưng nhằm vào người tiêu dùng hoặc người
dùng tin tại Việt Nam.

Chế tài: Chế tài xử phạt được quy định tại Nghị định 131/2013/NĐ-CP, Bộ luật hình sự
2015 sđbs 2017.

Một số hành vi được xem là xâm phạm quyền tác giả:

– Bản sao tác phẩm được tạo ra một cách trái phép;

– Tác phẩm phái sinh được tạo ra một cách trái phép;

– Tác phẩm giả mạo tên, chữ ký của tác giả, mạo danh hoặc chiếm đoạt quyền tác giả;

– Phần tác phẩm bị trích đoạn, sao chép, lắp ghép trái phép;

– Sản phẩm có gắn thiết bị kỹ thuật bảo vệ quyền tác giả bị vô hiệu hóa trái phép.

87. Phân tích hành vi xâm phạm quyền liên quan?

61
Khái niệm: Hành vi xâm phạm quền liên quan đến tác giả là những hành vi xâm phạm đến các
như: quyền của tổ chức, cá nhân đối với cuộc biểu diễn, bản ghi âm, ghi hình, chương trình phát
sóng, tín hiệu vệ tinh mang chương trình được mã hóa.

Quy định của pháp luật: Các hành vi sau được coi là xâm phạm quyền liên quan đến
quyền tác giả là: Theo quy định của Điều 35 Luật SHTT 2005 SĐBS 2009,2019 có 10 hành vi
xâm phạm quyền liên quan bao gồm:

1. Chiếm đoạt quyền của người biểu diễn, nhà sản xuất bản ghi âm, ghi hình, tổ chức
phát sóng.

2. Mạo danh người biểu diễn, nhà sản xuất bản ghi âm, ghi hình, tổ chức phát sóng.

3. Công bố, sản xuất và phân phối cuộc biểu diễn đã được định hình, bản ghi âm, ghi
hình, chương trình phát sóng mà không được phép của người biểu diễn, nhà sản xuất bản ghi
âm, ghi hình, tổ chức phát sóng.

4. Sửa chữa, cắt xén, xuyên tạc dưới bất kỳ hình thức nào đối với cuộc biểu diễn gây
phương hại đến danh dự và uy tín của người biểu diễn.

5. Sao chép, trích ghép đối với cuộc biểu diễn đã được định hình, bản ghi âm, ghi hình,
chương trình phát sóng mà không được phép của người biểu diễn, nhà sản xuất bản ghi âm, ghi
hình, tổ chức phát sóng.

6. Dỡ bỏ hoặc thay đổi thông tin quản lý quyền dưới hình thức điện tử mà không được
phép của chủ sở hữu quyền liên quan.

7. Cố ý hủy bỏ hoặc làm vô hiệu các biện pháp kỹ thuật do chủ sở hữu quyền liên quan
thực hiện để bảo vệ quyền liên quan của mình.

8. Phát sóng, phân phối, nhập khẩu để phân phối đến công chúng cuộc biểu diễn, bản
sao cuộc biểu diễn đã được định hình hoặc bản ghi âm, ghi hình khi biết hoặc có cơ sở để biết
thông tin quản lý quyền dưới hình thức điện tử đã bị dỡ bỏ hoặc đã bị thay đổi mà không được
phép của chủ sở hữu quyền liên quan.

9. Sản xuất, lắp ráp, biến đổi, phân phối, nhập khẩu, xuất khẩu, bán hoặc cho thuê thiết
bị khi biết hoặc có cơ sở để biết thiết bị đó giải mã trái phép một tín hiệu vệ tinh mang chương
trình được mã hóa.

10. Cố ý thu hoặc tiếp tục phân phối một tín hiệu vệ tinh mang chương trình được mã hóa
khi tín hiệu đã được giải mã mà không được phép của người phân phối hợp pháp.

Căn cứ xác định hành vi vi phạm:

Theo điều 5 Văn bản hợp nhất số 03 VNHN/BKHCN năm 2014 quy định như sau: Hành
vi bị xem xét bị coi là hành vi xâm phạm quyền sở hữu trí tuệ quy định tại các Điều 28, 35, 126,
127, 129 và 188 của Luật Sở hữu trí tuệ, khi có đủ các căn cứ sau đây:

62
1. Đối tượng bị xem xét thuộc phạm vi các đối tượng đang được bảo hộ quyền sở hữu trí
tuệ.

2. Có yếu tố xâm phạm trong đối tượng bị xem xét.

3. Người thực hiện hành vi bị xem xét không phải là chủ thể quyền sở hữu trí tuệ và
không phải là người được pháp luật hoặc cơ quan có thẩm quyền cho phép theo quy định tại các
Điều 25, 26, 32, 33, khoản 2 và khoản 3 Điều 125, Điều 133, Điều 134, khoản 2 Điều 137, các
Điều 145, 190 và 195 của Luật Sở hữu trí tuệ.

4. Hành vi bị xem xét xảy ra tại Việt Nam.

Hành vi bị xem xét cũng bị coi là xảy ra tại Việt Nam nếu hành vi đó xảy ra trên mạng
internet nhưng nhằm vào người tiêu dùng hoặc người dùng tin tại Việt Nam.

Một số dạng hành vi xâm phạm quyền liên quan như: Bản định hình đầu tiên cuộc biểu
diễn được tạo ra một cách trái phép; Bản sao bản định hình cuộc biểu diễn, bản sao bản ghi âm,
ghi hình, bản sao chương trình phát sóng được tạo ra một cách trái phép;…

88. Phân tích hành vi xâm phạm quyền đối với sáng chế?

Sáng chế là giải pháp kỹ thuật được thể hiện dưới dạng sản phẩm hoặc quy trình nhằm giải quyết
một vấn đề xác định dựa trên việc ứng dụng các quy luật tự nhiên. Tùy vào việc đáp ứng các điều
kiện quy định, sáng chế sẽ được bảo hộ theo hình thức cấp bằng độc quyền sáng chế hoặc bằng
độc quyền giải pháp hữu ích. Tuy nhiên, việc ứng dụng những hiệu quả sáng chế cũng gặp nhiều
trở ngại bởi thực trạng khai thác sử dụng sáng chế một cách bất hợp pháp.

Quy định của pháp luật: Theo điều 126 Luật SHTT 2005 sđbs 2009,2019 thì có 2 hành vi
bị coi là xâm phạm quyền sáng chế:

Các hành vi sau đây bị coi là xâm phạm quyền của chủ sở hữu sáng chế, kiểu dáng công
nghiệp, thiết kế bố trí:

1. Sử dụng sáng chế được bảo hộ… trong thời hạn hiệu lực của văn bằng bảo hộ mà
không được phép của chủ sở hữu;

2. Sử dụng sáng chế mà không trả tiền đền bù theo quy định về quyền tạm thời quy định
tại Điều 131 của Luật này.

Căn cứ xác định hành vi xâm phạm: Theo điều 5 Văn bản hợp nhất số 03
VNHN/BKHCN năm 2014 quy định như sau: Hành vi bị xem xét bị coi là hành vi xâm phạm
quyền sở hữu trí tuệ quy định tại các Điều 28, 35, 126, 127, 129 và 188 của Luật Sở hữu trí tuệ,
khi có đủ các căn cứ sau đây:

1. Đối tượng bị xem xét thuộc phạm vi các đối tượng đang được bảo hộ quyền sở hữu trí
tuệ.

2. Có yếu tố xâm phạm trong đối tượng bị xem xét.

63
3. Người thực hiện hành vi bị xem xét không phải là chủ thể quyền sở hữu trí tuệ và
không phải là người được pháp luật hoặc cơ quan có thẩm quyền cho phép theo quy định tại các
Điều 25, 26, 32, 33, khoản 2 và khoản 3 Điều 125, Điều 133, Điều 134, khoản 2 Điều 137, các
Điều 145, 190 và 195 của Luật Sở hữu trí tuệ.

4. Hành vi bị xem xét xảy ra tại Việt Nam.

Hành vi bị xem xét cũng bị coi là xảy ra tại Việt Nam nếu hành vi đó xảy ra trên mạng
internet nhưng nhằm vào người tiêu dùng hoặc người dùng tin tại Việt Nam.

Một số trường hợp được xem là xâm phạm quyền đối với sáng chế:

Sản phẩm hoặc bộ phận của sản phẩm trùng lặp hoặc tương đương với sản phẩm hoặc bộ phận
của sản phẩm khác thuộc phạm vi bảo hộ sáng chế; Quy trình trùng lặp hoặc tương đương với
quy trình thuộc phạm vi bảo hộ sáng chế
89. Phân tích hành vi xâm phạm quyền đối với kiểu dáng công nghiệp?

Khái niệm: Kiểu dáng công nghiệp là hình dáng bên ngoài của sản phẩm được thể hiện bằng
hình khối, đường nét, màu sắc hoặc sự kết hợp những yếu tố này. Hiện nay, khi doanh nghiệp tạo
ra sản phẩm mới, họ luôn chú trọng đến kiểu dáng công nghiệp của sản phẩm, bởi đây là một
trong những nhân tố đánh đến sức tiêu thụ của sản phẩm. Theo đó hành vi xâm phạm kiểu dáng
công nghiệp là hành vi xâm phạm đến hình dáng bên ngoài, xâm phạm đến bảo hộ kiểu dáng
công nghiệp…

Quy định của pháp luật: Theo quy định tại Điều 126 Luật SHTT 2005 SĐBS 2009, 2019
có 2 hành vi được xem là xâm phạm kiểu dáng công nghiệp bao gồm:

1. Sử dụng sáng chế được bảo hộ, kiểu dáng công nghiệp được bảo hộ hoặc kiểu dáng
công nghiệp không khác biệt đáng kể với kiểu dáng đó, thiết kế bố trí được bảo hộ hoặc bất kỳ
phần nào có tính nguyên gốc của thiết kế bố trí đó trong thời hạn hiệu lực của văn bằng bảo hộ
mà không được phép của chủ sở hữu;

2. Sử dụng sáng chế, kiểu dáng công nghiệp, thiết kế bố trí mà không trả tiền đền bù theo
quy định về quyền tạm thời quy định tại Điều 131 của Luật này.

Xác định hành vi xâm phạm: Theo điều 5 VBHN số 03 của BKHCN “Hành vi bị xem xét
bị coi là hành vi xâm phạm quyền sở hữu trí tuệ quy định tại các Điều 28, 35, 126, 127, 129 và
188 của Luật Sở hữu trí tuệ, khi có đủ các căn cứ sau đây:

1. Đối tượng bị xem xét thuộc phạm vi các đối tượng đang được bảo hộ quyền sở hữu trí
tuệ.

2. Có yếu tố xâm phạm trong đối tượng bị xem xét.

3. Người thực hiện hành vi bị xem xét không phải là chủ thể quyền sở hữu trí tuệ và
không phải là người được pháp luật hoặc cơ quan có thẩm quyền cho phép theo quy định tại các
Điều 25, 26, 32, 33, khoản 2 và khoản 3 Điều 125, Điều 133, Điều 134, khoản 2 Điều 137, các
Điều 145, 190 và 195 của Luật Sở hữu trí tuệ.

64
4. Hành vi bị xem xét xảy ra tại Việt Nam.

Hành vi bị xem xét cũng bị coi là xảy ra tại Việt Nam nếu hành vi đó xảy ra trên mạng
internet nhưng nhằm vào người tiêu dùng hoặc người dùng tin tại Việt Nam.”

Các yếu tố xâm phạm: Theo điều 10 của VBHN số 03 của Bộ KHCN “1. Yếu tố xâm
phạm quyền đối với kiểu dáng công nghiệp là sản phẩm hoặc phần của sản phẩm mà hình dáng
bên ngoài không khác biệt đáng kể với kiểu dáng công nghiệp được bảo hộ.

2. Căn cứ để xác định yếu tố xâm phạm quyền đối với kiểu dáng công nghiệp là phạm vi
bảo hộ kiểu dáng công nghiệp được xác định theo Bằng độc quyền kiểu dáng công nghiệp.

3. Sản phẩm, phần sản phẩm bị coi là yếu tố xâm phạm quyền đối với kiểu dáng công
nghiệp thuộc một trong các trường hợp sau đây:

a) Trên sản phẩm hoặc phần sản phẩm bị xem xét, kể cả trường hợp đã được cấp Bằng
độc quyền kiểu dáng công nghiệp, có tập hợp các đặc điểm tạo dáng hợp thành một tổng thể là
bản sao hoặc về bản chất là bản sao (gần như không thể phân biệt được sự khác biệt) của kiểu
dáng công nghiệp của chủ sở hữu khác đã được bảo hộ mà không được sự đồng ý của người đó;

b) Trên sản phẩm hoặc phần sản phẩm bị xem xét có tập hợp các đặc điểm tạo dáng hợp
thành một tổng thể là bản sao hoặc về bản chất là bản sao của kiểu dáng công nghiệp của ít nhất
một sản phẩm trong bộ sản phẩm được bảo hộ của người khác.

4. Kiểu dáng công nghiệp của một sản phẩm (phần sản phẩm) chỉ bị coi là không khác
biệt đáng kể với kiểu dáng công nghiệp được bảo hộ theo quy định tại khoản 1 Điều này khi kiểu
dáng công nghiệp đó là bản sao hoặc về bản chất là bản sao của kiểu dáng công nghiệp đã được
bảo hộ.”

90. Phân tích hành vi xâm phạm quyền đối với nhãn hiệu?

Khái niệm: Hành vi xâm phạm quyền đối với nhãn nhiệu là các hành vi sao chép, làm giả, sử
dụng trái phép toàn bộ hoặc một phần của nhãn hiệu đã được cấp giấy chứng nhận đăng ký nhãn
hiệu mà không được sự đồng ý của chủ sở hữu nhãn hiệu đó.

Quy định của pháp luật: theo khoản 1 Điều 129 Thì có 4 hành vi được xem là xâm phạm
quyền đối với nhãn hiệu.

“a) Sử dụng dấu hiệu trùng với nhãn hiệu được bảo hộ cho hàng hóa, dịch vụ trùng với
hàng hóa, dịch vụ thuộc danh mục đăng ký kèm theo nhãn hiệu đó;

b) Sử dụng dấu hiệu trùng với nhãn hiệu được bảo hộ cho hàng hóa, dịch vụ tương tự
hoặc liên quan tới hàng hóa, dịch vụ thuộc danh mục đăng ký kèm theo nhãn hiệu đó, nếu việc
sử dụng có khả năng gây nhầm lẫn về nguồn gốc hàng hóa, dịch vụ;

c) Sử dụng dấu hiệu tương tự với nhãn hiệu được bảo hộ cho hàng hóa, dịch vụ trùng,
tương tự hoặc liên quan tới hàng hóa, dịch vụ thuộc danh mục đăng ký kèm theo nhãn hiệu đó,
nếu việc sử dụng có khả năng gây nhầm lẫn về nguồn gốc hàng hóa, dịch vụ;

65
d) Sử dụng dấu hiệu trùng hoặc tương tự với nhãn hiệu nổi tiếng hoặc dấu hiệu dưới
dạng dịch nghĩa, phiên âm từ nhãn hiệu nổi tiếng cho hàng hóa, dịch vụ bất kỳ, kể cả hàng hóa,
dịch vụ không trùng, không tương tự và không liên quan tới hàng hóa, dịch vụ thuộc danh mục
hàng hóa, dịch vụ mang nhãn hiệu nổi tiếng, nếu việc sử dụng có khả năng gây nhầm lẫn về
nguồn gốc hàng hóa hoặc gây ấn tượng sai lệch về mối quan hệ giữa người sử dụng dấu hiệu đó
với chủ sở hữu nhãn hiệu nổi tiếng.”

Xác định hành vi xâm phạm: Theo điều 5 VBHN số 03 của BKHCN “Hành vi bị xem xét
bị coi là hành vi xâm phạm quyền sở hữu trí tuệ quy định tại các Điều 28, 35, 126, 127, 129 và
188 của Luật Sở hữu trí tuệ, khi có đủ các căn cứ sau đây:

1. Đối tượng bị xem xét thuộc phạm vi các đối tượng đang được bảo hộ quyền sở hữu trí
tuệ.

2. Có yếu tố xâm phạm trong đối tượng bị xem xét.

3. Người thực hiện hành vi bị xem xét không phải là chủ thể quyền sở hữu trí tuệ và
không phải là người được pháp luật hoặc cơ quan có thẩm quyền cho phép theo quy định tại các
Điều 25, 26, 32, 33, khoản 2 và khoản 3 Điều 125, Điều 133, Điều 134, khoản 2 Điều 137, các
Điều 145, 190 và 195 của Luật Sở hữu trí tuệ.

4. Hành vi bị xem xét xảy ra tại Việt Nam.

Hành vi bị xem xét cũng bị coi là xảy ra tại Việt Nam nếu hành vi đó xảy ra trên mạng
internet nhưng nhằm vào người tiêu dùng hoặc người dùng tin tại Việt Nam.”

Các yếu tố xâm phạm: Theo điều 11 của VBHN số 03 của Bộ KHCN có các yếu tố xâm
phạm đến nhãn hiệu như sau:

1. Yếu tố xâm phạm quyền đối với nhãn hiệu là dấu hiệu gắn trên hàng hóa, bao bì hàng
hóa, phương tiện dịch vụ, giấy tờ giao dịch, biển hiệu, phương tiện quảng cáo và các phương
tiện kinh doanh khác, trùng hoặc tương tự tới mức gây nhầm lẫn với nhãn hiệu được bảo hộ.

2. Căn cứ để xem xét yếu tố xâm phạm quyền đối với nhãn hiệu là phạm vi bảo hộ nhãn
hiệu, gồm mẫu nhãn hiệu và danh mục hàng hóa, dịch vụ được xác định tại Giấy chứng nhận
đăng ký nhãn hiệu hoặc Giấy chứng nhận nhãn hiệu đăng ký quốc tế được bảo hộ tại Việt Nam.

3. Để xác định một dấu hiệu bị nghi ngờ có phải là yếu tố xâm phạm quyền đối với nhãn
hiệu hay không, cần phải so sánh dấu hiệu đó với nhãn hiệu, đồng thời phải so sánh sản phẩm,
dịch vụ mang dấu hiệu đó với sản phẩm, dịch vụ thuộc phạm vi bảo hộ. Chỉ có thể khẳng định có
yếu tố xâm phạm khi đáp ứng cả hai điều kiện sau đây:

a) Dấu hiệu bị nghi ngờ trùng hoặc tương tự đến mức gây nhầm lẫn với nhãn hiệu thuộc
phạm vi bảo hộ; trong đó một dấu hiệu bị coi là trùng với nhãn hiệu thuộc phạm vi bảo hộ nếu
có cùng cấu tạo, cách trình bày (kể cả màu sắc); một dấu hiệu bị coi là tương tự đến mức gây
nhầm lẫn với nhãn hiệu thuộc phạm vi bảo hộ nếu có một số đặc điểm hoàn toàn trùng nhau
hoặc tương tự đến mức không dễ dàng phân biệt với nhau về cấu tạo, cách phát âm, phiên âm
đối với dấu hiệu, chữ, ý nghĩa, cách trình bày, màu sắc và gây nhầm lẫn cho người tiêu dùng về

66
hàng hóa, dịch vụ mang nhãn hiệu;

b) Hàng hóa, dịch vụ mang dấu hiệu bị nghi ngờ trùng hoặc tương tự về bản chất hoặc
có liên hệ về chức năng, công dụng và có cùng kênh tiêu thụ với hàng hóa, dịch vụ thuộc phạm vi
bảo hộ.

4. Đối với nhãn hiệu nổi tiếng, dấu hiệu bị nghi ngờ bị coi là yếu tố xâm phạm nếu:

a) Dấu hiệu bị nghi ngờ đáp ứng điều kiện quy định tại điểm a khoản 3 Điều này;

b) Hàng hóa, dịch vụ mang dấu hiệu bị nghi ngờ đáp ứng điều kiện quy định tại điểm b
khoản 3 Điều này hoặc hàng hóa, dịch vụ không trùng, không tương tự, không liên quan tới hàng
hóa, dịch vụ mang nhãn hiệu nổi tiếng nhưng có khả năng gây nhầm lẫn cho người tiêu dùng về
nguồn gốc hàng hóa, dịch vụ hoặc gây ấn tượng sai lệch về mối quan hệ giữa người sản xuất,
kinh doanh sản phẩm, dịch vụ đó với chủ sở hữu nhãn hiệu nổi tiếng.

5. Trường hợp sản phẩm, dịch vụ mang dấu hiệu trùng hoặc khó phân biệt về tổng thể
cấu tạo và cách trình bày so với nhãn hiệu được bảo hộ cho sản phẩm, dịch vụ cùng loại thuộc
phạm vi bảo hộ thì bị coi là hàng hóa giả mạo nhãn hiệu theo quy định tại Điều 213 của Luật Sở
hữu trí tuệ.

Chế tài: xử phạt vi phạm hành chính tại Nghị định 99/2013/NĐ-CP ngày 29/08/2013, xử
lý hình sự tại Điều 226 Bộ luật hình sự 2015 sđbs 2017.

91. Phân tích hành vi xâm phạm quyền đối với chỉ dẫn địa lý?

Khái niệm: hành vi xâm phạm quyền đối với chỉ dẫn địa lý là hành vi sử dụng chỉ dẫn địa lý trái
với quy định của pháp luật như việc Sử dụng chỉ dẫn địa lý được bảo hộ cho sản phẩm mặc dù có
nguồn gốc xuất xứ từ khu vực địa lý mang chỉ dẫn địa lý, Sử dụng chỉ dẫn địa lý được bảo hộ
cho sản phẩm tương tự với sản phẩm mang chỉ dẫn địa lý.

Quy định pháp luật: Hành vi xâm phạm quyền đối với chỉ dẫn địa lý được quy định tại
khoản 3 Điều 129 Luật SHTT 2005 sđbs 2009,2019 theo đó có 4 hành vi được xem là xâm phạm
như sau: a) Sử dụng chỉ dẫn địa lý được bảo hộ cho sản phẩm mặc dù có nguồn gốc xuất xứ từ
khu vực địa lý mang chỉ dẫn địa lý, nhưng sản phẩm đó không đáp ứng các tiêu chuẩn về tính
chất, chất lượng đặc thù của sản phẩm mang chỉ dẫn địa lý;

b) Sử dụng chỉ dẫn địa lý được bảo hộ cho sản phẩm tương tự với sản phẩm mang chỉ
dẫn địa lý nhằm mục đích lợi dụng danh tiếng, uy tín của chỉ dẫn địa lý;

c) Sử dụng bất kỳ dấu hiệu nào trùng hoặc tương tự với chỉ dẫn địa lý được bảo hộ cho
sản phẩm không có nguồn gốc từ khu vực địa lý mang chỉ dẫn địa lý đó làm cho người tiêu dùng
hiểu sai rằng sản phẩm có nguồn gốc từ khu vực địa lý đó;

d) Sử dụng chỉ dẫn địa lý được bảo hộ đối với rượu vang, rượu mạnh cho rượu vang,
rượu mạnh không có nguồn gốc xuất xứ từ khu vực địa lý tương ứng với chỉ dẫn địa lý đó, kể cả
trường hợp có nêu chỉ dẫn về nguồn gốc xuất xứ thật của hàng hóa hoặc chỉ dẫn địa lý được sử
dụng dưới dạng dịch nghĩa, phiên âm hoặc được sử dụng kèm theo các từ loại, kiểu, dạng, phỏng

67
theo hoặc những từ tương tự như vậy.

Hành vi xâm phạm được xác định như sau: Theo điều 5 VBHN số 03 của BKHCN “Hành
vi bị xem xét bị coi là hành vi xâm phạm quyền sở hữu trí tuệ quy định tại các Điều 28, 35, 126,
127, 129 và 188 của Luật Sở hữu trí tuệ, khi có đủ các căn cứ sau đây:

1. Đối tượng bị xem xét thuộc phạm vi các đối tượng đang được bảo hộ quyền sở hữu trí
tuệ.

2. Có yếu tố xâm phạm trong đối tượng bị xem xét.

3. Người thực hiện hành vi bị xem xét không phải là chủ thể quyền sở hữu trí tuệ và
không phải là người được pháp luật hoặc cơ quan có thẩm quyền cho phép theo quy định tại các
Điều 25, 26, 32, 33, khoản 2 và khoản 3 Điều 125, Điều 133, Điều 134, khoản 2 Điều 137, các
Điều 145, 190 và 195 của Luật Sở hữu trí tuệ.

4. Hành vi bị xem xét xảy ra tại Việt Nam.

Hành vi bị xem xét cũng bị coi là xảy ra tại Việt Nam nếu hành vi đó xảy ra trên mạng
internet nhưng nhằm vào người tiêu dùng hoặc người dùng tin tại Việt Nam.”

Các yếu tố xâm phạm: Theo điều 12 của VBHN số 03 của Bộ KHCN có các yếu tố xâm
phạm đến chỉ dẫn địa lý là:

1. Yếu tố xâm phạm quyền đối với chỉ dẫn địa lý được thể hiện dưới dạng dấu hiệu gắn
trên hàng hóa, bao bì hàng hóa, phương tiện dịch vụ, giấy tờ giao dịch, biển hiệu, phương tiện
quảng cáo và các phương tiện kinh doanh khác, trùng hoặc tương tự tới mức gây nhầm lẫn với
chỉ dẫn địa lý được bảo hộ.

2. Căn cứ để xem xét yếu tố xâm phạm quyền đối với chỉ dẫn địa lý là phạm vi bảo hộ chỉ
dẫn địa lý được xác định tại Quyết định đăng bạ chỉ dẫn địa lý.

3. Để xác định một dấu hiệu bị nghi ngờ có phải là yếu tố xâm phạm quyền đối với chỉ
dẫn địa lý được bảo hộ hay không, cần phải so sánh dấu hiệu đó với chỉ dẫn địa lý và so sánh
sản phẩm mang dấu hiệu bị nghi ngờ với sản phẩm mang chỉ dẫn địa lý được bảo hộ dựa trên
các căn cứ sau đây:

a) Dấu hiệu bị nghi ngờ trùng hoặc tương tự đến mức gây nhầm lẫn với chỉ dẫn địa lý,
trong đó một dấu hiệu bị coi là trùng với chỉ dẫn địa lý nếu giống với chỉ dẫn địa lý được bảo hộ
về cấu tạo từ ngữ, kể cả cách phát âm, phiên âm đối với chữ cái, ý nghĩa hoặc về hình ảnh, biểu
tượng thuộc phạm vi bảo hộ của chỉ dẫn địa lý; một dấu hiệu bị coi là tương tự đến mức gây
nhầm lẫn với chỉ dẫn địa lý nếu tương tự đến mức gây nhầm lẫn với chỉ dẫn địa lý đó về cấu tạo
từ ngữ, kể cả cách phát âm, phiên âm đối với chữ cái, ý nghĩa hoặc về hình ảnh, biểu tượng
thuộc phạm vi bảo hộ của chỉ dẫn địa lý;

b) Sản phẩm mang dấu hiệu bị nghi ngờ trùng hoặc tương tự với sản phẩm mang chỉ dẫn
địa lý được bảo hộ, trong đó sản phẩm bị coi là trùng hoặc tương tự nếu giống nhau hoặc tương
tự nhau về bản chất, chức năng, công dụng và kênh tiêu thụ;

68
c) Đối với rượu vang, rượu mạnh, ngoài quy định tại điểm a, điểm b khoản này, dấu hiệu
trùng với chỉ dẫn địa lý được bảo hộ, kể cả thể hiện dưới dạng dịch nghĩa, phiên âm hoặc kèm
theo các từ loại, kiểu, dạng, phỏng theo hoặc những từ tương tự như vậy được sử dụng cho sản
phẩm không có nguồn gốc xuất xứ từ khu vực địa lý mang chỉ dẫn địa lý được bảo hộ cũng bị coi
là yếu tố xâm phạm quyền đối với chỉ dẫn địa lý.

4. Trường hợp sản phẩm mang dấu hiệu trùng hoặc khó phân biệt về tổng thể cấu tạo và
cách trình bày so với chỉ dẫn địa lý được bảo hộ cho sản phẩm cùng loại thuộc phạm vi bảo hộ
thì bị coi là hàng hóa giả mạo chỉ dẫn địa lý quy định tại Điều 213 của Luật Sở hữu trí tuệ.

Chế tài: xử phạt vi phạm hành chính tại Nghị định 99/2013/NĐ-CP ngày 29/08/2013, xử
lý hình sự tại Điều 226 Bộ luật hình sự 2015 sđbs 2017.

92. Phân tích hành vi xâm phạm quyền đối với tên thương mại?

Khái niệm: Hành vi bị coi là xâm phạm quyền đối với tên thương mại là: Mọi hành vi sử dụng
chỉ dẫn thương mại trùng hoặc tương tự với tên thương mại của người khác đã được sử dụng
trước cho cùng loại sản phẩm, dịch vụ hoặc cho sản phẩm, dịch vụ tương tự, gây nhầm lẫn về
chủ thể kinh doanh, cơ sở kinh doanh, hoạt động kinh doanh dưới tên thương mại đó đều bị coi
là xâm phạm quyền đối với tên thương mại.

Quy định pháp luật: Khoản 2 Điều 129 Luật SHTT hiện hành “Mọi hành vi sử dụng chỉ
dẫn thương mại trùng hoặc tương tự với tên thương mại của người khác đã được sử dụng trước
cho cùng loại sản phẩm, dịch vụ hoặc cho sản phẩm, dịch vụ tương tự, gây nhầm lẫn về chủ thể
kinh doanh, cơ sở kinh doanh, hoạt động kinh doanh dưới tên thương mại đó đều bị coi là xâm
phạm quyền đối với tên thương mại.”

Các yếu tố xâm phạm: Theo điều 13 của VBHN số 03 của Bộ KHCN có các yếu tố xâm
phạm đến tên thương mại là: 1. Yếu tố xâm phạm quyền đối với tên thương mại được thể hiện
dưới dạng chỉ dẫn thương mại gắn trên hàng hóa, bao bì hàng hóa, phương tiện dịch vụ, giấy tờ
giao dịch, biển hiệu, phương tiện quảng cáo và các phương tiện kinh doanh khác, trùng hoặc
tương tự tới mức gây nhầm lẫn với tên thương mại được bảo hộ.

2. Căn cứ để xem xét yếu tố xâm phạm quyền đối với tên thương mại là phạm vi bảo hộ
tên thương mại được xác định trên cơ sở các chứng cứ thể hiện việc sử dụng tên thương mại đó
một cách hợp pháp, trong đó xác định cụ thể về chủ thể kinh doanh, cơ sở kinh doanh, hoạt động
kinh doanh và sản phẩm, dịch vụ mang tên thương mại.

3. Để xác định một dấu hiệu bị nghi ngờ có phải là yếu tố xâm phạm quyền đối với tên
thương mại hay không, cần phải so sánh dấu hiệu đó với tên thương mại được bảo hộ và phải so
sánh sản phẩm, dịch vụ mang dấu hiệu đó với sản phẩm, dịch vụ thuộc phạm vi bảo hộ, dựa trên
các căn cứ sau đây:

a) Dấu hiệu bị nghi ngờ trùng hoặc tương tự đến mức gây nhầm lẫn với tên thương mại
được bảo hộ; trong đó một dấu hiệu bị coi là trùng với tên thương mại được bảo hộ nếu giống
với tên thương mại về cấu tạo từ ngữ, kể cả cách phát âm, phiên âm đối với chữ cái; một dấu
hiệu bị coi là tương tự với tên thương mại được bảo hộ nếu tương tự về cấu tạo, cách phát âm,
phiên âm đối với chữ cái, gây nhầm lẫn cho người tiêu dùng về chủ thể kinh doanh, cơ sở kinh
69
doanh, hoạt động kinh doanh dưới tên thương mại được bảo hộ;

b) Sản phẩm, dịch vụ mang dấu hiệu bị nghi ngờ bị coi là trùng hoặc tương tự với sản
phẩm, dịch vụ mang tên thương mại được bảo hộ nếu giống nhau hoặc tương tự nhau về bản
chất, chức năng, công dụng và kênh tiêu thụ.

Chế tài: xử phạt vi phạm hành chính tại Nghị định 99/2013/NĐ-CP ngày 29/08/2013

93. Phân tích các hành vi cạnh tranh không lành mạnh theo Luật Sở hữu trí tuệ?

Khái niệm: Hành vi cạnh tranh không lành mạnh là hành vi của doanh nghiệp trái với nguyên tắc
thiện chí, trung thực, tập quán thương mại và các chuẩn mực khác trong kinh doanh, gây thiệt hại
hoặc có thể gây thiệt hại đến quyền và lợi ích hợp pháp của doanh nghiệp khác.

Quy định pháp luật: Theo khoản 1 điều 130 Luật SHTT có các hành vi cạnh tranh không
lành mạnh như sau: a) Sử dụng chỉ dẫn thương mại gây nhầm lẫn về chủ thể kinh doanh, hoạt
động kinh doanh, nguồn gốc thương mại của hàng hóa, dịch vụ;

b) Sử dụng chỉ dẫn thương mại gây nhầm lẫn về xuất xứ, cách sản xuất, tính năng, chất lượng,
số lượng hoặc đặc điểm khác của hàng hóa, dịch vụ; về điều kiện cung cấp hàng hóa, dịch vụ;

c) Sử dụng nhãn hiệu được bảo hộ tại một nước là thành viên của điều ước quốc tế có quy định
cấm người đại diện hoặc đại lý của chủ sở hữu nhãn hiệu sử dụng nhãn hiệu đó mà Cộng hòa xã
hội chủ nghĩa Việt Nam cũng là thành viên, nếu người sử dụng là người đại diện hoặc đại lý của
chủ sở hữu nhãn hiệu và việc sử dụng đó không được sự đồng ý của chủ sở hữu nhãn hiệu và
không có lý do chính đáng;

d) Đăng ký, chiếm giữ quyền sử dụng hoặc sử dụng tên miền trùng hoặc tương tự gây nhầm lẫn
với nhãn hiệu, tên thương mại được bảo hộ của người khác hoặc chỉ dẫn địa lý mà mình không
có quyền sử dụng nhằm mục đích chiếm giữ tên miền, lợi dụng hoặc làm thiệt hại đến uy tín,
danh tiếng của nhãn hiệu, tên thương mại, chỉ dẫn địa lý tương ứng.

Chế tài:

+ Hành chính :Điều 211 Luật Sở hữu trí tuệ đã quy định những hành vi xâm phạm quyền
sở hữu trí tuệ bị xử phạt vi phạm hành chính, trong đó có hành vi cạnh tranh không lành mạnh về
sở hữu trí tuệ (tại khoản 3 Điều 211).

+ Hình sự: Cá nhân thực hiện hành vi xâm phạm quyền sở hữu trí tuệ có yếu tố cấu thành
tội phạm thì bị truy cứu trách nhiệm hình sự theo quy định của pháp luật hình sự.

94. Phân biệt hàng hóa xâm phạm quyền sở hữu trí tuệ và hàng hóa giả mạo về sở
hữu trí tuệ?

Xâm phạm quyền sở hữu trí Giả mạo quyền sở hữu trí tuệ

70
tuệ

Phạm vi đối tượng Hàng hóa xâm phạm quyền Hàng hóa giả mạo về SHTT
SHTT là các hàng hóa có bao gồm hàng hóa giả mạo
chứa yếu tố xâm phạm đến nhãn hiệu, giả mạo chỉ dẫn
quyền tác giả, quyền liên địa lý và hàng hóa sao chép
quan, sáng chế, thiết kế bố trí lậu ( Điều 213 LSHTT).
mạch tích hợp bản dẫn, kiểu Trong đó :
dáng công nghiệp, nhãn hiệu, Hàng hóa giả mạo nhãn hiệu
chỉ dẫn địa lý, tên thương mại là hàng hóa, bao bì của hàng
và giống cây trồng. hóa có gắn nhãn hiệu, dấu
hiệu trùng hoặc khó phân biệt
với nhãn hiệu, chỉ dẫn địa lý
đang được bảo hộ dùng cho
chính mặt hàng đó mà không
được phép của chủ sở hữu
nhãn hiệu hoặc tổ chức quản
lý chỉ dẫn địa lý. Hàng hóa
sao chép lậu là bản sao được
sản xuất mà không được phép
của chủ thể quyền tác giả và
quyền liên quan.
-> Đối tượng của hàng hóa
giả mạo về SHTT hẹp hơn rất
nhiều so với đối tượng của
hàng hóa xâm phạm quyền
SHTT, bao gồm các đối
tượng đó là nhãn hiệu, chỉ dẫn
địa lý và đối tượng quyền tác
giả và quyền liên quan. Trong
khi đó, đối tượng của hàng
hóa xâm phạm quyền SHTT
rất rộng, tất cả các đối tượng
SHTT đều có thể trở thành
đối tượng của hàng hóa xâm
phạm quyền SHTT.

Về tính chất và mức độ xâm Chủ thể có hành vi sản xuất, Chủ thể có hành vi sản xuất,
phạm kinh doanh, buôn bán... hàng kinh doanh, buôn bán... loại
hóa xâm phạm quyền SHTT hàng hóa giả mạo về SHTT
thường ít nghiêm trọng và thường là nguy hiểm và
nguy hiểm hơn ( lỗi cố ý/ vô ý nghiêm trọng hơn. ( lỗi cố ý ).
). VD : Nhà sách A mặc dù đã
VD : Công ty A mặc dù đã biết được bản quyền của cuốn
biết trên thị trường đã có sản sách Giáo trình lý luận chung
phẩm thuốc giảm đau hạ sốt về nhà nước và pháp luật là

71
“Panadol” là của Công ty của Trường Đại học Luật Hà
Sanofi. Nhưng tin rằng việc Nội, nhưng nhà sách này vẫn
sử dụng nhãn hiệu "Hapadol" cố tình in lậu cuốn sách này
cho sản phẩm cùng loại là để bán.
không xâm phạm quyền SHTT
đối với nhãn hiệu trên của
Công ty Sanofi. Tuy nhiên, khi Mức độ gây thiệt hại : Đối với
Công ty Sanofi yêu cầu cơ loại hàng hóa giả mạo về
quan thực thi xử lý và tiến SHTT, chủ thể thực hiện hành
hành giám định SHTT thì hai vi xâm phạm luôn có lỗi cố ý
nhãn hiệu trên bị kết luận là nên thông thường mức độ gây
có khả năng gây nhầm lẫn thiệt hại của hành vi này gây
cho người tiêu dùng. ra cho chủ thể quyền SHTT
và cho xã hội luôn cao hơn.
Mức độ thấp hơn.

95. Trình bày các biện pháp bảo vệ quyền sở hữu trí tuệ.

● Biện pháp tự bảo vệ : Biện pháp tự bảo vệ cho phép chủ thể quyền sở hữu trí tuệ được
tiến hành các biện pháp trong khuôn khổ pháp luật nhằm bảo vệ quyền và lợi ích hợp
pháp của họ liên quan quan đến các đối tượng của quyền sở hữu trí tuệ.
Quy định tai Điều 198 LSHTT 2019 :
1. Chủ thể quyền sở hữu trí tuệ có quyền áp dụng các biện pháp sau đây để bảo vệ quyền sở hữu
trí tuệ của mình:
a) Áp dụng biện pháp công nghệ nhằm ngăn ngừa hành vi xâm phạm quyền sở hữu trí tuệ;
b) Yêu cầu tổ chức, cá nhân có hành vi xâm phạm quyền sở hữu trí tuệ phải chấm dứt hành vi
xâm phạm, xin lỗi, cải chính công khai, bồi thường thiệt hại;
c) Yêu cầu cơ quan nhà nước có thẩm quyền xử lý hành vi xâm phạm quyền sở hữu trí tuệ theo
quy định của Luật này và các quy định khác của pháp luật có liên quan;
d) Khởi kiện ra tòa án hoặc trọng tài để bảo vệ quyền, lợi ích hợp pháp của mình.
2. Tổ chức, cá nhân bị thiệt hại do hành vi xâm phạm quyền sở hữu trí tuệ hoặc phát hiện hành vi
xâm phạm quyền sở hữu trí tuệ gây thiệt hại cho người tiêu dùng hoặc cho xã hội có quyền yêu
cầu cơ quan nhà nước có thẩm quyền xử lý hành vi xâm phạm quyền sở hữu trí tuệ theo quy định
của Luật này và các quy định khác của pháp luật có liên quan.
3. Tổ chức, cá nhân bị thiệt hại hoặc có khả năng bị thiệt hại do hành vi cạnh tranh không lành
mạnh có quyền yêu cầu cơ quan nhà nước có thẩm quyền áp dụng các biện pháp dân sự quy định
tại Điều 202 của Luật này và các biện pháp hành chính theo quy định của pháp luật về cạnh
tranh.
4.[44] Tổ chức, cá nhân là bị đơn trong vụ kiện xâm phạm quyền sở hữu trí tuệ, nếu được Tòa án
kết luận là không thực hiện hành vi xâm phạm có quyền yêu cầu Tòa án buộc nguyên đơn thanh
toán cho mình chi phí hợp lý để thuê luật sư hoặc các chi phí khác theo quy định của pháp luật.
5.[45] Tổ chức, cá nhân lạm dụng thủ tục bảo vệ quyền sở hữu trí tuệ mà gây thiệt hại cho tổ
72
chức, cá nhân khác thì tổ chức, cá nhân bị thiệt hại có quyền yêu cầu Tòa án buộc bên lạm dụng
thủ tục đó phải bồi thường cho những thiệt hại do việc lạm dụng gây ra, trong đó bao gồm chi
phí hợp lý để thuê luật sư. Hành vi lạm dụng thủ tục bảo vệ quyền sở hữu trí tuệ bao gồm hành vi
cố ý vượt quá phạm vi hoặc mục tiêu của thủ tục này.
● Biện pháp hành chính : Là biện pháp được các cơ quan NN có thẩm quyền áp dụng để xử
lý những hành vi xâm phạm quyền tác giả, quyền sở hữu công nghiệp và quyền đối với
giống cây trồng của cá nhân, tổ chức mà chưa đến mức truy cứu trách nhiệm hình sự.
● Các hành vi VP bị xử phạt hành chính được quy định tại Điều 211 LSHTT, bao gồm :
- Xâm phạm quyền sở hữu trí tuệ gây thiệt hại cho tác giả, chủ sở hữu, người tiêu
dùng hoặc cho xã hội
- Sản xuất, nhập khẩu, vận chuyển, buôn bán hàng hóa giả mạo về sở hữu trí tuệ
quy định tại Điều 213 của Luật này hoặc giao cho người khác thực hiện hành vi
này;
- Sản xuất, nhập khẩu, vận chuyển, buôn bán, tàng trữ tem, nhãn hoặc vật phẩm
khác mang nhãn hiệu hoặc chỉ dẫn địa lý giả mạo hoặc giao cho người khác thực
hiện hành vi này.
● Biện pháp xử lý hình sự : Điều 212 LSHTT quy định : Cá nhân thực hiện hành vi xâm
phạm quyền SHTT có yếu tố cấu thành tội phạm thì bị truy cứu TNHS theo quy định của
pháp luật hình sự.
● Biện pháp dân sự : Biện pháp dân sự được sd để xử lý hành vi xâm phạm theo yêu cầu
của chủ thể quyền SHTT hoặc của tổ chức, cá nhân bị thiệt hại do hành vi vi phạm gây
ra, kể cả hvi đó đã hoặc đang bị xử lý bằng biện pháp hành chính hoặc biện pháp hình sự.
● Biện pháp kiểm soát hàng hóa xuất, nhập khẩu liên quan đến SHTT.
Theo qdinh của PLVN, cơ quan hải quan có thẩm quyền áp dụng các biện pháp kiểm soát
hàng hóa xuất, nhập khẩu liên quan đến SHTT, cụ thể là : Biện pháp tạm dừng làm thủ
tục hải quan đối với hàng hóa bị nghi ngờ xâm phạm QSHTT và biện pháp kiểm tra, gsat
để phát hiện hàng hóa có dấu hiệu xâm phạm quyền SHTT.
96. Trình bày thẩm quyền xử lý hành vi xâm phạm quyền sở hữu trí tuệ.
Được quy định tại Điều 200 Luật sở hữu trí tuệ 2019, theo đó :
Điều 200. Thẩm quyền xử lý hành vi xâm phạm quyền sở hữu trí tuệ
1. Trong phạm vi nhiệm vụ, quyền hạn của mình, các cơ quan Tòa án, Thanh tra, Quản lý thị
trường, Hải quan, Công an, Ủy ban nhân dân các cấp có thẩm quyền xử lý hành vi xâm phạm
quyền sở hữu trí tuệ.
2. Việc áp dụng biện pháp dân sự, hình sự thuộc thẩm quyền của Tòa án. Trong trường hợp cần
thiết, Tòa án có thể áp dụng biện pháp khẩn cấp tạm thời theo quy định của pháp luật.
3. Việc áp dụng biện pháp hành chính thuộc thẩm quyền của các cơ quan Thanh tra, Công an,
Quản lý thị trường, Hải quan, Ủy ban nhân dân các cấp. Trong trường hợp cần thiết, các cơ quan
này có thể áp dụng biện pháp ngăn chặn và bảo đảm xử phạt hành chính theo quy định của pháp
luật.
4. Việc áp dụng biện pháp kiểm soát hàng hóa xuất khẩu, nhập khẩu liên quan đến sở hữu trí tuệ
thuộc thẩm quyền của cơ quan hải quan.

97. Trình bày về giám định sở hữu trí tuệ.


73
● Khái niệm : Giám định về sở hữu trí tuệ là việc tổ chức, cá nhân quy định tại khoản 2 và
khoản 3 Điều này sử dụng kiến thức, nghiệp vụ chuyên môn để đánh giá, kết luận về
những vấn đề có liên quan đến quyền sở hữu trí tuệ.
● Thẩm quyền giám định :
- Doanh nghiệp, hợp tác xã, đơn vị sự nghiệp, tổ chức hành nghề luật sư, trừ tổ
chức hành nghề luật sư nước ngoài hành nghề tại Việt Nam đáp ứng các điều kiện
sau đây được thực hiện hoạt động giám định về sở hữu trí tuệ:
+ Có nhân lực, cơ sở vật chất - kỹ thuật đáp ứng yêu cầu tổ chức hoạt động
giám định theo quy định của pháp luật;
+ Có chức năng thực hiện hoạt động giám định về sở hữu trí tuệ được ghi
nhận trong Giấy chứng nhận đăng ký kinh doanh, Giấy chứng nhận đăng
ký hoạt động;
+ Người đứng đầu tổ chức hoặc người được người đứng đầu tổ chức ủy
quyền có Thẻ giám định viên sở hữu trí tuệ.
- Cá nhân có đủ các điều kiện sau đây được cơ quan nhà nước có thẩm quyền cấp Thẻ
giám định viên sở hữu trí tuệ:
+ Là công dân Việt Nam, có năng lực hành vi dân sự đầy đủ;
+ Thường trú tại Việt Nam;
+ Có phẩm chất đạo đức tốt;
+ Có trình độ đại học trở lên về chuyên ngành phù hợp với lĩnh vực đề nghị cấp thẻ
giám định, đã qua thực tế hoạt động chuyên môn trong lĩnh vực đó từ năm năm
trở lên và đạt yêu cầu kiểm tra nghiệp vụ về giám định.
- Cơ quan nhà nước có thẩm quyền xử lý hành vi xâm phạm quyền sở hữu trí tuệ có quyền
trưng cầu giám định về sở hữu trí tuệ khi giải quyết vụ việc mà mình đang thụ lý.
- Chủ thể quyền sở hữu trí tuệ và tổ chức, cá nhân khác có liên quan có quyền yêu cầu
giám định về sở hữu trí tuệ để bảo vệ quyền và lợi ích hợp pháp của mình.

98. Phân tích quyền và nghĩa vụ chứng minh của đương sự trong vụ kiện xâm phạm
quyền sở hữu trí tuệ?
- Nguyên đơn và bị đơn trong vụ kiện xâm phạm quyền sở hữu trí tuệ có quyền và nghĩa
vụ chứng minh theo quy định tại Điều 79 của Bộ luật tố tụng dân sự và theo quy định tại
Điều này, bằng một trong các chứng cứ sau :
+ Bản sao Giấy chứng nhận đăng ký quyền tác giả, Giấy chứng nhận đăng
ký quyền liên quan, văn bằng bảo hộ; bản trích lục Sổ đăng ký quốc gia về
quyền tác giả, quyền liên quan, Sổ đăng ký quốc gia về sở hữu công
nghiệp, Sổ đăng ký quốc gia về giống cây trồng được bảo hộ;
+ Chứng cứ cần thiết để chứng minh căn cứ phát sinh quyền tác giả, quyền
liên quan trong trường hợp không có Giấy chứng nhận đăng ký quyền tác
giả, Giấy chứng nhận đăng ký quyền liên quan; chứng cứ cần thiết để
chứng minh quyền đối với bí mật kinh doanh, tên thương mại, nhãn hiệu
nổi tiếng;
+ Bản sao hợp đồng sử dụng đối tượng sở hữu trí tuệ trong trường hợp
quyền sử dụng được chuyển giao theo hợp đồng.
- Nguyên đơn phải cung cấp các chứng cứ về hành vi xâm phạm quyền sở hữu trí tuệ hoặc
hành vi cạnh tranh không lành mạnh.
- Trong vụ kiện về xâm phạm quyền đối với sáng chế là một quy trình sản xuất sản phẩm,
74
bị đơn phải chứng minh sản phẩm của mình được sản xuất theo một quy trình khác với
quy trình được bảo hộ trong các trường hợp sau đây:
+ Sản phẩm được sản xuất theo quy trình được bảo hộ là sản phẩm mới;
+ Sản phẩm được sản xuất theo quy trình được bảo hộ không phải là sản phẩm mới
nhưng chủ sở hữu sáng chế cho rằng sản phẩm do bị đơn sản xuất là theo quy
trình được bảo hộ và mặc dù đã sử dụng các biện pháp thích hợp nhưng vẫn
không thể xác định được quy trình do bị đơn sử dụng.
- Trong trường hợp một bên trong vụ kiện xâm phạm quyền sở hữu trí tuệ chứng minh
được chứng cứ thích hợp để chứng minh cho yêu cầu của mình bị bên kia kiểm soát do
đó không thể tiếp cận được thì có quyền yêu cầu Tòa án buộc bên kiểm soát chứng cứ
phải đưa ra chứng cứ đó.
- Trong trường hợp có yêu cầu bồi thường thiệt hại thì nguyên đơn phải chứng minh thiệt
hại thực tế đã xảy ra và nêu căn cứ xác định mức bồi thường thiệt hại theo quy định tại
Điều 205 của Luật này.

99. Phân tích những đặc thù của việc áp dụng biện pháp khẩn cấp tạm thời trong vụ kiện
xâm phạm quyền sở hữu trí tuệ so với vụ án dân sự thông thường?

100. Phân tích nguyên tắc xác định thiệt hại do xâm phạm quyền sở hữu trí tuệ
1. Thiệt hại do hành vi xâm phạm quyền sở hữu trí tuệ bao gồm:
a) Thiệt hại về vật chất bao gồm các tổn thất về tài sản, mức giảm sút về thu nhập, lợi nhuận, tổn
thất về cơ hội kinh doanh, chi phí hợp lý để ngăn chặn, khắc phục thiệt hại;
- Được coi là có tổn thất thực tế nếu có đủ các căn cứ sau đây:

+ Lợi ích vật chất hoặc tinh thần là có thực và thuộc về người bị thiệt hại;

+ Người bị thiệt hại có khả năng đạt được lợi ích quy định tại điểm a khoản này;

+ Có sự giảm sút hoặc mất lợi ích của người bị thiệt hại sau khi hành vi xâm phạm
xảy ra so với khả năng đạt được lợi ích đó khi không có hành vi xâm phạm và
hành vi xâm phạm là nguyên nhân trực tiếp gây ra sự giảm sút, mất lợi ích đó.

Tổn thất về tài sản được xác định theo mức độ giảm sút hoặc bị mất về giá trị tính được
thành tiền của đối tượng quyền sở hữu trí tuệ được bảo hộ.
- Tổn thất về cơ hội kinh doanh bao gồm:
+ Khả năng thực tế sử dụng, khai thác trực tiếp đối tượng quyền sở hữu trí tuệ trong
kinh doanh;

75
+ Khả năng thực tế cho người khác thuê đối tượng quyền sở hữu trí tuệ;
+ Khả năng thực tế chuyển giao quyền sử dụng đối tượng quyền sở hữu trí tuệ,
chuyển nhượng đối tượng quyền sở hữu trí tuệ cho người khác
+ Cơ hội kinh doanh khác bị mất do hành vi xâm phạm trực tiếp gây ra.

b) Thiệt hại về tinh thần bao gồm các tổn thất về danh dự, nhân phẩm, uy tín, danh tiếng và
những tổn thất khác về tinh thần gây ra cho tác giả của tác phẩm văn học, nghệ thuật, khoa học;
người biểu diễn; tác giả của sáng chế, kiểu dáng công nghiệp, thiết kế bố trí, giống cây trồng.
2. Mức độ thiệt hại được xác định trên cơ sở các tổn thất thực tế mà chủ thể quyền sở hữu trí tuệ
phải chịu do hành vi xâm phạm quyền sở hữu trí tuệ gây ra.

101. Phân tích căn cứ xác định mức bồi thường thiệt hại do xâm phạm quyền sở hữu trí
tuệ
- Trong trường hợp nguyên đơn chứng minh được hành vi xâm phạm quyền sở hữu trí tuệ
đã gây thiệt hại về vật chất cho mình thì có quyền yêu cầu Tòa án quyết định mức bồi
thường theo một trong các căn cứ sau đây:
+ Tổng thiệt hại vật chất tính bằng tiền cộng với khoản lợi nhuận mà bị đơn đã thu
được do thực hiện hành vi xâm phạm quyền sở hữu trí tuệ, nếu khoản lợi nhuận bị
giảm sút của nguyên đơn chưa được tính vào tổng thiệt hại vật chất;
+ Giá chuyển giao quyền sử dụng đối tượng sở hữu trí tuệ với giả định bị đơn được
nguyên đơn chuyển giao quyền sử dụng đối tượng đó theo hợp đồng sử dụng đối
tượng sở hữu trí tuệ trong phạm vi tương ứng với hành vi xâm phạm đã thực hiện;
+ Thiệt hại vật chất theo các cách tính khác do chủ thể quyền sở hữu trí tuệ đưa ra
phù hợp với quy định của pháp luật;
+ Trong trường hợp không thể xác định được mức bồi thường thiệt hại về vật chất
theo các căn cứ quy định tại các điểm a, b và c khoản này thì mức bồi thường thiệt
hại về vật chất do Tòa án ấn định, tùy thuộc vào mức độ thiệt hại, nhưng không
quá năm trăm triệu đồng.
- Trong trường hợp nguyên đơn chứng minh được hành vi xâm phạm quyền sở hữu trí tuệ
đã gây thiệt hại về tinh thần cho mình thì có quyền yêu cầu Tòa án quyết định mức bồi
thường trong giới hạn từ năm triệu đồng đến năm mươi triệu đồng, tùy thuộc vào mức độ
thiệt hại.
- Ngoài khoản bồi thường thiệt hại quy định tại khoản 1, khoản 2 Điều này, chủ thể quyền
sở hữu trí tuệ có quyền yêu cầu Tòa án buộc tổ chức, cá nhân có hành vi xâm phạm
quyền sở hữu trí tuệ phải thanh toán chi phí hợp lý để thuê luật sư.

76

You might also like